You are on page 1of 220

Tables of CONTENTS

Chapters Page no 

1. Alphabetical Series 2 - 12
2. Coding Decoding 13 - 44
3. Inequalities 45 - 59
4. Blood relation 60 - 76
5. Distance and Direction 77 - 90
6. Order and Ranking 91 - 96
7. Syllogism 97 - 112
8. Seating Arrangement 113 - 143
9. Puzzle 144 - 154
10. Machine Input Output 155 - 173
11. Data Sufficiency 174- 181
12. Statement and Assumption 182-190
13. Statement and Argument 191 -200
14. Statement and conclusion 201 - 205
15. Course of Action 206 -211
16. Decision Making 212 - 213
17. Cause and Effect 214 -217
18. Combined Reasoning 218 -220

Total no. of Pages 220

Plutus Academy-(1)
​ ​Alphabetical Series 
To solve the problem of alphabets ,we should know about the position of alphabets from 
beginning to end. 

Left to right(1-26) 
1  2  3  4  5  6  7  8  9  10  11  12  13 

A  B  C  D  E  F  G  H  I  J  K  L  M 

26  25  24  23  22  21  20  19  18  17  16  15  14 

14  15  16  17  18  19  20  21  22  23  24  25  26 

N  O  P  Q  R  S  T  U  V  W  X  Y  Z 

13  12  11  10  9  8  7  6  5  4  3  2  1 


Right to left (26-1) 

Trick to remember alphabets: 


E J O T Y C F I L O R U X 
5 10 15 20 21 3 6 9 12 15 18 21 24 

(Left side)​A B C D E F G H I J K L M N O P Q R S T U V W X Y Z ​(Right side) 

Reverse place value (Position of alphabet from right) = 27 - Forward place value(Position of 
alphabet from left)  

TRICK TO REMEMBER REVERSE OF ALPHABETS: 


A-Z (Amazon)
B-Y (Boy)
C-X (Crax)
D-W (Dew)
E-V (Even)
F-U (Fun)
G-T (GST)
H-S (His)
I-R (Indian Railways)
J-Q (Jack Queen)
K-P (Kanpur)
L-O (Love)
M-N (Man)

Plutus Academy-(2)
 
Ex 1 : ​Which element is the 8th to the left of 17th element from the left side of the alphabetical 
series.? 
Short trick:​ Like - Subtraction 
Unlike - Addition 
 
From which side  To the left/right  ACTION TO BE TAKEN 

RIGHT  RIGHT  SUBTRACT(From right side) 

LEFT  LEFT  SUBTRACT(From left side) 

RIGHT  LEFT  ADDITION (From right side) 

LEFT  RIGHT  ADDITION(From left side) 


 
Sol: ​Since in question, both is left (i.e like sides), hence subtraction: 
= (17-8)th from left side  
= 9th from left side 
= I 
 
Ex 2 : ​Which element is the 8th to the right of 17th element from the right side of the 
alphabetical series.? 
Sol: ​Since in question, both is right(i.e like sides), hence subtraction, 
= (17-8)th from right side  
= 9th from right side 
= (27-9)th from left side 
= 18th from left side 
= R 
 
Ex 3 : ​Which element is the 8th to the right of 17th element from the left side of the alphabetical 
series.? 
Sol: ​Since in question, one is right and the other is left (i.e unlike sides), hence addition, 
= (17+8)th from left side  
= 25th from left side 
= Y 
 
Ex 4 : ​Which element is the 8th to the left of 17th element from the right side of the alphabetical 
series.? 
Sol: ​Since in question, one is right and the other is left (i.e unlike sides), hence addition, 
= (17+8)th from right side  
= 25th from right side 
= (27-25)th from left side 
= 2nd from left side 

Plutus Academy-(3)
= B 
 
Ex 5 : ​J Y 2 = S $ D E G M * 7 & H P 9 K L b @ W Q 1 3 # C D % 
1. If all the numbers and symbols are dropped from the above series, which element is 4th 
to the left of 5th element from the right side of the series.? 
Sol: H 
 
2. How many such symbols are there in the above arrangement each of which is either 
followed by a number or preceded by a letter but not both. 
Sol: ​3 
 
3. How many such letters are there in the above arrangement each of which is either 
followed by a number or preceded by a number but not both. 
Sol: ​4 
 
​4. If 2YS is related to ED$, what is PHK related to in the same way? 
Sol: ​@bL 
 
Complete the missing Series:  
 
Ex-1 APZ BQY CRX DSW ? 
a) ETV b) DXV c) FWXd) EVX e)None 
Sol: ​a)ETV 
 
Ex-2 Z WV SRQ NMLK ? 
a) HEFED b) HGFED c) JKLMN d) GHLKJ e) None 
Sol: ​b) HGFED 
 
Ex-3 10za 8yb 6xc ? 2ve 
a) 3wd b) 5we c) 4dw d) 4wd e) None 
Sol: ​d) 4wd  
 
ODD ONE OUT: 
 
Ex-1​ MNPK, STVQ, FGID, VWYT, JKMI 
Sol: ​JKMI as there are 3 letters between the last two letter(i.e M and I) unlike in the other words 
where there are 4 letter each between the last two letters. 
 
Ex-2 ​XVTU, LJHI, POMN, FDBC, MKIJ 
Sol: ​POMN. 
There is one letter between 1st, 2nd and 3rd letter each. But this pattern is not followed in 
POMN. 

Plutus Academy-(4)
Pair formation  
Ex1 :​ How many pairs of letters are there in the word ​STUDENT​ , which have the same number 
of letters between them as in the english alphabets? 

 
We have to find alphabetical order of letters such as after “​S​” “​T​” is come...and after that “U” 
come. And so on. 5 pairs are formed while comparing letters. 
  
Ex2 : ​How many such pairs of letters are there in the word ​OBILEAUTO​ , which have the same 
number of letters between them in the word as in the english alphabets? 

Sol:  
5 Pairs 
 
 
 
 
 
 
 
 
 
 
 
 
 
 
 
 
 

Plutus Academy-(5)
EXERCISE 
1.Which  letter  is  fifth  to  the  right  of  the  eighteenth  letter  from  your  right  in  the  alphabetical 
series? 
1)N  2)M  3)E  4)C  
   
2.  If  the  alphabets  are  arranged  in  the reverse order, Which letter will be twelfth to the left of the 
sixteenth from your left ? 
1)X  2)W  3)V  4)U 
 
3. If all the vowels are removed from the alphabet , which letter will be the seventh to the right of 
the fifth letter from the left ? 
1)L 2)V  3)J  4)P 
 
4. If A and B written as B and A, C and D is written as D and C and so on.What will be the 
position of M from your right ? 
1)11  2)12 3)13 4)14 
 
5. If every alternative letter starting with A, is removed from the alphabet ,Which letter among 
the remaining letters would be the third to the right of the fifth letter from the right ? 
1)J 2)L 3)V 4)X 
 
6. How many meaningful letters can be made from the word ESDO by using each letter only 
once? 
1) One 2) two 3) three 4)none 
 
7. How many meaningful letters can be made from the word EIDT by using each letter only 
once? 
1) ONE 2)TWO 3) THREE 4) FOUR 
 
8. How many meaningful letters can be made from the word EDNO by using each letter only 
once? 
1) ONE 2)TWO 3) THREE 4) FOUR 
 
9. How many meaningful letters can be made from the word EOPR by using each letter only 
once? 
1) ONE 2)TWO 3) THREE 4) FOUR 
 
10.How many meaningful letters can be made from the word INATS by using each letter only 
once? 
1) ONE 2)TWO 3) THREE 4) FOUR 
 
(11-15).​ ​M 3 R # A P 4 9 K % D 1 U H @ J 2 N ^ W E Q 5 T 6 $ V 7 * 1 F 8 B Y  
11. Which of the following is the eighth to the left of the 21st from the right end ? 
1)%  2)P  3)H  4)B 
 
12. If all the number from the above arrangement are removed then what will be the 7th from 
the right end ? 

Plutus Academy-(6)
1)@  2)Q  3)T  4)V 
 
13. Three of the following four are alike in a certain way based on their position. Which is the 
one that does not belong to that group ? 
1)YBF  2)VTQ  3)EWN  4)HUD 
  
14. How many such symbols are there in the above arrangement each of which is immediately 
followed by the number and preceded by the vowels ? 
1)None  2)One 3)Two 4)Three 
 
15. How many such number in the above arrangement each of which is immediately followed by 
the letter and preceded by the Number? 
1)None  2)One  3)Two  4)Three 
 
(16-20) ​H 3 R % M A $ K 2 P 5 E ^ N 4 W @ F & Q 1 U / 9 J I D 7 8 
16. Which of the following is the fifth to the right of the 12th from the left end of the above 
arrangement ? 
1)$  2)@ 3)/ 4)W 
 
17. If all the numbers are dropped from the above arrangement then what will be the 10th from 
the right end ? 
1)W  2)@  3)/  4)N 
 
18. How many such symbol in the above arrangement that is either immediately preceded by 
letter or immediately followed by a number but not both ? 
1)More than three  2)One 3)Two 4)Three 
 
19. What should come in the following series based on the above arrangement ? 
HR% AK2 5^N ? 
1)@Q1  2)F&Q 3)W@F  4)WF& 
 
20. How many such numbers are there in the above arrangement, each of which are 
immediately preceded by letter and immediately followed by letter ? 
1)None 2)One 3)Two 4)More than Three 
 
21.The letters in the word ​PROTEIN​ are arranged in such a way that the consonants are 
arranged in alphabetical and then the vowels are arranged in alphabetical order. 
How many letters will be ter between R and I after the rearrangement. 
1) None 2) one 3) three 4) two 
 
22. Which of the following will be the third from the right after rearrangement? 
1) T 2) O 3) None 4) I 
 
23. Which of the following will be third from the left after rearrangement? 
1) R 2) N 3) O 4) P 
 
24. Which of the following will be 4th from the left to the 2nd from the right after 
rearrangement? 

Plutus Academy-(7)
2) R 2) N 3) O 4) P 
 
25​. BMRG, DLTF, FKVE, HJXD, ? (SSC CGL Tier-I 2011) 
a) JIZC b) JZIBc) GIFBd) MOLC 
 
26​. A, CD, GHI, ? , UVWXY (SSC CGL Tier-I 2014) 
a) LMNP b) MNOL c) NOPL d) MNOP 
 
27. FAK IEM LIO  ? (SSC CGL Tier-I 2015) 
a) OMQ b) OPQ c) MNO d) NOP 
 
28. ​What should come in place of question mark (?) in the following series based on the above 
arrangement? (IBPS PO Pre 2017) 
BED EIG HMJ KQM ? 
(a) PUN (b) OUQ (c) NUQ (d) NUP (e) NUR 
 
29.  ​Consider  the  word  “YOURSELF”,  arrange  the  vowels  in  alphabetical  order  from  left  to  right 
and  then  consonant  letters  in  alphabetical  order  from  left  to  right.  Now,  take  the  next  alphabet 
for each letter in the changed arrangement. Which letter comes fourth from the right side.   
(IBPS PO Pre 2017)  
(a) N (b) M (c) L (d) S (e) R 
 
30.  If  it  is  possible  to  make  only  one  meaningful  word  with  the  second,third,  sixth,and  eighth 
letters  of  the  word  ​FRAGMENTS  using  each letters only once, which of the following will be the 
third  letter  of  that  word?  If  no  such  word  can  be  formed,  give  X  as  answer  and  more  than  one 
such word can be formed, give “Y” as answer? 
1) A 2) X 3) Y 4) E 
 
31.If  it  is  possible  to  make  only  one  meaningful  word  with  the  third,  sixth,  ninth  and  twelfth 
letters  of  the  word  ​ELECTRIFICATION   using  each  letters  only  once,  which  of  the  following  will 
be  the  third  letter  of  that  word? If no such word can be formed, give N as answer and more than 
one such word can be formed, give “D” as answer? 
1) R 2)N 3) D 4) S 
 
32.How  many  meaningful  english  words  can  be  formed  with  the  letters  of  the  word  “  STOP” 
each using only once in a word but in different sequence, starting with letter P. 
1) One 2) two 3) three 4)none 
 
33.  Each  vowel  in  the word “JOURNEY” is replace by the previous letter in the english alphabets 
and  each  consonant  is  replaced  by  the  next  letters  in  the  alphabets,  then  arrange  the  letters 
alphabetically . which of the following will be the fifth from the left end? 
1) None 2) D 3) K 4)O 
 
34.  If  it  is  possible  to  make  only  one  meaningful  word  with  the  FIRST,sixth,eighth  and  ninth 
letters  of  the  word  ​HOARDINGS  using  each  letters  only  once,  which  of  the  following  will be the 
third  letter  of  that  word?  If  no  such  word  can  be  formed,  give  X  as  answer  and  more  than  one 
such word can be formed, give “Y” as answer? 

Plutus Academy-(8)
1) G 2) X 3) Y 4) I 
 
35. ​A B C D E F G H I J K L M N O P Q R S T U V W X Y Z 
Which letter in this alphabetical series is the eighth letter to the right of the letter, which is tenth 
letter to the left of the second last letter of the alphabet? 
1)X 2)W 3)I 4)H 
 
36. What should come in place of question mark (?) in the following series based on the above 
arrangement? (IBPS RRB Pre 2017)  
ZB XD UG QK ? 
(a) LK (b) LO (c) LP (d) KP (e) Other than the given options  
 
37. How many such pair of numbers are there in the given number “46579739” (Both backward 
and forward) same as far as according to numeric series? (IBPS RRB Pre 2017)  
(a) One (b) Two (c) Three (d) More than three (e) None of these.  
 
38. If it is possible to make only one meaningful word with the 1st ,2nd ,4th and 7th letters of the 
word  ‘ECUADOR’  which  would  be  the  second  letter  of  the  word  from the right? If more than one 
such  word  can  be  formed  give  ‘Y’  as the answer. If no such word can be formed, give ‘Z’ as your 
answer. (IBPS RRB Pre 2017)  
(a) Y (b) E (c) I (d) Z (e) M  
 
39.  If  1  is  subtracted  from  each  odd  number  and  2  is  added  to  each  even  in  the  number 
9436527, then how many digits will appear twice in the new number thus formed?  
(IBPS RRB Pre 2017)  
(a) Only 8 (b) Only 8 and 6 (c) 8, 6 and 4 (d) 2, 4 and 6 (e) None of these  
 
40. How many letter will be remain the same position in the word ‘MONSTER’ when they 
arranged in the ascending order from left to right? (IBPS RRB Pre 2017)  
(a) One (b) Two (c) Three (d) More than Three (e) None 
 
41. ​Direction: How many pairs of letters are there in the following words , which have the same 
number of letters between them in the word as in the english alphabets? 
 
1.COMPUTER 
a) 1   b) 3 c) 4 d) none 
2. CONTROVERSY 
a) 3 b) 5 c) 6 d) none 
3. BEAUTIFUL 
a) 1 b) 2 c)4 d) none 
4. MEDITATION 
a) 1 b) 2 c)4 d) none 
5. POPULATION 
a) 1 b) 2 c)5 d) more than 5 
6. TECHNOLOGY 
a) 1 b) 3 c)4 d) none 

Plutus Academy-(9)
7. POPULAR 
a) 1 b) 2 c)4 d) more than 4 
8. MISTRY 
a) 1 b) 2 c)4 d) none 
9. COMMERCIAL 
a) 7 b) 2 c)4 d) none 
10. PRIYANKA 
a) 1 b) 2 c)4 d) none 
11. SAURABH 
a) 1 b) 2 c)4 d) none 
12. DEEPALI 
a) 1 b) 2 c)4 d) none 
13. SUPPORT 
a) 1 b) 2 c)5 d) none 
14. ROMANTIC 
a) 1 b) 2 c)4 d) none 
15. DIGITIZER 
a) 1 b) 2 c)4 d) none 
16. EXPRESS 
a) 1 b) 2 c)4 d) none 
17.DOCUMENT 
1 b) 2 c)4 d) none  
18. RECOVERED 
a) 3 b) 4 c)2 d)5 
19. NUTRITION 
a) 4 b) 5 c) 6 d) 7 
 
42. How many digits are there in the number ​367849​ each of which is as far away from the 
beginning of the number as when the digits are arranged in descending order within the number.   
a) 1 b) 2 c)4 d) none  
 
43. How many digits are there in the number ​243896715​ each of which is as far away from the 
beginning of the number as when the digits are arranged in ascending order within the number. 
a)1 b) 2 c)3 d) none  
 
 
 
 
 
 
 
 
 

Plutus Academy-(10)
Answer sheet 
1. 1 
2. 2 
3. 4 
4. 3  
5. 4 
6. 2 (DOES, DOSE) 
7. 4 (DIET, EDIT, TIDE, TIED) 
8. 2 (NODE, DONE) 
9. 2 (ROPE,PORE) 
10. 2 (STAIN,SAINT) 
11. 2)P 
12. 3)T 
13. 2)VTQ 
14. 1)None 
15. 2)One 4 9 K 
16. 2)@ 
17. 1)W 
18. 1)More than three 
19. 4)WF& 
20. 4)More than Three 
21. 4) two ​NPRTEIO 
22. 3) None 
23. 1) ​R 
24. 4) P 
25. a) ​JIZC 
26. d) MNOP 
27. a) ​OMQ 
28. d) NUP 
29. (b) M 
30. 3) ​TEAR,RATE Thus,ANS is 'Y'. 
31. 3) D (TIRE, TIER and RITE) 
32. 2) two POTS and SPOT 
33. 1) ​None 
34. 3) Y (SIGN, SING) 
35. 2​ ​In the given alphabet, last but one letter of alphabet is Y. 10th letter to the left 
of Y is O, 8th letter to the right of O is W. 
36. (c) LP 
37. (d) More than three 
38. (a) Y( RACE, CARE) 
39. (c) 8, 6 and 4 
40. (a) One 
41.  
41.1. C 
41.2. C 

Plutus Academy-(11)
41.3. C 
41.4. C 
41.5. D 
41.6. B 
41.7. D 
41.8. A 
41.9. A 
41.10. A 
41.11. B 
41.12. B 
41.13. C 
41.14. B 
41.15. B 
41.16. B 
41.17. D 
41.18. B 
41.19. 7 
42. Sol. 987643 means only 2 digits are same when arranged in descending order. 
43. c)3 
 
 
  
 
 
 
 
 
 
 
 
 
 
 
 
 
 
 
 
 
 
 
 
 
For video solutions: ​https://www.youtube.com/watch?v=lzadplZo2-A 
 

Plutus Academy-(12)
Coding Decoding 
 
In  any  communication(verbal  or  non-verbal),  there  are  always  two  parties  i.e.  Sender  (The  one 
who  wants  to  send  the  data)  and Receiver(one who receives the data).We always want to send 
our  data  securely.  For  this  purpose,  we  use  the  mechanism of coding and decoding so that the 
third  party  (other  than  the  sender  and  receiver)  can  never  understand  the  communication  of 
both  the  parties.Coding  means  transforming  the  original  data  into  an  encrypted  form  which 
follows  a  certain  pattern.  Coding  is  also  known  as  ​encryption​.Decoding  means  converting  the 
coded data into an original form. It is also known as ​Decryption​. 
 
The  coding  and  decoding  test  is  setup  to  determine the ability of a candidate that he is capable 
to  decipher  any  code  or  message.  In  english  alphabetically  coding  search  the  code  and  try  to 
find the pattern. 
 
Opposite Alphabet Coding:​ ​Pair of alphabets is also given as codes. 
A-Z (Amazon) 
B-Y (Boy) 
C-Z (Crax) 
D-W (Dew) 
E-V (Even) 
F-U (Fun) 
G-T (GST) 
H-S (His) 
I-R (Indian Railways) 
J-Q (Jack Queen) 
K-P (Kanpur) 
L-O (Love) 
M-N (Man) 
 
Types of Coding 
1). Letter Coding 
2). Number Coding 
3). Conditional Coding 
4). Substitutional Coding 
 
Type 1-Letter Coding​: ​In letter coding, letters are coded in letters by using different 
patterns or sequence. 
 
Ex.  1​.  In  a  certain  code  language,  ​TEACHER  is  written  as  ​UGDGMKY​,  then  how  is  STUDENT 
written in the same code language? 
Sol.   

Plutus Academy-(13)
 
Ex.  2​.  In  a  certain  code  ‘​MONARCH​’  is  written as ‘​NPOBSDI​’. How will ‘​STANDARD​’ be written in 
that code? 
 
Sol​:  In  this  question,  each  letter  of  ‘MONARCH’  is  simply  replaced  by  its  next  letter  as  per 
English Alphabet. 

M + 1 = N; 

O + 1 = P; 

N + 1 = O; 

A + 1 = B; 

R + 1 = S; 

C + 1 = D; 

H + 1 = I;. 

Based on similar pattern, code for ‘CITIZEN’ is ‘DJUJABO’. 

Ex 3​: In a certain code ‘​ARCHERY​’ is written as ‘​DSBGZSF​’. How will the word ‘​TERMITE​’ be 
written in that code? 

Sol​: 

1. Write down the letters in one line and its code in the line below. 

2. Analyze the coding pattern by matching the code with the word. 

Plutus Academy-(14)
 

We  can  see  that  the  word  has  been  divided  into  three  parts,  where  letters  of  the  first  and  third 
parts  are  increased  by  1  position  and  then  reversed  among  themselves  while  the  lone  letter  in 
the middle part is decreased by 1. 

NOTE:  Here  increasing  or  decreasing  by  ‘n’  place  means,  exchanging  the  current  letter  with  a 
letter that is ‘n’ places to the right or left, respectively, in the English Alphabet. 

3. Once the pattern has been identified, find the code for the word asked in the question: 

Hence, code for TERMITE is ‘SFULFUJ’ 

Type  2-Number  Coding:  ​In  number  coding,  letters  are  coded  in  numbers  by  using 
different patterns and sequence. 

There are two coding mechanism that is used in these type of questions. 
 

Plutus Academy-(15)
i) Alphabetical code letters are assigned to the numbers  
 
OR 
 
ii) Numerical code values are assigned to a word. 
 
i)Alphabetical code letters are assigned to the numbers- 
Ex1.​ In a code Language the following alphabets are coded in a particular way. 
 
V  U  N  L  J  E  A  T  K  P 

1  5  8  6  3  4  7  2  9  0 
Which group of alphabets can be decoded from the following? 
762539 
1) ALTJUK 2) ALTUKJ 3) ALTVJK 4) ALTUJK 
 
Sol: ​4) ALTUJK 
 
 
Ex 2​. In a certain code PEN is Coded as 123,PENCIL as 123456,CABLE as 48962, then  
6283123456 means what?  
 
1) LAENPENCIL 2) LEANPNCLI 3) LANPENCIL 4) LEANPENCIL 
Sol: ​4) LEANPENCI 
 
ii)numerical code values are assigned to a word- 
Ex  3.  ​In  a  certain  code  language,  ​ASKED  is  written  as  ​45211​,  then  how  is  ​EIGHT  written  in the 
same code language? 

1) 59782 2) 28795 3) 597820 4) 95728 5) None of these 

Sol: 

Plutus Academy-(16)
Ex 4. ​If BROAD means 19812.Then,CLOCK means  
1) 68262 2) 68622 3) 26826 4) 37836 
 
Sol: ​3) 26826 

Ex  5.  ​In  a  certain  code  language,  ​PERSON  is written as ​957561​, then how is ​SHOULD written in 


the same code language? 

1) 186334 2) 198152 3) 681433 4) 861334 5) None of these 

Sol: 

  

Type  3-  Conditional  Coding:  ​In  conditional  coding,  few  conditions  are  given  in  the 
question, we have to follow the same to code the characters. 

Ex1. Study the following information to answer the given questions. 

Letter  M  I  K  T  A  B  E  J  W  F  H  U  D  P  Y 

Code  6  *  5  ©  7  8  @  1  2  3  4  %  9  #  $ 

Condition: 

i)If  the  first  letter  is  a  vowel  and  the  last  letter  is  a  consonant,  the  codes  are  to  be 
interchanged. 

ii)If  the  first  letter  is  consonant  and  the  last  letter  is  a  vowel,both  are to be coded as the code 
for the vowel. 

Plutus Academy-(17)
iii)If both the first and the last letters are vowels, both are to be coded as ‘&’. 

Q1. MEAPTD 

1) 6@7#©9 2) 6@7#©6 3) 9@7#©9 4) 9@7#©6 5) None of these 

Sol:​ ​1) 6@7#©9 

Q2​. ​BMJKPU 

1) %615#8 2)%615#% 3)8615#% 4)8615#8 5)None of these 

Sol: ​2) %615#% 

Q3. ​IPTWHY 

1) $#©24$ 2) $#©24* 3) *#©24$ 4) *#©24* 5)None of these 

Sol:​2) $#©24* 

Q4. ABJFEP 

1) 7813@7 2) 7813@# 3) #183@7 4) #813@7 5)None of these 

Sol: ​4) #813@7 

Q5. EYBEJA 

1) &$8@1& 2) @$8@17 3) 7$8@1@ 4) 7$8@1& 5) None of these 

Sol:​ 1) &$8@1& 

TYPE 4 -SUBSTITUTIONAL CODING 


 
Ex 1​. If air is called ‘green’,’green’ is called ‘blue’,’blue is called ‘sky’,’sky’ is called ‘yellow’,’yellow’ 
is called ‘water’ and ‘water’ is called ‘pink’ then what is the colour of clear ‘sky’?  
1) Sky 2) Water 3) Blue 4) Yellow 
 
Sol:​ Sky 
 
Ex 2​. In a certain code language FIRE is written as #%@$ and DEAL is written as !$*^. How is 
FAIL Written in that code Language? 
1) #*%^ 2) #$%^ 3) #*!$ 4) #*!^ 5)None of these 

Plutus Academy-(18)
 
Sol: ​#*%^  
 
Ex 3​ In a certain code, 
‘786’ means ‘study very hard’, 
‘958’ means ‘hard work pays’ 
‘645’ means ‘study and work’. 
Which of the following is the code for ‘very’? 
1. 7 
2. 5 
3. 8 
4. 6 
5. Either 6 or 7 
Sol​: A 
Explanation: 
In the first and second statements, the common word is ‘hard’ and the common code digit is ’8’. 
So, ‘8’ means ‘hard’. 
In the first and third statements, the common word is ‘study’ and the common code digit is ‘6’. 
So, ’6’ means ‘study’. 
Thus, in the first statement ‘7’ means ‘very’ 
 
Ex 4. ​Study the following information carefully and answer the questions given below: 
In a certain code language 
“tell them young”​ means ​“ke la pa” 
“wise young sharp tell” ​means ​“na pa kup ke”  
“bring clever young them” ​means “ ​se to pa la​” 
“​clever sharp young tomorrow​” means “​to kup pa jo​” 
 
1. What is the code for “​sharp​” ? 
1) pa 2) na 3) ke 4) kup 5) la 
Sol: ​4) kup 
 
2. What does ​“To” ​stand for? 
1) clever 2) young 3) bring 4) them 5) Can’t be determined  
Sol: 1) ​clever  
 
3. Which of the following represents ​“Tell tomorrow” 
1) la jo 2) in ke 3) ke jo 4) jo in 5) pa ke 
Sol: ​3) ke jo  

Plutus Academy-(19)
 
4. What is the code for “​Clever young girl​”  
1) be ve pa 2) to pa be 3) la na to 4) to pa jo 5) ke ni to 
Sol: ​2) to pa be 
 
5. Which of the following code represents “​kup la so na​” 
1) wise them sharp tell   
2) young tell sharp them 
3) some wise them and 
4) clever wise them sharp 
5) sharp them not wise 
Sol: ​sharp them not wise 
Ex 5: ​Study the following information carefully and answer the questions given below: 
In a certain code language 
 
‘given time single plan’ is written as ‘@E4 &N4 %N5 #E6’ 
‘timed solution plant great’ is written as ‘#N8 @D5 %T5 &T5’ 
‘sick point good turn’ is written as ‘#K4 %D4 @N4 &T5’ 
‘garden sister phone team’ is written as ‘&E5 #R6 %N6 @M4’ 
 
1.Which of the following code for ‘translate’? 
A. @E8 
B. @E9 
C. #E8 
D. #T8 
E. #T9 
Sol​: B. @E9 
 
Here are some points to keep in minds when solving these kinds of questions- 
■ There  is  always  a  concept  behind  this  so  we  need  to  find  it.  When  we  look  at  the 
questions  we  can  see  that  in  all  steps  all  the  4  words start with same alphabet. They all 
start  with-  G,  T,  S  and  P  and  their  code  words  also  starts  with  same  symbol  in  each 
steps-  @,  %,  &  and  #.  So  we  just  have  to  find  which  of  the  first  alphabet  represent  the 
particular symbol. 
■ Now  we  can  see  that  their  code  words  end  with  digits.  The  digits  are  total  number  of 
letters for each word. 
■ Now we can see the middle code word is last letter of that particular word. Now we solve 
the questions.  
 
2. Which of the following code for ‘plane’? 
A. %N5 
B. @E4 
C. &E5 
D. #E6 
E. None of these 

Plutus Academy-(20)
Sol:​ &E5 
Total count of letters is 5 and last letter is E and ‘p’ is coded as & so Final answer will be- &E5 
 
3. In the given code language, what does the code ‘%D4’ stands for? 
A. point 
B. turn 
C. sick 
D. good 
E. None of these 
Sol:​ D. good 
 
Ex 6: Study the following information to answer the given questions: 

​In a certain code, 

‘superstring park avail prelim’ is written as ‘#22T %8Q $10B @12Q’, 


‘cereal revolving crook newsroom’ is written as ‘#10S @16O $12D %10D’, 
‘training system unequal talk’ is written as ‘$14V %8U @12T #16U’, and 
‘payback dream using ethical’ is written as ‘%14Q $14F #10V @10E’. 
 
1. What is the code for ‘shooting upstream’? 

A) @16V %16T 

B) @16V #18T 

C) @16V #16T 

D) @16U %16T 

E) Cannot be determined 

Sol: ​Option C 

Symbols are according to last letter in words: 

g – #, k – %, l – $, m – @ 

Number represents double the number of letters in words 

Alphabet represents the next letter in alphabetical series to first letter of word. 

So superstring — # for last letter g, 22 for 2*11 (11 letters in superstring), and T for next letter 
to s 

So superstring – #22T 

2) What is the code for ‘making framework’? 

A) #12N %18G 

Plutus Academy-(21)
B) #12M $18G 

C) #12M %20G 

D) #12N @18G 

E) Cannot be determined 

Sol: ​A) #12N %18G 


 
3) What is the code for ‘logical coding’? 
A) @12M $12E 
B) #14M $12D 
C) @14M $12C 
D) #14M $12D 
E) Cannot be determined 
Sol: ​#14M $12D 
 
4) What is code for ‘environmental petroleum’? 
A) @18Q $24F 
B) @16Q $26E 
C) @18Q $26F 
D) #16Q $26E 
E) Cannot be determined 
Sol: ​C) @18Q $26F 
 
5)​ ​What is code for ‘maximum cashback’? 
A) #14N %16D 
B) @16N %16D 
C) @14N %16E 
D) @14N %16D 
E) Cannot be determined 
Sol: ​D) @14N %16D 
 
Ex 7: Study the following information to answer the given questions: 

In a certain code, 

‘letter found option result’ is written as ‘w#6 m@15 i#12 g@18’, 


‘input sneak never final’ is written as ‘o#6 p@19 i@14 g#9’, 
‘travel pirate height office’ is written as ‘g#8 v@16 o@20 v@15’, and 
‘stark mutual outward quantum’ is written as ‘w@15 n@17 o#13 p@19’. 
 
1) What is the code for ‘store gallery’? 

A) u#19 b#8 

B) v@18 c#7 

Plutus Academy-(22)
C) u@18 d#7 

D) v@19 b#7 

E) Cannot be determined 

Sol: ​Option D 
 
There  are  only  2  symbols  in  the  above  coding.  They  have  been  set  according  to  first  letter  of 
word. Words having numerical value less than equal to 13 are kept in one group and are denoted 
by #. and from 14-26 by @. (i.e. for first half letter (a-m) – #, and for last 13 letters (n-z) – @ 
Number represents the numerical value of first letter 
Alphabet  represents  the  reverse  of  last  letter  according  to their number in alphabetical series, a 
= 26, b = 25, …….z = 1 
So letter — r -> i, # for l and 12 for l 
So letter – i#12 
 
2) What is the code for ‘inshort phone’? 
A) g#9 v@18 
B) g#9 v@16 
C) h@10 v@18 
D) h#10 v#16 
E) Cannot be determined 
Sol: ​ B) g#9 v@16 
 
3) What is the code for ‘messages internet’? 
A) h@13 g#8 
B) h#13 g#9 
C) h#12 g@8 
D) h#13 g#8 
E) Cannot be determined 
Sol: ​B) h#13 g#9 
 
4) What is code for ‘settling clock’? 
A) p#13 t@19 
B) p#3 t@18 
C) p#3 t@19 
D) p@3 t#19 
E) Cannot be determined 
Sol: ​C) p#3 t@19 
 
5) What is code for ‘word money’? 
A) w@24 c#14 
B) v@22 b#13 
C) w@24 b#13 
D) w#24 c@13 
E) Cannot be determined 
 
Sol: ​C) w@24 b#13 

Plutus Academy-(23)
 
Ex 8:​ Directions (1-5): Study the following information to answer the given questions: 
In a certain code,  

‘Aspirant Time Stock Zone’ is written as ‘TP20 SG32 OV16 IV16’,  


‘Jump Marker Reach Against ’ is written as ‘ES20 UK16 GG28 AI24’,  
‘Refer Wave Further Belong’ is written as ‘AV16  ET24  UI28 EI20 ’, and  
‘Private Keen Child Red ’ is written as ‘HW20 RV28  EW12  EM16 ’. 
 
1) Which is the code for ‘Beyond Team ’?  

A) EW32 EN16 

B) EW24 AN20 

C) EW12 EN12 

D) EW24 EN16 

E) None of these 

Sol:​ Option D 
 
Numbers: Number of letters in word multiplied by 4 
First letter in code is second letter of respective word. Second letter in code is reverse of last 
letter of respective word. 
So Aspirant – S for second letter S, G for reverse of T, 8 letter word so 8*4 = 32 makes SG32  
 
2. What is the code for ‘Massive Alert’? 
A) AV28 LG20 
B) AW24 LG20 
C) AV28 LH20 
D) AW28 LH20 
E) None of these 
Sol:​ A) AV28 LG20 
 
3. What is the code for ‘National Park’? 
A) AO32 AP12 
B) AP32 AP16 
C) AN32 AP12 
D) AO32 AP20 
E) None of these 
Sol: ​ A) AO32 AP12 
 
4. “DT20 OM24’ will be the code for 
A) Ports Plant 
B) Edits Poles  
C) Admit Power 
D) Edits Notion 
E) None of these 

Plutus Academy-(24)
Sol: ​D) Edits Notion 
 
5. Which is the code for ‘Normal Cupboard’?  
A) ON24 UV36 
B) OO24 UW32 
C) OP24 UU36 
D) OO24 UW36 
E) None of these 
Sol: ​OO24 UW32 
 
 
TYPE-5(Capital Letter to Small Letter)   
 
Ex1. Using the following code and Key,Decode the given coded word: 
Code  L X P Z J Y Q M N B 
Key b a e s p r h i g t 
 
What is the Code for the Word- ZBYXMNQB  
1) strength 2) height 3) struggle 4) straight 
Sol: ​4) straight 
 
TYPE-6(Matrix Coding​) 
Q 1. Study the matrix below and answer the following:
Matrix2 
  5  6  7  8  9 

5  S  E  H  B  T 

6  H  S  E  T  B 

7  B  T  S  E  H 

8  E  H  T  B  S 

9  T  S  E  H  B 
Matrix 1 
  0  1  2  3  4 

0  F  A  N  O  I 

1  I  O  F  A  N 

2  A  N  O  I  F 

3  O  F  I  N  A 

4  N  I  A  F  O,. 

Plutus Academy-(25)
 
Ex 1​. NEST 
a) 33,85,88,86 b) 21,76,77,76 c) 14,67,66,67 d) 02,56,55,59 
Sol:​ d) 02,56,55,59 
 
Ex 2 ​.FAITH 
a) 43,42,41,78,89 b) 31,34,23,76,79 c) 24,31,10,59,57 d) 12,20,40,68,65 
Sol:​b) 31,34,23,76,79 
Ex 3​. FINE 
a) 31,32,33,82 b) 24,19,21,78 c) 12,10,13,67 d) 00,04,02,56 
Sol: ​d) 00,04,02,56 
 
Ex 4.​ HEAT 
a) 79,53,20,87 b) 65,56,13,57 c) 57,56,01,59 d) 29,85,34,93 
Sol: ​c) 57,56,01,59 
 
Ex 5​. BOTH 
a) 88,30,85,86 b) 75,22,76,79  c) 69,67,68,59  d) 58,02,68,65 
Sol:​b) 75,22,76,79 
 
 
 
 
 
 
 
 
 
 
 
 
 
 
 
 
 

Plutus Academy-(26)
EXERCISE 

Q1. TAP is SZO, then FREEZE is   

 
1) EQDFYG 2) EQDDYD  3) ESDFYF 4) GQFDYF  
 
Q2. If the word PRINCIPAL is written as LAPICNIRP, how ADOLESCENCE can be written in that 
code?  
1) ECNCESELODA  
2) ECNECSLEODA 
3) ECNSCEELODA 
4) ECNECSELODA 
 
Q3. In a certain Language PROSE is coded as PPOQE. How will LIGHT be coded? 
1) LIGFT 2) LGGHT 3) LLGFE 4) LGGFT 
 
Q4. If JOSEPH is coded as FKOALD,then how GEORGE will be coded in that code language? 
1) CADMNO 2) CAKNIT 3) CAKNCA 4) JAKINS  
 
Q5. In a certain code language DHIE is written as WSRV and AEFB is written as ZVUY.How will 
GKLH be written in that code language?  
1) TOPQ 2)TPOS 3)TPOG 4)TOPH 
 
Q (6-10)Study the following information carefully and answer the questions given below: 
In a certain code Language, 
 
‘Rural  and  urban divide’ is coded as 
Na ku za la 
‘Gap in  rural  infrastructure’ is coded as  
Kt  la  vm  pi 
‘Urban planning more important’ is coded as  
ti na  cu bu 
‘More divide than gap’ is coded as 
Pi cu dm za 
 
 
Q6.What is the code for ‘and’? 
 
1) zu 2) na 3) ku 4) la 5) cannot be determined 
 
Q7.What is the code for ‘gap’? 
1) pi 2) na 3) ku 4) la 5) cannot be determined 
 
Q8.What is the code for ‘rural distance’?  
1) zu la 2) la dm 3) pi zu  4) ku 5) cannot be determined 
 

Plutus Academy-(27)
Q9.What is the code for ‘than’? 
 
1) pi 2) dm 3) cu 4) zu 5) ti 
 
 
Q10. Which of the following may possibly be the code for ”infrastructure gap divide rural and 
urban planning’? 
1)bu ku na zu pi la cu 
2)vm la zu pi na cu ku 
3)kt bu zu pi ti vm la 
4)la ku vm kt ti bu na 
5)cannot be determined 
 
Q11. In a certain code  
‘253’ means ‘books are old’; 
’546’ means ‘man is old’ and  
‘378’ means ‘buy good books.’ 
what stands for “are” in that code?  
1) 6 2) 2 3) 4 4) 5 
 
Q12. If C=3, and CAT=24,what is FAULT?  
1) 60 2) 57 3) 64 4) 72 
 
Q13. In a certain code,LONDON is coded as 24-30-28-8-30-28.How will FRANCE be coded? 
1) 10-24-6-28-6-12 
2) 12-26-6-28-8-10 
3) 12-36-2-28-6-10 
4) 12-36-2-28-8-10 
 
Q14. If REASON is coded as 5 and BELIEVED as 7,what is the code number for GOVERNMENT? 
 
1) 6 2) 8 3) 9 4) 10 
 
 
Q15. Following words are written in a code language.Study them carefully and find out the word 
in the given code. 
CAR- $ * ^ 
SIT- # ! @ 
WELL- % & ( ( 
MAP- ~ * ) 
 
Given Code- ( * ~ * 
 
1)LAMP 2)LAMA 3)LAME 4)LAMB  
 
Q 16. If SENSATIONAL can be written as 12314567348 in a certain code, then how will STATION 
be written in the same code? 
1) 1554673  2) 1545673  3) 1455673 4) 1545763 

Plutus Academy-(28)
 
Q 17. In a certain code language TEACH is written as SXZVG. Then how will CHILD be written in 
that code language? 
1) BPRSZ  2) WORSX  3) NMRTU 4) APRRI 5) XSWRK 
 
Q 18. In a certain code language REASONING is written as HZVILTMRM. Then how will 
DIGINOTES be written in that code language? 
1) RTRMMHVVG  2) RSTMMHVVG  3) RTRWMHVGL  
4) RTRMMHVVG 5) MTRMMHVXG 
 
Q 19. In a certain code language TITAN is written as 48797. Then how will WATCH be written in 
that code language? 
1) 16784 2) 17894 3) 68174 4) 79784 5) 17846 
 
Q 20. In a certain code language ZEBRA is written as WBYAJ. Then how will HORSE be written in 
that code language? 
1) MIWIT 2) MIWTI 3) MTIMW 4) MTIWI 5) None of these 
 
Q 21. In a certain code language GGOLD is written as IKUTN. Then how will STARS be written in 
that code language? 
1) GZCUI 2) UXGZC 3) XGCCD 4)YXDEW 5) None of these 
 
Q 22. In a certain code language RTS is written as 19 and MTS as 25. Then how will IPL be 
written in that code language? 
1) 21 2) 35 3) 13 4) 32 5) None of these 
 
Q 23. In a certain code language UNION is written as WLKQL. Then how will PERSON be written 
in that code language? 
1) NGPPOL 2) NGPQQL 3) NPGQQL 4) NGGPOL 5) None of these 
 
Q 24. In a certain code language DIGITAL is written as GKHISYI. Then how will PRINTER be 
written in that code language? 
1) STJNSCO 2) TJUSCOS 3) JNSOLTE 4) NTJNSOC 5) None of these 
 
Q 25. In a certain code language BLOCK is written as NDLIA. Then how will UNION be written in 
that code language? 
1) MLRWP 2) MSRWP 3) MLZWP 4) PWRLM 5) None of these 
 
Q 26. In a certain code language TIMES is written as 79548. Then how will INDIA be written in 
that code language? 
1) 98495 2) 94598 3) 47958 4) 45897 5) None of these 
 
Q 27. In a certain code language SUPPORTERS is written as 8780. Then how will INEVITABLE be 
written in that code language? 
1) 5970 2) 7985 3) 7925 4) 5940 5) None of these 
 
Q 28. If GIVE is coded as 5137 and BAT is coded as 924, then how is GATE coded? 
1) 5427 2) 2547 3) 5247 4) 5724 

Plutus Academy-(29)
Q29. In a certain code, TOGETHER is written as RQEGRJCT. In the same code, PAROLE will be 
written as- 
1) RYPQJG 2) RCPQJG 3) NCPQJG 4) NCPQJC 
 
Q 30. ​If green means red, red means yellow, yellow means blue, blue means orange and orange 
means green, what is the colour of clean sky ? 
 
1)​ Blue ​2)​ Red 3) Yellow ​4)​ Orange 
 
Q31. A word is represented by only one set of numbers as given in any one of the alternatives. 
The sets of numbers given in the alternatives are represented by two classes of letters as in two 
matrices given below. The columns and rows of Matrix-I are numbered from 0 to 4 and that of 
Matrix-II are numbered from 5 to 9.You have to identify the set for the word ​‘POLE’​. 
 
  0  1  2  3  4 

0  G  V  E  A  C 

1  R  O  N  G  L 

2  M  N  E  L  I 

3  O  T  I  T  A 

4  N  L  N  E  P 

 
  5  6  7  8  9 

5  R  E  O  N  G 

6  N  P  V  E  L 

7  M  T  I  O  N 

8  E  A  I  O  C 

9  N  T  A  R  L 
 
1) 66, 11, 67, 02  2) 44, 11, 69, 85  3) 66, 89, 32, 56  4) 44, 13, 69, 65 
 
Q 32. A word is represented by only one set of numbers as given in any one of the alternatives. 
The sets of numbers given in the alternatives are represented by two classes of letters as in two 
matrices given below. The columns and rows of Matrix-I are numbered from 0 to 4 and that of 
Matrix-II are numbered from 5 to 9.You have to identify the set for the word​ 'MOST'. 
 
 

Plutus Academy-(30)
  0  1  2  3  4 

0  O  S  W  H  N 

1  W  N  S  O  H 

2  H  O  N  W  S 

3  S  W  H  N  O 

4  N  H  O  S  W 

 
  5  6  7  8  9 

5  T  A  P  M  D 

6  M  D  A  T  P 

7  D  T  A  P  M 

8  A  P  M  D  T 

9  P  M  T  D  A 
 
 
1) 79, 13, 30, 68  2) 56, 12, 41, 86  3) 97, 43, 21, 79 4) 65, 21, 32, 76 
 
Q 33.A word is represented by only one set of numbers as given in any one of the alternatives. 
The sets of numbers given in the alternatives are represented by two classes of letters as in two 
matrices given below. The columns and rows of Matrix-I are numbered from 0 to 4 and that of 
Matrix-II are numbered from 5 to 9.You have to identify the set for the word ​“MATH”. 
  0  1  2  3  4 

0  E  S  T  M  N 

1  T  N  S  E  M 

2  M  E  N  T  S 

3  S  T  M  N  E 

4  N  M  E  S  T 

 
 
 

Plutus Academy-(31)
  5  6  7  8  9 

5  W  A  P  H  D 

6  H  D  A  W  P 

7  D  W  A  P  H 

8  A  P  H  D  W 

9  P  H  W  D  A 
  
1) 32, 77, 23, 79  2) 23, 76, 32, 97  3) 41, 57, 23, 85  4) 14, 78, 23, 58 
 
Q 34. A word is represented by only one set of numbers as given in any one of the alternatives. 
The sets of numbers given in the alternatives are represented by two classes of letters as in two 
matrices given below. The columns and rows of Matrix-I are numbered from 0 to 4 and that of 
Matrix-II are numbered from 5 to 9.You have to identify the set that is not for the word ​“SAND”. 
  0  1  2  3  4 

0  E  S  R  U  N 

1  R  N  S  E  U 

2  U  E  N  R  S 

3  S  R  U  N  E 

4  N  U  E  S  R 

 
  5  6  7  8  9 

5  W  A  P  T  D 

6  T  D  A  W  P 

7  D  W  A  P  T 

8  A  P  T  D  W 

9  P  T  W  D  A 
 
1) 12, 67, 40, 66  2) 43, 85, 22, 87 3) 24, 67, 22, 66  4) 30, 77, 33, 59 
 
Q 35.​A word is represented by only one set of numbers as given in any one of the alternatives. 
The sets of numbers given in the alternatives are represented by two classes of letters as in two 

Plutus Academy-(32)
matrices given below. The columns and rows of Matrix-I are numbered from 0 to 4 and that of 
Matrix-II are numbered from 5 to 9.You have to identify the set for the word ​“PENT”. 
 
  0  1  2  3  4 

0  E  S  R  U  N 

1  R  N  S  E  U 

2  U  E  N  R  S 

3  S  R  U  N  E 

4  N  U  E  S  R 

 
  5  6  7  8  9 

5  W  O  P  T  I 

6  T  I  O  W  P 

7  I  W  O  P  T 

8  O  P  T  I  W 

9  O  P  T  I  W 
 
1) 86, 21, 33, 65 2) 95, 34, 40, 79  3) 57, 02, 34, 87  4) 78, 42, 11, 56 
 
Q36. ​ Study the following information to answer the given questions. 
 

Letter  S  P  W  J  K  I  L  T  Q  O  D  R  Y  H  E  A  F  G  U 

Code  X  +  1  =  #  4  ?  $  @  5  6  3  8  9  2  0  7  *  & 
Condition:  

i)If  the  first  letter  and  last  letter’s  place  value  are  even  according  to  the  English  Alphabetical 
order, then both are to be coded by the code of U. 
ii)If  the  first  letter  and  last  letter’s  place  value  are  odd  according  to  the  English  Alphabetical 
order, then both are to be coded by the code of S. 
iii)If  the  first  letter  is  a  vowel  and  last  letter is a consonant, and there are more than two vowels 
in the group, then both are to be coded by each other’s code. 
 

Plutus Academy-(33)
Q1. EDAIFH 
 
a)260479 b) 960472 c) 960479 d) 604792 e) None of these 
 
Q2. QERTYW 

a)X23$8X b) @23$81 c) 123$8@ d) &23$8& e) None of these 

Q3. RWYHPJ 

a)X189+X b) @189+@ c) &189+& d) 1189+X e) None of these 


 
Q4. TQODSP 

a)&@56X& b) X@56XX c) *@56X* d) *@56X& e) None of these 


 
Q5. SWJKIL 

a)X1=#4? b) X1=#4X c) &1=#4& d) ?1=#4X e) None of these 


 
Q37. ​ Study the following information to answer the given questions. 
 

Letters  A  B  C  D  E  F  G  H  I  J  K  L  M 

Code  1  2  3  4  5  6  8  7  9  11  10  13  12 


Condition:  

i)If  the  first  and  last  letter’s  place  value  according  to  the  English  Alphabetical  order  has 
difference of 1,, then both are to be coded by the code of G. 
ii)If  4th  letter’s  place  value  is  multiple  of  3, then the 2nd letter is to be coded by the code for 6th 
letter. 
iii)If  the  first  letter  is  a  vowel  and  last  letter  is  a  consonant,  then  the  code  of  both  are  to  be 
interchanged. 
 
Q 1. CBJIKM 
a) 3 2 10 9 11 12 b) 3 2 11 10 9 12 c) 3 12 11 9 10 12 
d) 3 12 10 9 11 12 e) None of these 
 
Q 2. DMIEAC 
a) 8 12 9 5 1 8 b) 4 12 9 5 1 3 c) 4 12 9 5 1 4 
d) 3 12 9 5 1 3 e) None of these 
 
Q 3. HGLBCA 

Plutus Academy-(34)
a) 7 8 13 2 3 7 b) 7 8 13 2 3 1 c) 1 8 13 2 3 7  
d) 8 8 13 2 3 8 e) None of these 
 
Q 4. EIJKDC 
a) 5 9 11 10 4 5 b) 3 9 11 10 4 3 c) 3 9 11 10 4 5 
d) 5 9 11 10 4 3 e) None of these 
 
Q 5. EFGMJA  
a) 5 6 8 12 11 1 b) 1 6 8 12 11 1 c) 5 6 8 12 11 5 
d) 8 6 8 12 11 8 e) None of these 
 
Q38. ​(1 –5) Study the information below and answer the following question: – 
In a certain code language, 
‘Thin paper neatly folded’ is written as @D6, %R5, !N4, ?Y6 
‘Four people from USA’ is written as @M4, %E6, #A3, @R4 
‘Urban development programme launched’ is written as % E9, *T11, #N5 &D8 
‘Dhaya likes forties hero’ is written as @E7, &E5, *A5, $O4 

1. The code for the word ‘People’ is 


  1.@M4 2.%E6  3.#A3  4.@R4 5.None of these 
  
2. ​The code ‘ *A5 ‘ denotes which of the following word ? 
  1.Likes 2.Hero 3.Forties 4.Dhaya 5.None of these 
  
3. The code word of ‘Four’ is 
  1. @R4 2.%E6 3.@M4 4.#A3 5.None of these 
  
4. ‘#’ denotes which letter of the given words ? 
1.N 2.F 3.L 4.D 5.U 
 
5. According to the given code word, what will be the code for ‘ Data Line reach points’ ? 
  1.*4A &4E @5H %6S 
  2.*4A &4E !5H %6S 
  3.*4A &4E #5H %6S 
  4.*4A &4E $5H %6S 
  5.None of these 
 
Q 39 (1-5) Study the information below and answer the following question: 

In a certain code language,  

‘Arrive today eagles later’ is written as 21*R,6$A, 14$O, 25*A 


‘Begin work faster table’ is written as 14$A, 17%O, 26*A, 22$E 
‘Length error arrow burn’ is written as 6*E, 25$R, 22%U, 21$R 
‘Trial better than wisdom’ is written as 14$R, 14%H, 22*E, 17*I 
 
1. The code for the word ‘ table’ 

Plutus Academy-(35)
1) ​26*A 2) 17%O 3) 14$A 4) 22$ E 5) None of these 

2. ​The code word 6$A for the word 

1.Later 2.Arrive 3.Earlier 4.Today 5.Either 1 or 3 

3. ​Find the code word for the word ‘Burn’ ? 

1) 25$R 2) 22%U 3) 21$R 4) 6*E 5) Can’t be determined 

4. ​According to the given code, what is the code for ‘M’ ? 

1) 12 2) 8 3) 10 4) 7 5) Can’t be determined 

5. ​By using the given code word, find the code word for ‘ Better Luck Next Arrive’ ? 
1) 22$E, 21$R, 6*U, 8%E 
2) 8*E, 21*R, 22%E, 6%U 
3) 22*E, 6%U, 8%E, 21*R 
4) 21%R, 22$E, 6*U, 8%E 
5) 6$U, 22*E, 8*E, 21%R 
 
Q40(1-5) Study the following information to answer the given questions: 
In a certain code,  
Give solution for problem - #E4, $R3, *M7, @N8 
Pure first sale great - @E4, $T5, #T5, *E4 
Pet sensed ghostly form - *T3, $M4, @D6, #Y7 
Spa guards picked flake - #S6, *D6, $E5, @A3 
 
1. What is the code for ‘solution’? 
(a) #E4 (b) $R3 (c) *M7 (d) @N8 (e) Cannot be determined 
 
2. Which of the following code can be coded for ‘FRIENDS’? 
(a) #S7 (b) $R7 (c) $S7 (d) @S6 (e) Cannot be determined 
 
3. What is the code for ‘search from page’? 
(a) *E4 @H6 $M4 (b) $R3 *E4 @D5 (c) *M7 @H5 #E4 
(d) @N8 $M4 @H5 (e) Cannot be determined 
 
4. What is the code for ‘Gross’? 
(a) #S5 (b) $S3 (c) *S7 (d) #N5 (e) Cannot be determined 
 
5. What is the code for ‘Payment’? 
(a) #T4 (b) $T7 (c) *T7 (d) @N7 (e) Cannot be determined 

Q41: (1-5) Study the following information to answer the given questions: 

Plutus Academy-(36)
In a certain code,  
‘Most safety high level’ is written as ‘8*Y, 7?L, 6#H, 6%T 
‘Made in India project’ is written as ‘9@T, 7!A, 6%E, 4!N’ 
‘Set list new home’ is written as ‘5*T, 6#E, 6?T, 5&W’ 
‘Large sale post interval’ is written as ‘6@T, 10!L, 7?E, 6*E’ 
(All the codes are three-letter codes only.) 
 
1) According to the given code word, what will be the code for ‘Leave his much peace’ ? 
a) 5@E, 3#S, 5?E, 3%H 
b) 7@E, 5#S, 7?E, 6%H 
c) 9@E, 7#S, 7?E, 8%H 
d) 9#E, 5#S, 5@E, 5%H 
e) None of these 
 
2) ‘?’ denotes which letter of the given words ? 
a) P b) M c) L d) S e) H 
 
3) The code word of ‘Intend’ according to the given code is 
a) 8!N b) 6!D c) 8!D d) 6!N e) None of these 
 
4) The code ‘6*E’ denotes which of the following word ? 
a) Large 
b) Set 
c) Sale 
d) Home 
e) None of these 
 
5) The code for the word ‘Person’ is 
a) 8*E 
b) 6*N 
c) 8@N 
d) 6@E 
e) None of these 
 
Q42: (1-5) Study the following information to answer the given questions: 
In a certain coded language, 
‘Built above abuse catch’ is coded as ‘d20z?’ ‘t21a%’ ‘w15a%’ ‘m9t^’, 
‘Chair dance drill equal’ is coded as ‘b21p!’ ‘m9q!’ ‘d14z%’ ‘j1g#’, 
‘Frank house image lives’ is coded as ‘f22h&’ ‘h1l%’ ‘t21n%’ ‘o1q*’. 
‘Minus needs peace refer’ is coded as ‘f6d#’ ‘d1d%’ ‘e5d&’ ‘v14h&’ 
 
1) What is the code for the word “order” using the above coded method? 
a) g8v# b) e5d# c) f4q# d) a2l# e) n1n# 
 
2) What does the code “s1g%” represents? 
a) Make b) Share c) Rural d) Shape e) Green 
 
3) How “third teams” can be coded using the above method? 

Plutus Academy-(37)
a) o9i$ y15z& b) m4b© d7g& c) k6e● u65& 
d) s9g$ n1d& e) q9g$ n1d& . 
 
4) What does the code “f22h&” represents? 
a) Lives b) Image c) House d) Franks e) Cannot be determined 
 
5) What does the code “o5qΔ b13n£” represents? 
a) Until have 
b) Trend woman 
c) where comes 
d) Loves others 
e) be happy 
 
Q43: 1-5) Study the following information to answer the given questions: 
‘Students needs more placements’ is coded as ‘t5m t10o f4l t8r’ 
‘Pakistan win the match’ is coded as ‘o3v i5l f3s o8o’ 
‘Practice makes perfect man’ is coded as ‘u7o f8o o3l t5l’ 
 
1. What is the code for “toys makers” in the above code language? 
a) r4t t6l b) s4r l6t c) t4s t6l d) s4t t5l e) None of these 
 
2. what is the code for “property”? 
a) x7o b) z8o c) w8o d) z8m e) None of these 
 
3. which of the following is the exact code for “place for give respect”? 
a) f5o r3s f4f u3r 
b) g3h h5m f4f v7q 
c) u7q f4f s3e f5o 
d) u7r f4e s3e f5o 
e) cannot be determined 
 
4. Which of the following is code for “pleasure”? 
a) f8o b) o8m c) r7q d) f8q e) d80 
 
5. In given code language, what does the code “u5q e6o f4f” stands for? 
a) right more played 
b) right game played 
c) maths easy subject 
d) very sense thing 
e) played game thing 
 
Q43: (1-5) Study the following information to answer the given questions: 
In a certain code language, 
 
“New economic development begin” is written as “38%v 30~p 17#x 46&s” 
“Great influence to development” is written as “19@y 30~p 27%v 40*r” 
“New opportunity to start” is written as “19@y 15%v 14~p 17#x” 
“Have less economic opportunity” is written as “46&s 14~p 41$w 23$w” 

Plutus Academy-(38)
1. Which of the following is the code for the word “economic”? 
(a) 38%v (b) 38%v (c) 17#x (d) 46&s (e) None of these 
 
2. The code “27%v 15%v” denotes which of the following word? 
(a) Development Start (b) Great influence (c) Great start   
(d) Either a or b (e) None of these 
 
3. The code word of “capacity” according to the given code is? 
(a) 26?t (b) 26&t (c) 25&s (d) 26*s (e) 26&s 
 
4. “38%v 46?u” denotes which of the following word? 
(a) New Economic (b) Great influence (c) Development start 
(d) Course begin (e) None of these 
 
5. According to the given code word, what will be the code for “given opportunity for 
development”? 
(a) 30~p 33%v 14~p 30&x (b) 30~p 33$v 14~p 30#x 
(c) 30~p 33%v 14*p 30#x (d) 30?p 33%v 14~p 30#x 
(e) 30~p 33%v 14~p 30#x 
 
Q44: 1-5) Study the following information to answer the given questions: 
 
In a certain code language, 
‘Picture worth thousand words’ is coded as ‘xi6 ue9 xt6 qf8’ 
‘watched pot never boil’ is coded as ‘qu4 os6 xe8 ct6’ 
‘Beggars can’t be choosers’ is coded as ‘ct8 dt9 du5 cf3’ 
 
1. What is the code for “island”? 
a) jd6 b) jf6 c) he7 d) je7 e) None of these 
 
2. What may be the code for “vigorous”? 
a) ut8 b) ut9 c) wt9 d) wt8 e) None of these 
 
3. “Grass is always greener” will be coded as? 
a) hs8 jt3 ht6 bt7 
b) ht5 ht7 js3 bs8 
c) bt7 jt4 hs8 ht5 
d) bs7 ht6 hs8 jt3 
e) None of these 
 
4. How “beggars” is coded in the language? 
a) ct8 b) dt9 c) cf3 d) du5 e) None of these 
 
5. How “choosers choice” will be coded in the language? 
a) df6 dt8 b) dt9 df7 c) dt9 df8 
d) df9 dt7 e) None of these 
 
 

Plutus Academy-(39)
Q45: Directions (1-5): Study the following information to answer the given questions.(IBPS PO 
2017)  
In a certain code, 
‘​detail event year this​’ is written as ‘​bi gv oc st​’  
‘​event revised of awaited​’ is written as ‘​tm oc da pu​’ 
‘​of detail results first​’ is written as ‘​nh mk tm gv​’  
‘​awaited great year of​’ is written as ‘​da strx tm​’  
 
1. In the given code language, what does the code ‘mk’ stand for?  
(a) Either ‘detail’ or ‘year’ (b) this (c) of (d) Either ‘first’ or ‘results’ (e) great  
 
2. What is the code for ‘awaited’ in the given code language? 
(a) tm (b) rx (c) st (d) da (e) None of these  
 
3. What is the code for ‘event’ in the given code language?  
(a) nh (b) oc (c) gv 6 (d) pu (e) rx  
 
4. What may be the possible code for ‘awaited revised’ in the given code language?  
(a) ve be (b) da nh (c) nh ve (d) pu da (e) bi da  
 
5. If ‘of year home’ is written as ‘aj tm st’ in the given code language, then what is the code for 
‘home revised this’?  
(a) aj gv nh (b) pu aj bi (c) nh bi pu (d) bi da aj (e) aj oc pu  
 
Coding Decoding Answer Sheet 
 
Q1.(2) EQDDYD 
Q2.(4)ECNECSELODA (Reverse of the word) 
Q3.(4)LGGFT 
Q4.(3)CAKNCA 
Q5- TPOS (Reverse of each Letter) 
Q6-ku 
Q7.pi 
Q8.zu la 
Q9.dm 
Q10.cannot be determined 
Q11.4) 5 
Q12.(1)60 
Q13.(3) 12-36-2-28-6-10 (Two times the place value of each letter) 
Q14. 3) 9( NUMBER OF LETTERS - 1) 
Q15.(2) LAMA 
Q16. 2) 1545673 
Q17. WORSX 
Q18. 3) RTRWMHVGL 

Plutus Academy-(40)
Q19. 1)16784 
Q20. 4) MTIWI 
Q21. 2) UXGZC 
Q22. 3) 13 
Q23. 3) NPGQQ 
Q24. 1) STJNSCO 
Q25. 4) PWRLM  
Q26. 2) 94598 
Q27. 4) 5940 
Q28. 3) 5247 
Q29. 3) NCPQJG 
Q30. ​4)​ Orange 
Q31. ​2) 44, 11, 69, 85 
Q32. 1) 79, 13, 30, 68 
Q33. 1) 32, 77, 23, 79  
Q34. 2) 43, 85, 22, 87 
Q35. 1) 86, 21, 33, 65 
Q36. 1) b) 960472 
2) d) &23$8& 
3) c) &189+& 
4) a)&@56X& 
5) a)X1=#4? 
Q37. 1) c) 3 12 11 9 10 12 
2) a) 8 12 9 5 1 8   
3) b) 7 8 13 2 3 1  
4) c) 3 9 11 10 4 5 
5) a) 5 6 8 12 11 1 
Q38. 1. ​The code for the word ‘People’ is %E6 
Explanation : 
People – ​%E6 
P – % 
No of letters – 6 
Last letter – E 
T =!, P = %, N = ?, F = @, U =#, D = *, L = &, H = $ 
2. ​4.Dhaya 
3. 1. @R4 
4. 5.U 
5 None of these 
​ ​Data Line reach points is​ ​ ​*4A &4E ?5H %6S 
Q39. 1. 3) 14$A 
2. 1) Later   
3. 2) 22%U 
4. 4) 7 

Plutus Academy-(41)
5. 1) 22$E, 21$R, 6*U, 8%E 
 
Q40 1. ​ (d) @N8  
Every symbol represents the first letter of each word in the following manner 
# - G 
@ - S 
$ - F 
* - P 
Every Letter in the code represents the last letter of the respective word. 
Every number represent the total number of letters in the word. 
2. (c) $S7 
3. (a) *E4 @H6 $M4 
4. (a) #S5 
5. (c) *T7 
 
Q41 1. b) 7@E, 5#S, 7?E, 6%H 
2. ​c) L  
3. ​c) 8!D 
4. ​c) Sale 
5. ​c) 8@N 
(Hint: I – ! 
D – last letter of the word D and 
Total no of letter 6 + 2 = 8 
I =!,M = %,L = ?,P = @,H =#,S = *,N = &) 
Q42 1. c) f4q# 
2. b) Share 
3. d) s9g$ n1d&   
4. a) Lives 
5. b) Trend woman 
(Hint: (c) 
Every symbol represent last letter in the word. 
e – %,  r – #,  s – &,  l – !s  t–^ k–* h -? 
Above – w15d% 
Number 15 – Middle letter position in alphabet 

Plutus Academy-(42)
 
Q43. 1. c) t4s t6l 
2. b) z8o  
3. c) u7q f4f s3e f5o 
4. a) f8o 
5. b) right game played 
(Hint: Pattern: Number Number of letter count 
1st letter –  1st letter (last letter +1) 
Last letter –  Last letter (first letter -1) 

 
Q44. 1. (d) 46&s 
2. (c) Great start 
3. (e) 26&s 
4. (d) Course begin 
5. (d) 30?p 33%v 14~p 30#x 
 
Hint: Example 
● N E W —————————————— 17 # x 
  
● The First Letter of the word is N 
Take the Reverse Letter of N which is M 
M’s Position is 13 
Take the Last word which is W 
Take the reverse letter of W which is D 
D’s Position is 4 
Add the Position of M + D = 13+4 = 17 
● Count the Number of Letters in the word. Each number corresponds to a certain 
symbol 
No of Letters  Corresponding Symbol 

2  @ 

3  # 

4  $ 

Plutus Academy-(43)
5  % 

8  & 

9  * 

10  ~ 
 
● Total Number of letters in the word is 3 , So the reverse letter of the number 3 is X. 
 
Q44. 1. d) je7 
2. c) wt9 
3. a) hs8 jt3 ht6 bt7 
4. a) ct8 
5. b) dt9 df7 
Hints: Ex: ‘Picture’ coded as ‘qf8’ 
‘q’ – first letter + 1 (i.e. p+1=q) 
‘f’ – last letter + 1 (i.e. e+1=f) 
‘8’ Number of letters in the word Picture +1 
 
Q45. Word-Code  
Detail-gv  
Event - oc  
Year - st  
Of - tm  
This - bi  
Awaited - da  
Revised - pu  
Great - rx  
Results/ first---mk/nh  
 
1. (d) Either ‘first’ or ‘results’ 
2. (d) da 
3. (b) oc 
4. (d) pu da  
5. (b) pu aj bi 
 
 
 
 
 
 
For video solutions visit:​ https://www.youtube.com/watch?v=cQNK4wTSJ9g&t=32s 
​https://www.youtube.com/watch?v=jt4XjN1ZU80&t=4s 
 

Plutus Academy-(44)
Inequality 
 
Inequality is basically comparison of two or more than two variables. 
 
Example  of  Inequality  Questions-​In  these  type  of  questions  relationship  between  different 
elements  is  shown  in  the  Statements.  These  statements  are  followed  by  number  of 
conclusions.We  have  to  study  all  the  statement  carefully  after  that  we  have  to  identify  which 
conclusion is/are correct according to the relationships given in the statements. 
Question Pattern- 
Statements- _ _ _ _ _ _ _ 
Conclusions- _ _ __ _ _ _  
 
Ex- ​Statement -  A>B=C 
Conclusion - i)  A>B 
ii) A>C 
 
Options: 1) Only i follows. 
2) Only ii follows. 
3) Either i or ii follows. 
4) Both follows. 
5) None follows.  
 
Symbols of Inequalities: 
There is 3 basic fundamental symbol. 
> (greater than) 
< (smaller than) 
= (Equal to) 
 
> and < are known as Single or definite Inequality symbol. 
The presence of = does not have any significant effect on the conclusion. 
 
Along with these, there are 3 more symbols that are of significance. 
1) ≥ (Either greater than or equal to) 
2) ≤ (Either smaller than or equal to) 
3) =/ (Not Equal to) 
 
≤ and ≥ are known as double or possible inequality symbols.  
The concept of =/ is totally different. So it doesn’t come under our purview. 
 

Plutus Academy-(45)
Symbols and their definitions: 
Symbols  Definition 
>  Greater than 
Ex- A > B means A is greater than B 

<  Less than 


Ex- A < B means A is less than B 

=  Equal to 
Ex- A = B means A is equals to B  

>= or ≥   Greater than equal to 


Ex- A ≥ B means A is greater than and equal to B 

<= or   Less than equal to 


Ex- A ≤ B means A is smaller than and equal to B 

≠  Not equal to 


Ex- A ≠ B means A is not equal to B  

Neither greater than nor equal to  


 
Ex- A B means A is neither greater than nor equal to B 

  Neither smaller than nor equal to 


Ex- A B means A is neither smaller than nor equal to B 
 
 
Steps to follow while solving: 
1) Check if the symbols are in same order(direction) 
Conclusions can only be determine if all the symbols used within the statements are in the 
same direction- 
 
For Ex - A>B>C or A>B=C or A>B>=C or A>=B>C or A<B<=C or A<B<C 
 
2) Check if single inequality symbol is present. If single inequality symbol is present, conclusion 
will be that single inequality symbol. 
3) If single inequality symbol is not present and double inequality symbol is present, then 
conclusion will always be double. 
 
Types of Inequalities: 
Type I) Simple Inequality(Statement and Conclusion) 

Plutus Academy-(46)
Type II) Coded Inequality (Coded statement and conclusion) 
Type III) Conditional Inequality( Statement condition based)  
 
Tricks:- 
1. Normal Situation- 
Sl. No.  Type(if statements contains below  Conclusion  Example 
symbols in any order)  (Statements and true 
conclusions) 

1  >,>,>  >  Statement- 


A>B>C 
Conclusions- 
1)A>C 
2)B>C 

2  > , >= , =   >  Statement- 


A>B>=C=D 
Conclusions- 
1)A>C 
2)A>D 
 

3  > , >=  >  Statement- 


A>B>=C 
Conclusions- 
1)A>C 

4  > , =   >  Statement- 


A>B=C 
Conclusions- 
1)A>C 

5  >= , =  >=  Statement- 


A>=B=C 
Conclusions- 
1)A>=C 

6  <,<,<  <  Statement- 


A<B<C 
Conclusions- 
1)A<C 
2)B<C 

7  < , <= , =   <  Statement- 


A<B<=C=D 
Conclusions- 

Plutus Academy-(47)
1)A<C 
2)A<D 
 

8  < , <=  <  Statement- 


A<B<=C 
Conclusions- 
1)A<C 

9  < , =   <  Statement- 


A<B=C 
Conclusions- 
1)A<C 

10  <= , =  <=  Statement- 


A,=B=C 
Conclusions- 
1)A<=C 
 
2. Either Situation- 
 
Either condition occurs when 2 conclusions follow for the same elements in any 
statement. 
Ex1- 
 
Statement - A>=B 
Conclusion- 
A>B 
A=B 
 
Here either I or II will follow. 
 
Ex-2- 
 
Statement - A < B >= C 
Conclusions- 
I A < C 
II A >= C 
 
Here either I or II will follow. 
 
 

Plutus Academy-(48)
 
 
3. False Situations- 
Statement: 
P > Q 
P > R 
Conclusion:  Q > R 
R > Q 
Both conclusions are false. 
 
Statement: 
A > B < C 
Conclusion: A>C 
A<C 
Both conclusions are false as both symbols are not in same direction. 
 
 
Type-1 
In these questions relationships between different elements is shown in the statements. These 
statements are followed by four conclusions numbered I,II,III and IV. Study the statements and 
Conclusions carefully and select the correct answer: (IBPS Bank PO/MT CWE-IV 2014 ) 
Q1.Statement: 
Y>R ≥ S=T<M ≤ N<J 
Conclusions: 
I. T<Y 
II. N>S 
III. J<S 
IV. S>=Y 
 
1) Only Conclusions I,II and III are true. 
2) Only Conclusions I and II are true. 
3) Only Conclusions II and III are true. 
4) Only Conclusions I,II,III and IV are true. 
 
Q2.Statements: 
S>H>=T<=C; 
Y>T>=N; 
Conclusions: 
I. H>Y 
II. S>C 

Plutus Academy-(49)
III. C=Y 
IV. C>=N 
 
1) Only conclusions I and IV are true. 
2) Only conclusion III is true. 
3) Only conclusions II and IV are true. 
4) Only Conclusions IV is true. 
5) Only Conclusions I and II are true. 
 
Q3.Statement: 
K>H<Y<=Z<A>R>=L 
 
Conclusions: 
I. A>Y 
II. L>A 
III. H<A 
IV. K>R 
 
1) Only conclusions I and III are true. 
2) Only conclusions II and IV are true. 
3) None of the conclusion is true. 
4) Only conclusions II and III are true. 
5) Only conclusions II,III and IV are true. 
 
 
Q4. Statements: 
D > M > U >=E; 
N < U >=J 
Conclusions: 
I D>E 
II E<=J 
III M>N 
IV D>J 
1) Only conclusions I,II and III are true. 
2) Only conclusions II,III and IV are true. 
3) Only conclusions I,III and IV are true. 
4) None of the conclusions is true. 
5) All the conclusions I,II,III and IV are true. 
 
Q5. Statement: 
P>L=U>=C<=K>S 
Conclusions: 
1) Only conclusions I and II are true. 

Plutus Academy-(50)
2) Only conclusions III and IV are true. 
3) Only conclusions I and IV are true. 
4) Only conclusions I,II and IV are true. 
5) Only conclusions II,III and IV are true. 
 
Q(6-10)In these questions , relationship between different elements is shown in the statements. 
These statements are followed by the two conclusions. (Indian overseas bank PO 2011)  
Mark answer (1) if only conclusion I follow 
Mark answer (2) if only conclusion II follow 
Mark answer (3) if either conclusion I or II follow 
Mark answer (4) if neither conclusion I or II follow 
Mark answer (5) if both conclusion I and II follow 
 
 
Q6. Statements: 
A > = B = C; 
B < D < = E; 
Conclusions: I. D > A 
II. E > C 
 
 
Q7. Statements: 
L > U > = K; 
Z < U < R 
Conclusions: I L > Z 
II K < R 
 
Q8. Statements: 
Y < J = P > = R > I 
Conclusions: I J > I 
II Y > R 
 
Q9.Statements: 
V > = K > M = N; 
M > S; 
T < K 
Conclusions: I T < N 
II V = S 
 
Q10.Statements: 
F <= X < A; 
R < X <= E; 
Conclusions: I F <= E 

Plutus Academy-(51)
II R < F 
 
 
 
 
Type-2 
 
Q(11-15) In the following questions, the symbols #, %,@,$ and are used with the following 
meanings as illustrated below 
‘P # Q’ means ‘P is not smaller than Q’ 
‘P % Q’ means ‘P is not greater than Q’ 
‘P @ Q’ means ‘P is neither smaller than nor equal to Q’ 
‘P $ Q’ means ‘P is neither greater than nor equal to Q’ 
‘P Q’ means ‘P is neither smaller than nor greater than Q’ 
 
In each of the following questions assuming the given statements to be true, find out which of 
the two conclusions I and II given below them is/are definitely true. 
 
Mark answer (1) if only conclusion I is true 
Mark answer (2) if only conclusion II is true 
Mark answer (3) if either conclusion I or II is true 
Mark answer (4) if neither conclusion I or II is true 
Mark answer (5) if both conclusion I and II is true (Indian Overseas Bank PO 2009) 
 
Q11.Statements : 
H W, W%R,R @F 
Conclusions: I R H 
II R @ H 
 
Q12. Statements: 
M$T, T@K, K D 
Conclusions: I D$ T 
II K$M 
 
Q13. Statements: 
R%N, N#F, F@B 
Conclusions:   I F R 
II B$N 
 
Q14. Statements: 
H@W, W$M, M#K 
Conclusions: I K$W 

Plutus Academy-(52)
II H@M 
 
Q15. Statements: 
R#T, T M, M@D 
Conclusions:  I D $ T 
II R#M 
 
Q16. Statements: 
F#K, K$B, B%M 
Conclusions: I M@F 
II B@F  
 
Q(17-21) In the following question , the symbols @ , $, # , * and % are used with the following 
meaning as illustrated below. 

P $ Q means ‘ P is not smaller than Q ‘ . 

P* Q means ‘ P is neither greater than nor equal to Q’. 

P# Q means ‘P is neither smaller than nor equal to Q’. 

P% Q means ‘P is not greater than Q’. 

P@ Q means ‘P is neither greater than nor smaller than Q’. 

Now in each of the following questions, assuming the given statement to be true , find which of 
four Conclusion a, b, c,d given below them is / are definitely true and give your answer 
accordingly. 

Q17. Statement- R# J, J$ D, D@ K, K% T 

Conclusion-1. T# D  

2. T@ D  

3.R# K   

4. J$ T 

Options:a) Either 1 or 2 is true 

b) Only 3 is true 

c) 3 and 4 are true 

d) Either 1 or 2 and 3 are true 

e) None of these 

Plutus Academy-(53)
Q18. Statement-T% R, R$ M, M@D, D* H 

Conclusion-1. D%R  

2.H# R  

3.T* M  

4.T% D 

Options: a) Only 1 is true 

b) 1 and 4 are true 

c) 1 and 2 are true 

d) 2 and 4 are true 

e) None of these 

Q19. Statement-M@ B, B# N, N$ R, R* K 

Conclusion-1.K# B 2. R* B 3. M$ R 4. N* M 

Options: a) 1 and 3 are true 

b) 1 and 2 are true 

c) 2 and 4are true 

d) 2, 3,4 are true 

e) None of these 

Q20. Statement- F# H, H@ M, M* E, E$ J 

Conclusion-1. J* M  

2. E# H  

3. M* F  

4. F# E 

Options: a) 1 and 2 are true 

b) 2 and 3 are true 

c) 1 , 2 and 3 are true 

d) 2, 3 and 4 are true 

e) None of these 

Plutus Academy-(54)
Q21. Statement-D% A, A@ B, B* K, K% M 

Conclusion- 1. B$ D  

2. K# A  

3. M # B  

4. A* M 

Options: a) 1, 2 and 4 are true 

b) 1, 2 and 3 are true 

c) 2, 3 and 4 are true 

d) 1, 3 and 4 are true 

e) None of these 

Q(22-). @ means ‘ less than’ 

($) means ‘not greater than’ 

(-) means ‘equal to’ 

(+) means’ not equal to’ 

(*) means ‘not less than’ 

(=) means ‘greater than’ 

Q22. X@ Y + Z implies 

Options: a)X- Y = Z 

b) X * Y – Z 

c) X $ Y @ Z 

d) X + Y = Z 

Q23. X@ Y – Z implies 

Options a) X* Y – Z 

b) X –Y * Z 

c) X + Y * Z 

d) X* Y = Z 

Q24. X –Y + Z implies 

Plutus Academy-(55)
Options​: ​a) X+ Y + Z 

b) X@ Y – Z 

c) X- Y @ Z 

d) None of these 

Q25. X- Y = Z implies 

Options: a) X + Y + Z 

b) X $ Y * Z 

c) X + Y @ Z 

d) X+Y * Z 

Q26. X = Y Z implies 

Options: a) Y@ X = Z 

b) Z @Y @ X 

c) Z@ X = Y 

d) None of these 

 
 
 
Type 3: 
Read the following statement carefully and answer the following questions- 
Q27. Which of the following expressions will be true if the expression R>O =A >S < T is definitely 
true? 
1) O > T 
2) S < R 
3) T > A 
4) S = O 
5) T < R 
 
Q28. Which of the following symbols should replace the question mark(?) in the given 
expression in order to make the expressions ‘P>A’ as well as ‘T<L’ definitely true? 
P > L ? A > = N = T 
1) <= 
2) > 

Plutus Academy-(56)
3) < 
4) >= 
5) Either <= or < 
 
Q29. Which of the following symbols should be placed in the blank spaces respectively (in the 
same order from left to right) in order to complete the given expression in such a manner that 
makes the expressions ‘B>N’ as well as ‘D <= L’ definitely true? 
B__L__O__N__D 
1) =,=,>=,>= 
2) >,>=,=,> 
3) >,<,=,<= 
4) >,=,=,>= 
5) >,=,>=,> 
 
Q30. Which of the following should be placed in the blank spaces respectively (in the same 
order from left to right) in order to complete the given expression in such a manner that makes 
the expression ‘A<P’ definitely false? 
 
___<___<___>____ 
1) L ,N, P ,A 
2) L , A,P, N 
3) A, L, P, N 
4) N, A ,P, L 
5) P, N, A, L 
 
Q31. Which of the following symbols should be placed in the blank spaces respectively *(in the 
same order, from left to right) in order to complete the given expression in such a manner that 
makes the expression ‘F > N’ and ‘U > D’ definitely false? 
F__O__U__N__D 
1) < , < , > , = 
2) < , = , = , > 
3) < , =, = , < 
4) >=, = , = ,>= 
5) > , > , = ,< 
 
Q32. Which of the following expressions has E>G and C>F definitely true? 
 
(A) A < E ≥ B > D > F = G < C 
(B) B ≤ A < C < D > E > F > G 
(C) A ≤ C ≥ B ≥ D < G ≥ E > F 
(D) D ≤ G > E ≥ F > C ≤ A ≤ B 
(E) C ≥ A > D ≥ B ≥ E = G > F 
Q33. ​Which of the following should replace the question mark so that C>D is definitely true? 

Plutus Academy-(57)
C = B ? A ≥ D ≤ G > F<E 

Options: (A) = 

(B) > 

(C) ≤ 

(D) ≥ 

(E) < 

Q34.Which of the following order of letters (from left to right) in the blanks makes the 
expression, D<C is definitely true? 

____ < ____≤ ____ = _____> ____ 

Options:(A) B,C,D,E,A 

(B) C,A,D,B,E 

(C) D,B,A,C,E 

(D) E,C,A,B,D 

(E) A,B,C.D,E 

Inequality Answer Key 


 
Q1-2 
Q2-4 
Q3-1 
Q4-3 
Q5-3 
Q6-2 

Plutus Academy-(58)
Q7-5 
Q8-1 
Q9-4 
Q10-1 
Q11-3 
Q12-1 
Q13-2 
Q14-4 
Q15-5 
Q16-4 
Q17-​D 
Q18-A 
Q19-C 
Q20-B 
Q21-E 
Q22-C 
Q23-C 
Q24-C 
Q25-B 
Q26-4 
Q27-2 
Q28-4 
Q29-4 
Q30-5 
Q31-3 
Q32-(A) A < E ≥ B > D > F = G < C 
Q33-(B) > 
Q34-(C) D,B,A,C,E 
 
 
 
 
For video solution: ​https://www.youtube.com/watch?v=oxBIAvZKxco&t=228s 
   
 
 
 

 
 
 
 
 

Plutus Academy-(59)
Blood Relations 
In these type of questions, information is given about the different relationships among different 
people.  The  relationship  can  be  direct  or  indirect  .  After  examining  the  question  carefully  we 
have to identify the relationship between people and answer the given questions accordingly. 
 
Basic term used in blood relations question: 
It  is  important  to  be aware of the terminologies used in blood relations questions. We are aware 
of the relationships like mother, father,husband, wife, brother, sister, son and daughter.  
Given below are few relationships: 
Parents:​ Mother and father. 
Children :​ Son or Daughter 
Siblings :​ Brother or Sister( of the same parents) 
Spouse :​ Husband or wife. 
Aunt:  ​In  english  aunt  means  father’s  sister,  mother’s  sister,  father’s  brother’s  wife  or  mother’s 
brother’s wife. 
Uncle:  ​In  english  uncle  means  father’s  brother,  Mother’s  brother,  Father's  sister’s  husband, 
mother’s sister’s husband. 
Niece :​ Brother’s and sister’s daughter. 
Nephew : ​Brother’s and sister’s son. 
Cousins : ​Children of aunt and uncle​ . 
Father in-law : ​Father of spouse 
Mother in-law : ​Mother of Spouse. 
Sister in-law ​could be​ ​sister of spouse, wife of brother, wife of spouse’s brother. 
Brother in-law ​could be brother of spouse, husband of sister, husband of spouse’s sister. 
 
A  relation  on  the  mother’s  side  is  called  ​“  ​maternal​”  ​while  that  on  the  father’s  side  is  called 
“​paternal​”. ​Thus mother’s brother is ​“​ maternal uncle​” ​while father’s brother is​ “​paternal uncle” 
 
Generations: ​Generation can be depicted as 
​(G=Grand) 
(Gr=Great) 
 
Gr.G Father Gr.G Mother 
G Father G Mother 
Father Mother 
I/We 
Son Daughter 
Grand son Granddaughter 
Gr.G Son Gr.G Daughter 
  
 

Plutus Academy-(60)
Relationship Chart 
Type  Relation 

Mother’s Or father’s son  brother 

mother’s or father’s daughter  sister 

mother’s or father’s brother  uncle 

mother’s or father’s sister  aunt 

Mother’s or father’s father  grandfather 

mother’s or father's mother  grandmother 

son’s wife  daughter in law 

daughter’s husband  son-in-law 

husband’s or wife’s sister  sister in law 

husband’s or wife’s brother  brother in law 

brother's/sister’s son  nephew 

brother’s/sister’s/ daughter  niece  

uncle or aunt’s son or daughter  cousin 

sister's husband  brother in law 

Brother’s wife  sister in law 

grandson or granddaughter of daughter  great granddaughter 

Grandfather’s grandson  Brother 

Grandfather’s granddaughter  Sister 


 
TRICKS TO SOLVE BLOOD RELATION QUESTIONS 
While  solving  blood  relation  question,  it  is  convenient  if  the  given  information  is  depicted  in 
pictorial  form.  Hence  family  tree  will  be  used  for  representation.  For  easy  calculation,  few 
symbols will be used to denote the relationships in the family tree. The symbols are as follows: 

​denotes ​Male 

​denotes ​female 

Plutus Academy-(61)
 
​denotes couple 
denotes siblings. 

​ denotes ​generation gap 


 
There are three types of questions in blood relationship​. 
1. Indicative form also known as Pointing/ introducing form 
2. Puzzle form 
3. Coded form 
 
Indicative Type 
 
Q1. Introducing a man, a woman said” his mother is the only daughter of my father”. how is the 
man related to the woman?  (Bank PO 1998) (SSC CGL Tier-I 2015)  
1) brother 
2) son 
3) father 
4) uncle 
Sol: ​The man is the son of the woman. 

 
 
Q2. Pointing to a girl in the photograph, Amar said ,” her mother’s brother is the only son of my 
mother’s father.My mother has only one sibling ” How is the girl’s mother related to Amar? 
a) Mother 
b) sister 
c) aunt 
d) grandmother 
e) none of these 
Sol: ​The girl’s mother is also Amar’s mother. 

Plutus Academy-(62)
 
 
Q3.  Pointing  to  a  gentleman,  Deepak  who  is  male  ,said”  his  only  brother  is  the  father  of  my 
daughter's father.” how is the gentleman related to Deepak? (Bank PO 1997) 
a) Grandfather 
b) Father 
c) Brother-in-law 
d) Uncle 
e) none of these 

 
Sol: Uncle 
 
Q4.  Pointing  towards  a girl raj said that she is the daughter of my mother's daughter then how is 
the girl related to Raj? 
a) niece 
b) daughter 
c) mother 
d) aunt 
e) none of these 
Sol: ​Niece 

Plutus Academy-(63)
 
Puzzle Type 
Q5. L is the Father of P, who is the father of S.M is the grandmother of R, who is the son of O.N 
is paternal uncle of R, who is brother of S. What is the relationship between O and R?  
a) Mother and son 
b) uncle and nephew 
c) Father and son 
d) No relation 
e) Cannot be determined 
 
Q6. A,B,C,D,E and F are 6 member of a family.A is the son of C. and E is the daughter of A.D is 
the daughter of F who is the mother of E.B is the spouse of C. Find number of male and 
Female .Also find the relationship between B and D? 
a) 2,4,no relation 
b) 2,4,cannot determined 
c) 4,2,grandfather-granddaughter 
d) 4,2,grandmother-granddaughter 
e) cannot be determined 
 
Q7. Karan has a brother ‘Prem’ and a sister Nisha.Karan’s wife is naaz and has a daughter 
‘Naksha’. Naksha got married with Nisha’s Son Akbar and has a baby girl ‘Riya’. what is relation 
between ‘Naksha’ and Nisha?(SSC CHSL 2015) 
a) Sister 
b) Niece and Aunt 
c) Mother and Daughter 
d) Mother and Granddaughter 
e) none of these 
 
Q8. Among her children, Ganga favourite are Ram and Rekha. Rekha is the mother of the sharat, 
who is loved more by his uncle Mithun. the head of the family is Ram Lal, who is the succeeded 
by his sons Gopal and Mohan.Gopal and Ganga have been married for 35 years and have 3 
children. what is the relation between Mithun and Mohan? 

Plutus Academy-(64)
(SSC CGL 2011) 
a) Uncle 
b) son 
c) brother 
d) no relation 
e) none of these  
 
Coded Form 
 
Study the following information carefully and answer the questions given below: 
P @ Q means ‘P is the sister of Q’ 
P + Q means ‘P is the brother of Q’ 
P $ Q means ‘P is the husband of Q’ 
P % Q means ’P is the mother of Q’ 
P & Q means ’P is the father of Q’ (SBI Management Executive exam 2014)  
 
 
Q9. Which of the following indicates that ‘D is niece of A’? 
a) A+M%R$D@V 
b) B&D$H+A%T 
c) A$F%W@D%M 
d) A+k&D@L$Q 
e) none of these 
 
Q10. What will come in the place of question mark (?) in the expression ‘A+B@C&D?E’ if it 
indicates that C is father in law of E? 
a) & 
b) % 
c) $ 
d) + 
e) @ 
 
Q11. Which of the following can be the correct conclusion drawn from the expression 
‘A%B@C+D$E%F’? 
a) D is daughter of 
b) C is paternal uncle of F 
c) A is the father of C and E 
d) A has two daughters and only one son. 
e) None of these 
 
Q12. Which of the following can be the correct conclusion drawn from the expression? 
‘B%C@D+E&F’? 
a) E is the sister of C and D 

Plutus Academy-(65)
b) C is aunt of E 
c) D is daughter of B 
d) B has two daughters 
e) B is grandmother of F 
 
Directions(13-15): 
Study the following information carefully and answer the questions given below: 
A and Y are brothers of K.Y is the son of P and S.P is the daughter of X.M is the father in law of 
S.Q is the son of X. (SBI Associate PO online Exam 2014) 
 
 
 
 
Q13. If J is the brother of X,then how is J related to Q? 
a) Uncle 
b) Nephew 
c) Cannot be determined 
d) brother in law 
e) son in law 
 
Q14. How is Y related to M? 
a) Nephew 
b) Father 
c) Brother in law 
d) Grandson 
e) Brother 
 
Q15. How is K related to Q? 
a) cannot be determined 
b) niece 
c) daughter 
d) Nephew 
e) Son-in-law 
 
Q 16. Pointing to harish , seema says, “ he is the paternal grandfather of my eldest son 
Mahesh.” how is harish related to seema? 
a) Father -in law b) uncle c) Brother-in-law d) data inadequate 
 
Q 17. ​ Pointing to a photograph, a man said, "I have no brother or sister but that man's father is 
my father's son." Whose photograph was it ? 
a) Himself  b) His Son c) His Father d) His grandfather 
 

Plutus Academy-(66)
Q 18. ​Pointing towards a person in a photograph, ankita said, "He is the only son of the father of 
my sister's brother." How is that person is related to Ankita ? 
a) Anjali Father  b)Anjali Mother c)Anjali Brother d)Maternal Uncle 
Q 19. Pointing to a man, sarita said, "His mother is the only daughter of my mother." How is the 
sarita related to the man ? 
a) Mother b)Daughter c)Sister d) GrandMother 
Q 20. If B’s mother’s was A’s mother’s daughter, how was A related to B? 
a) Uncle b) Auntc) sister d) data inadequate 
Q 21. A is the father of B and C. B is son of A but C is not the son of A. how is C related with A? 
a) Daughter  b) Son c) Niece d) Nephew 
Q 22. A is the mother of B. B’s father C has three children. Based on these information,which of 
following is true? 
a) C has three daughtersb) C has three sons c) B is a male child d) none . 
 
Q23 .Pointing to a woman, Nitin said, "She is the daughter of the only child of my grandmother." 
How is the woman related to Nitin? 
(a) Sister  (b) Niece (c) Cousin  (d) Data inadequate 
 
Q 24. Pointing to a photograph, a man tells his friend, "She is the granddaughter of the elder 
brother of my father." How is the girl in the photograph related to this man? 
(a) Niece (b) Siste (c) Aunt (d) Sister-in-law 
 
Q 25. A man said to a woman, "Your mother's husband's sister is my aunt." How is the woman 
related to the man? 
(a) Daughter  (b) Grand daughter (c) Mother  (d) Sister 
 
Q 26. Dipika said to sarita, “ your mother’s father’s own son is the husband of my sister”. How 
dipika related to sarita. 
a) Sister-in-law  b) Cousin c) Aunt  d) Data inadequate 
Q27. Pointing to the lady in the photograph, Mrinalini said, “Her son’s father is the only 
son-in-law of my mother”. How is Mrinalini related to the lady. 
a) Sister b) Mother c) Cousin d)none 

Plutus Academy-(67)
Q 28. Pointing to a photograph of Harish, Vijaya said, “The father of his sister is the husband of 
my wife’s mother’. How is Vijaya related to Harish? 
a)Brother b) Brother-in-law c) Uncle d) Data inadequate 
Q (29-30). A is a retired officer. He has 3 childrens. His two sons C and E have only one sister D. 
X and Y are the sister-in-law of D, E and C,D respectively. X and D have two children each. The 
grandchildren of A and B are P,Q,R and S. P is the male sibling of Q. Q is the cousin sister of R, S. 
S is husband of O. O has only one daughter Z. U and V are siblings( one male, one female). V is 
the cousin sister of Z. P and R are Unmarried.  
 
 
Q 29. If M is the father of V, how is he related to X. 
a) Son-in-law b) son c) great grandson in law d) cousins 
 
Q30 . If V grows up and gets married and has a baby girl K, then K will belongs to which 
generation of the family starting from A? 
a) First b) third c) fourth d) second 
 
Q 31. ​K is sister of S. S is married to M. M is father of R. Q is son of P. S is mother-in-law of P. M 
has only one son and no daughter. K is married to T. N is daughter of K.How is P related to M ? 
a) Uncle  b).Son c).Daughter in law d)Grandson 
 
Q (32-34)In a family there are two fathers, two mothers, one sister, one brother, two daughter, 
one mother-in-law, one daughter-in-law, two sister-in-law, one son, one grandfather, one 
grandmother and one granddaughter. 
 
Q 32.How many married couples are there in the family ? 
a) three  b) one  c) two d) four 
 
Q 33. How many female members are there in family? 
a) Three  b) two c) four d) one 
 
Q34 .What is the maximum number of members in family? 
a) 8 b) 7 c) 5 d) 6 
 
Q35 . M is daughter of N and sister of L. K is daughter of L and granddaughter of G. F is 
sister-in-law of M. L is only sibling of M. 
How is K related to N ? 
a) Grandson  b)Granddaughter c)Daughter d)Daughter-in-law 
 
Directions (36-37):​ Study the information carefully and answer the questions given below. P is 
the husband of Q. R is the grandchild of P.P has only one child(son) who is married to T’s child. 
T has only two children one son and one daughter. X is grandson of T. S is brother in law of son 
of T. U and V are children of T. W is married to the son of T. X is son of U’s brother.  

Plutus Academy-(68)
(SBI PO MAINS 2017) 
 
Q36. If T is married to Y than how is T related to R?  
(a) Grandfather (b) Grandmother (c) Maternal Grandfather  
(d) Maternal Grandmother (e) Either (c) or (d)  
 
Q37. How is X related to V?  
(a) Son (b) Daughter (c) Son in law (d) Daughter in law (e) Husband 
 
Q38. P's father is Q's son .M is the paternal uncle of P and N is the brother of Q .How is N 
related to M?​ (IBPS CLERK MAINS 2016) 
(a) Brother (b) Nephew (c) Cousin (d) Data inadequate (e) None of the above  
 
Q39. B is the husband of P. Q is the only grandson of E, Who is wife of D and mother - in-law of 
P. How is B related to D? ​(IBPS CLERK MAINS 2016) 
(a) Cousin (b) Son -in-law (c) Son (d) Nephew (e) None of these 
 
Q40. A retiring President was asked who would succeed him as head of the firm , and he replied 
,"The father of my successor is my father's son, but I have no brothers or sons."Who will 
succeed him ? ​(IBPS CLERK MAINS 2016) 
(a) Himself (b) His nephew (c) his Daughter (d) His sister  
 
Q41. Looking at a portrait, a man said ,"That man's father is my father's son. Brothers and 
Sisters I have none." At whose portrait was the man looking ? ​(IBPS CLERK MAINS 2016) 
(a) His own (b) His son's (c) His father's (d) His uncle's  
 
Q42. Pointing to the lady in the photograph , Seema said, "Her son's father is the son-in-law of 
my mother." How is Seema related to the lady? ​(IBPS CLERK MAINS 2016) 
(a) Sister (b) Mother (c) Cousin (d) Aunt  
 
Directions (43-44): ​Read the following information carefully to answer the given questions. 
R is the sister of Q. M is the father of R. V is the son of Q. C is the maternal grandfather of V. M 
does not have married daughter. ​(IBPS PO Prelims 2017) 
 
Q43. How is R related to V? 
(a) Uncle 
(b) Aunt 
(c) Mother 
(d) Can’t be determined 
(e) Nephew 
 
Q44. If B is married to Q, then how is B related to M? 
(a) Grandson 

Plutus Academy-(69)
(b) Son-in-law 
(c) Son 
(d) Daughter-in-law 
(e) Can’t be determined 
 
 
 
 
 
 
 
Blood Relations answer key 
 
Q1=Son 
 

 
 
Q2= 

 
 
Answer-Mother 
Q3= 

Plutus Academy-(70)
 
Answer- Uncle 
 
Q4 

 
Answer-Niece 
 
 
Q5 

 
Answer-Mother and son 
 
Q6 

Plutus Academy-(71)
 
Number of Male-2 
Number of Female-4 
Answer-2,4,cannot be determined 
 
Q7 

 
Niece and Aunt 
 
Q8 
 

 
 
 
Answer-No Relation 
 

Plutus Academy-(72)
Q9 ​@ Sister + Brother $ Husband % Mother & Father 
 
a)A+M%R$D@V 
 

Wrong Option  
 
b)B&D$H+A%T 

Wrong Option as ‘H’ is female. 


 
c)A$F%W@D%M 
 

Wrong option 
 
d)A+k&D@L$Q 

 
 
Answer-(d) 
 
Q10. 

Plutus Academy-(73)
Answer-(C) $ 
 
 
 
 
 
Q11. 
 

 
Answer-(b) C is the paternal uncle of F 
Q12. 
 

 
 
Answer-(E) B is grandmother of F 
 
Q13.(13-15) 
 

Plutus Academy-(74)
Main Question Diagram 
 
Q13. 
 

 
Uncle 
Q14. 
 
Answer-(D)Grandson 

Plutus Academy-(75)
 
 
Q15. Answer-(A) as we don’t know the gender of K. 
Q16. A 
Q17. B 
Q 18. C 
Q19. C 
Q 20. D 
Q 21. A 
Q 22. D 
Q23 A 
Q24. A 
Q25. D 
Q26 C 
Q27 D 
Q28 B 
Q29 A 
Q30 C 
Q31 C 
Q32 C 
Q33 D 
Q34 D 
Q35 B 
Q36 E 
Q37 A 
Q38 E 
Q39 C 
Q40 A 
Q41 A 
Q42 A 
Q43 B 
Q44 D 
 
For video solutions: ​https://www.youtube.com/watch?v=3FDggje9GEk&t=207s 

Plutus Academy-(76)
 

Distance and Direction


N(North)

NW NE

W(West) E(East)

SW SE

S(South)

In this chapter there are three important words to understand: 


  

1.  ​Distance  ​:  It  is  the  measurement  of  the  complete  path  from one point to 
other. 

2. ​Direction ​: It is the path to reach from one point to other. 

3.  ​Displacement ​: It is the direct measurement of the path between one point to 
other. 

There are ​Two ​types of Direction: 


  

1. ​Main direction ​: North, South, East and West 

2.  ​Cardinal  direction  ​:  The  direction  between  two  Main  directions  are  called 
cardinal  direction.  There  are  four  cardinal  directions  namely  :  North-West, 
North-East, South-West and South-East 

Plutus Academy-(77)
Sunrise and Sunset case: ​Sun always Rises from the ​East ​and Sets in the ​West​. 

S​hadow Cases: Two Cases - 


  
Case 1: Morning (sunrise time) 
● If a person is facing towards Sun, the shadow will be towards his back or in west.  
● If a person is facing towards South, the shadow will be towards his right. 
● If a person is facing towards West, the shadow will be towards his front.  
● If a person is facing towards North, the shadow will be towards his left. 
  
Case 2: Evening (Sunset time) 
● If a person is facing towards Sun, the shadow will be towards his back or in East. 
● If a person is facing towards North, the shadow will be towards his right. 
● If a person is facing towards East, the shadow will be towards his front.  
● If a person is facing towards South, the shadow will be towards his left. 
  
  
Rotation of Angles : 
There are angle movement questions in which it is required to have the knowledge of rotation of 
angles. 
 
Clockwise and anticlockwise rotation 
  
Right direction movement is a movement of clockwise rotation whereas left direction 
movement is a movement of anticlockwise rotation. 
 

Plutus Academy-(78)
Types of Question 

A)Distance & Direction Based 

Ex1. ​A man left home and cycled 10 km southwards, turned right and cycled 5km and turned 
right and cycled 10 km and turned left and cycled 10km.How many kilometers will he have to 
cycle to reach his home straight? 

Sol.= ​ E 10km D------------- A

10km 10km

C 5km B
Clearly man cycled A->B=10km,then B->C=5km,then C->D=10km,then D->E=10km. 

Then distance from A->E=ED+DA 

10km+5km=15km 

​Ex2.-​Komal walks 10 km towards North.From there she walks 6km towards south.then she 
walks 3km towards east.How far is she from starting point? 

Sol.= B

10km 6km

3km-→

C D

A AD=?
Clearly Komal moves a->B=10km,then B->C=6km,then C->D=3km.So,Komal ‘s distance from 
starting point i.e A->D 

AD²=AC²+CD² =>4²+3² =>25 

AD= √25  

Plutus Academy-(79)
=>5 

Ex3.​ Sourabh walks 12m towards east then he turn to his right and moves 8m.then he turns to 
his left and moves 6m again he turns to his left and moves 15m and finally he turns to his left 
and moves 18m.Now towards how far is he from starting point and in which direction? 

Sol.=Clearly saurabh walk A->B=12m,then B->C=8m,then C->D=6m,D->E=15m, then 


E->F=18m.So,FA=ED-BC.=>15m-8m=>7m towards North. 

F 18m E

A 12m B 15m

8m

C 6m D
 

Plutus Academy-(80)
B)Angle Based 

Ex1. ​A man is facing south.He turns 135˚ in the anti clockwise direction and then 180˚ in the 
clockwise direction.Which direction is he facing now 

A 135˚

SW 180˚

C
Ex2. ​I am facing South.I turn 100˚ in the clockwise direction and then 145˚ in the anti clockwise 
direction.Which direction am I facing now? 

100˚

A SW(South East)

Plutus Academy-(81)
Exercise
Q1. Kittu walks towards East and then towards South. After walking some distance he turns 
towards West and then turns to his left. In which direction is he walking now? 

a)North b)South c)East d)West 

Q2. A person is driving towards West. What sequence of directions should he follow so that he 
is driving towards South? 

a)left,right,right 

b)right,right,left 

c)left,left,left 

d)right,right,right 

Q3. Richa drives 8km to the south, turns left and drives 5km.Again, she turns left and drives 
8km. How far is she from her starting point? 

a)3km b)5km c)8km d)13km 

Q4. Dingi runs 40 km towards North then turns right and runs 50km. He turns right and runs 
30km, and once again turns right and runs 50km.How far is he from his starting point? 

a)90km b)50km c)10km d)5km 

Q5. If North is called North-West,North-West is called West,West is called South-west and so 
on.What will south-East be called? 

a)East b)West c)North-East  d)South-west 

Q6. A man travels 100km towards South.From there he turns right and travels 100km and again 
turns right to travel 50km. Which direction is he in from his starting point. 

a)North b)North-east c)East d)South-West 

Q7. A train runs 120km in west direction,30 km in south direction and then 80km in east 
direction before reaching the station. In which direction is the station from the train’s starting 
point? 

a)South-West b)North-West c)South-East d)South 

Q8. Facing the west direction, Priya jogs for 20m, turns left and goes further 40m.She turns left 
again and jogs for 20m.Then she turns right to go 20 m to reach the park. how far is the park 
from her starting point and in which direction? 

Plutus Academy-(82)
a)20m South b)40m West c)60m South d)100m East 

Q9.If all the directions are rotated i.e., North is changed to west and east to north and so on, 
then what will come in place of North-West? 

a)South-West b)North-East c)East-North d)East-West  

Q10 A and B start together from one point. They walk 10km towards North, A turns left and 
covers 5km whereas B turns right and covers his 15km. How far is A from B? 

a)18km b)10km c)5km d)8km 

Answer Key 

1)B 6)D 

2)D 7)A 

3)B 8)C 

4)C 9)A 

5)A 10)D 

Previous year Questions with Answer Key 


Q1.Prabir started walking towards South. He took a right turn after walking 10 meters. He again 
took a left turn after walking 20 meters .which direction is he facing now? (Bank PO Exam2011) 

a)South b)North c)West d)East e)Cannot be determined 

Q2.Alok walked 30 meters towards east and took a right turn and walked 40 meters. He again 
took right turn and walked 50 meters. Towards which direction is he from his starting point? 
(Bank PO Exam 2010) 

a)South b)West c)South-West d)South-East e)None of these 

Q3.A and B standing at a distance of 20 km. From each other on a straight East-West road. A 
and B start walking simultaneously ,eastwards and westwards respectively, and both cover a 
distance of 5km. Then A turns to his left and walks 10km.B turns to his right and walks 10 km, 
at the same speed. Then both turn to their left and cover a distance of 5km, at the same speed. 
What will be the distance between them? (Bank PO Exam 2009) 

a)10km b)5km c)20km d)25km e)None of these 

Plutus Academy-(83)
Q4.Maya starts at point T, walks straight to point U which is 4 feet away .She turns left at 90˚ 
right and walks to W which is 4 ft away, turns 90˚ right and goes 3 feet to P, turns 90˚ right and 
walks 1 feet to Q, turns left at 90˚ and goes to V, which is 1 feet away and once again turns 90˚ 
right and goes to R, 3 feet away. What is the distance between T and R? (SSC 2003) 

a)4 feet b)5 feet c)7 feet d)8 feet 

Q5.Namita walks 14 meters towards west, then turns to her right and walks 14 meters and then 
turns to her left and walks 10 meters. Again turning to her left she walks 14 meters. What is the 
shortest distance (in meters) between her starting point and the present position? (SSC 2002) 

a)10 b)24 c)28 d)38 

Q6.You go North, turn right ,then right again and then go to the left. In which direction are you 
now?(SSC 2002) 

a)North b)South c)East d)West 

Q7.Rohan walks a distance of 3 km towards North, then turns to his left and walks for 2km. He 
again turns left and walks for 3 km. At this point he turns to his left and walks for 3km. how 
many kilometers is he from the starting point?(SSC 2005) 

a)1km b)2km c)3km d)5km 

Answer Key: 

1.A 6.C 

2.C 7.A 

3.A 

4.D 

5.B 

 
 
 
 

Plutus Academy-(84)
Practice Set 

Q1.Kanika walks 12 m towards east, then turns to her right and walks 10m.she turns right and 
walks 12m.Again turning to her left she walks 5m.What is the shortest distance(in metres) 
between her starting point and the present position? 

a)10 b)24 c)15 d)17 

Q2.Rani walks towards West and then towards South. After walking some distance he turns 
towards East and then turns to his left. In which direction is he walking now? 

a)North b)South c)East d)West 

Q3.A man is facing North. He turns 120˚ in the anticlockwise direction & then 75˚ in clockwise 
direction then turn 135˚ in anticlockwise direction. which direction is he facing now. 

a)North-East b)South c)South-East  d)South-West 

Q4.A person is driving towards West. What sequence of directions should he follow so that he is 
driving towards South? 

a) left, right, right 

b) right, right, left 

c) left, left, left 

d) right, right, right 

Q5.A biker rides the bike 20km towards East. From there he ride 17 km towards West. Then he 
takes left and ride 4km.How far and in which direction is he with reference to his starting point? 

a)5km South b)5km South-East c)7km East d)7km West  

Q6.Meenu drives 8km to the south, turns left and drives 5km. Again, she turns left and drives 
8km. How far is she from her starting point? 

a)3km b)5km c)8km d)13km 

Q7.A student is searching an institute. He went 60m in west. Then he turns right & walks 
30m.Then he walks 30m towards his right. Again he turns left &walk 10m.Finally before 
reaching the institute he takes right turn and walks 60m.How far did the student from the 
starting point?  

a)60m b)70m c)50m d)40m 

Plutus Academy-(85)
Q8.Deepak runs 40 km towards North then turns right and runs 50km. He turns right and runs 
30km, and once again turns right and runs 50km.How far is he from his starting point? 

a)90km b)50km c)10km d)5km 

Q9.Ravi is standing at point P and facing towards East. He walks straight to point Q which is 5 
feet away. He turns left to reach the point R by walking 4ft. Again he turns 90˚ right & goes 3ft to 
reach the point S. Again He turns 90˚ right and walks 4 feet & reach the destination T. Now, how 
far is he from the starting point? 

a)7 feet b)9feet c)6feet d)8feet 

Q10.If north is called North-West, North-West is called West, West is called South-west and so 
on. What will south-East be called? 

a)East b)West c)North-East  d)South-west 

Q11.I am facing North. I turn 95˚ to clockwise direction & then 155˚ in anticlockwise direction. 
Again 75˚ in anticlockwise direction. Which direction am I facing now?   

a)North-West b)South-West c)South-East d)North-East 

Q12.Gaurav travels 100km towards South. From there he turns right and travels 100km and 
again turns right to travel 50km. Which direction is he in from his starting point? 

a)North b)North-east c)East d)South-West 

Q13.Sunil walked 25m towards South. Then he turned his right & walked 40m .then he turned to 
his right and walked 55m.Now, How far & in which direction is he from the starting position?   

a)30 North-East b)30 North c)40 North d)30 South-East 

Q14.A Bus runs 120km in west direction, 30 km in south direction and then 80km in east 
direction before reaching the destination. In which direction is the destination from the Bus’s 
starting point? 

a)South-West b)North-West c)South-East d)South 

Q15.Lokesh started walking from his office and covers 5km.Then he turns his left and cover 
8km and then turns to his right 45˚ and covers 8 km. Finally turns to his left and covers 3km and 
observe that at last he was going towards which south direction. Find out from which direction 
he starts walking? 

a)North-East b)North-West c)South-East d)South-West 

Plutus Academy-(86)
Q16.Facing the west direction, Nancy jogs for 20m, turns left and goes further 40m.She turns 
left again and jogs for 20m.Then she turns right to go 20 m to reach the park. How far is the 
park from her starting point and in which direction? 

a)20m South b)40m West c)60m South d)100m East 

Q17.A train facing west direction & travels 60km.Then it takes right turn & travels 40km.Then it 
takes right turn & travel 90km to reach the destination. Find the straight distance to reach the 
starting point from the destination point? 

a)60km b)30km c)40km d)50km 

Q18.If all the directions are rotated i.e., North is changed to west and east to north and so on, 
then what will come in place of North-West? 

a)South-West b)North-East c)East-North d)East-West  

Q19.Point A is on the west of Point B. Point C is on the east of Point A. Point D is on the west of 
Point B. Point C is on the east of Point E. If Point D is on the west of Point A, in which direction 
from Point C is Point D.  

a)East b)West c)North d)South 

Q20.A and B start together from one point. They walk 10km towards North, A turns left and 
covers 5km whereas B turns right and covers his 15km. How far is A from B? 

a)18km b)10km c)5km d)8km 

Q21.after starting from a point, Dewakar walks 3km towards East, then turning to his left he 
moves 3km.After this he again turns left and moves 3km.Which choice given below indicates 
the correct direction in which he is from his starting point?   

a)North b)East c)West d)South  

Q22.Sourabh walks 12m towards east then he turn to his right and moves 8m.then he turns to 
his left and moves 6m again he turns to his left and moves 15m and finally he turns to his left 
and moves 18m.Now towards how far is he from starting point and in which direction? 

a)7m towards North b)7m towards south c)8m towards West d)8m towards East 

Q23.Somya is facing North. She turns right and make an angle of 135˚.then she make an angle 
180 ˚ in anticlockwise direction. Which direction she is facing now?  

a)North b)North-West c)East d)West 

Plutus Academy-(87)
Q24.A dog is trying to catch a rat and runs 20m towards east and turns to right, runs 10m and 
turns to right, runs 9 and again turns to left, runs 5m and then turns to left, runs 12 and finally 
turns to left and runs 6m.Now,which direction is the dog facing? 

a)South b)West c)East d)North 

Q25.I went 15m to the west from my house, then turned left and walked 20m.Again I turned to 
the East and walked 25m and finally ,turning to left, I covered 20m.In which direction am I from 
my house?  

a)10m b)5m c)80m d)40m 

Answer Key 
 
1)15  
2)South  
3)South  
4)right,right,right 
5)5km South-East 
6)5km 
7)50m 
8)10km 
9)8feet 
10)East 
11)South-West 
12)South-West 
13)North-West 
14)South-West 
15)North-West 
16)60m South 
17)50km 
18)South-West 
19)West 
20)8km 
21)North 
22)7m towards North 
23)North-west 
24)North 
25)10m 
 
For video solutions: ​https://www.youtube.com/watch?v=of5WnNYIrEU&t=8s 

Plutus Academy-(88)
Coded Distance and Direction: 
Direction (1-4): Read the questions carefully and answer the following questions. (​IBPS PO 
MAINS 2017) 
P@Q: P is to the west of Q; P is either 2 or 12 km west of Q.  
P#Q: P is to the east of Q; P is either 2 or 12 km east of Q.  
P$Q: P is to the North of Q; P is either 5 or 9 km north of Q.  
P&Q: P is to the south of Q; P is either 5 or 9 km south of Q.  
P@$Q: P is to the north west of point Q.  
(Note: Distance between P and Q is not known).  
P&#Q: P is to the south east of Point Q.  
(Note: Distance between P and Q is not known).  
 
1. P@Q, R&#P, Q$R. If the distance between P and R is 13 find the distance between Q and R?  
A. 12 B. 2 C. 5 D. 9 E. None of these  
 
2. U @Q, U$T, T#S, R#T, which of the following cannot be the distance between S and R?  
A. 14 B. 24 C. 4 D. 16 E. None of these  
 
3. S@$T, T@Z, X&Z; X is in which direction with respect to S?  
A. North east B. South east C. South west D. North west E. None of these  
 
4. S&#T, T#Q, R@$Q; What is the position of R with respect to S?  
A. North east B. North west C. North D. South E. South west  
Solution: 
1. Ans. C  
P@Q means P is to the west of Q; P is either 2 or 12 km west of Q  
R&#P means R is to the south east of Point P.  
Q$R means Q is to the North of R; Q is either 5 or 9 km north of R.  
By the given statements, 
Now only condition satisfying the given condition will be only if the distance between  
P and Q = 12  
Q and R = 5  
By Pythagoras theorem,  
PR ² = PQ ² + QR ²  
13 ²= 12 ² + 5 ²  
169= 169  
Hence option c is the right answer.  

 
2 Ans.​ D  
U @Q means U is to the west of Q; U is either 2 or 12 km west of Q.  
U$T means U is to the North of T; U is either 5 or 9 km north of T.  
T#S means T is to the east of S; T is either 2 or 12 km east of S.  
R#T means R is to the east of T; R is either 2 or 12 km east of T.  

Plutus Academy-(89)
By the given statements.  
From the given combinations,  
Distance between S and R can be  
i. 2+12=14  
ii. 12+12=24 
iii. 2+2= 4  
Combination of 16 cannot be possible.  
Hence option D is the right answer. 

 
 
3. Ans. B  
S@$T means S is to the north west of point T.  
T@Z means T is to the west of Z; T is either 2 or 12 km west of Z  
X&Z means X is to the south of Z; X is either 5 or 9 km south of Z  
By the given statements:  
Clearly X is in South east of point S.  
Hence option b is the right answer. 

 
 
4. Ans. B  
S&#T;​ ​S is to the south east of Point T.  
T#Q; T is to the east of Q; T is either 2 or 12 km east of Q.  
R@$Q; R is to the north west of point Q.  
By the given statements;  
Clearly, R is to the northwest of point S. 

 
 

Plutus Academy-(90)
ORDER AND RANKING 
 
Type 1.​ ​To Find Total Number of Person in a Row
Total Number of Person= (Persons From Left+ Person From Right) - 1
Ex: In a Class A is 17th from the top and 33rd from bottom. How many students are there in the
class?
Solution = 17+33-1=>29
Position of a Person From One Side = ( Total Person- Position of Person From Given Side)+1
Ex: In a class of 50 students A is 17th from the top, what its position from the bottom?
Solution= (50-17)+1=>34

Type 2.​ To find Number Of Person Between two or more than two persons
● When there is no overlapping and find person between them
Total Persons-(sum of position of persons from both the sides.)
Ex: In a class of 50 students A is 15th from the top and S is 15th from the bottom. How many
student are there between A and S?
Solution: 50-(15+15) => 20
● When there is a overlapping between the persons
(Sum of positions of two different persons from both side) - Total number of person - 2.
Example: In a class of 50 students A is 35th from the top and S is 35th from the bottom. How
many students are there between A and S?
Solution : (35+35)- 50-2=>18

Type 3. If Total number of person asked and position of different persons are given, it is always
a case of “ cannot be determine”

Type 4.​ If positions are interchange.


Example: In a row Vivek is 12th from left side of row and Jitesh is 18th from right side of row. If
they interchange their positions vivek become 25th from left. Then Find
1. New Position of Jitesh From right.
2. Total number of person
3. Number of Persons Between them.
Solution:1. As Vivek position changes from 12 to 25 from left end so there is an increase of 13
rank so new position of jitesh from right side = 18+13=>31
2. Total no. of person
As vivek Position changes from 12 to 25 from lft means person from left are 11+ 13=> 24
add all left and right persons= 24+17+1=> 42 Total person.
3. Number of person between Them
(Position of Vivek from left after interchange- Position of vivek before interchange)-1
(25-12)-1=>12
In this type of questions, we have to identify the position of given person or sequence of order
(ascending, descending...etc.) of persons

Plutus Academy-(91)
EXERCISE 
 
Q1.​In a class of 50 students A is 17th from the top,what its position from the bottom? 
 
Q2.​In  a  Class  A  is  17th  from  the top and 33rd from bottom. How many students are there in the 
class? 
 
Q3.​In  a  class  of  50  students  A  is  15th  from  the  top  and  S  is  15th  from  the  bottom.How  many 
student are there between A and S? 
 
Q4.​In  a  class  of  50  students  A  is  35th  from  the  top  and  S  is  35th  from  the  botttom.How  many 
students are there between A and S? 
 
Q5.I​n  a  class  Amit  is  11th  from  the  left.Suraj  is  15th  from  the  right  if  their  positions  are 
interchange then the rank of amit from left is 15th. 
(i)How many students are there in a class? 
(ii)What is suraj’s rank from the right after interchange 
(ii)How many students are there b/w Amit and Suraj? 
 
Q6.​Four  kinds  P,Q,R  and  S  are  up  on  the  ladder.P  is  further  up the ladder than Q,Q is between P 
and R.If S is further up than P,who is the third from the bottom?(SSC 2012) 
a)Q b)R c)P d)S 
 
Q7​.Priti  scored  more  than  rahul.yamuna  scored  as  much  as  divya  .Lokita  scored  less  than 
Manju.Rahul  scored  more  than  yamuna.Manju  scored  less  than  divya.Who  scored  the 
lowest?(SSC CGL 2013) 
a)Rahul b)Manju c)Yamuna  d)Lokita 
 
Q8​.Akshay  is  16th  from  the  left  end  in  the  row  of  boys  and  vijay  is  18th  from  the  right 
end.Avinash  is  11th  from  akshay  towards  the  right  end  and  3rd  from  vijay  towards  the  right 
end.How many boys are there in the row?(bank PO 2004) 
a)Data inadequate b)42 c)40 d)48 e)None of these 
 
Study the following information carefully and answer the questions which follow: 
There  are  five  men,Anuj,Kunal,Sourav,Rahul  and  Harish.The  one  who  is  tallest  is  not  the 
youngest.Kunal  is  older  than  only  Harish.Sourav  is  older  than  Rahul  but  shorter  than  him.Only 
one  person  is  taller  than  Rahul.Anuj  is  shortest  while  younger  than  only  Sourav  and  Rahul.Only 
two men are shorter than Sourav. 
Q9.​Which of the following men is third tallest of the five? 
a)Sourav b)Rahul c)Harish d)Kunal e)Kunal or Rahul 
 
Q10.​Who among the following men is tallest? 
a)Sourav b)Rahul c)Kunal d)Harish e)Sourav or Harish 

Plutus Academy-(92)
 
Q11.  ​If  the  five  men  are  made  to  stand  in  a  line  according  to  the  height,first  in  ascending 
order,then in descending order,then whose position will remain the same in both arrangement? 
a)Harish b)Rahul c)Kunal d)Sourav e)Rahul and Anuj 
 
Q12​. Who among the following is older than kunal but younger than Rahul? 
a)Harish b)Sourav c)Anuj d)Data Inadequate e)Sourav or Anuj 
 
Q13. ​Who among the following is taller than Anuj only? 
a)Sourav b)Harish c)Rahul d)Kunal e)Kunal or Harish  
 
Q14.  ​Five  birds  Crow,Pigeon,Little  Pigeon,Big  Crow  and  Eagle  fly  one after the other from a tree 
branch.Big  Crow  flew  after  Crow  but  is  ahead  of  Eagle.pigeon  is  between  Crow  and  Big 
Crow.Little Pigeon is before crow.Which bird is the Last? 
a)Pigeon b)Big Crow c)Eagle d)none of these 
 
Q15.  ​In  a  row  of  trees  one  tree is the 7th from either end of the row.how many trees are there in 
the row?(SSC steno 2011) 
a)11 b)13 c)15 d)14 
 
Q16. ​Suit ranks 18th in a class of 49 students. What is his rank from the last?
a) 18 b) 19 c) 31 d) 32

Q17​. How many 6s are there in the following number series, each of which is immediately
preceded by 1 or 5 and immediately followed by 3 or 9.
2 6 3 7 5 6 4 2 9 6 1 3 4 1 6 3 9 1 5 6 9 2 31 6 5 4 3 2 1 9 6 7 1 6 3
a) None b) One c) Two d) Three

Q18​. Bharat is 8 ranks ahead of Divya who ranks 26 in a class of 42. What is Bharat’s rank
from the last?
a) 9th b) 24th c) 25th d) 34th

Q19​. ​In a row of trees, one tree is fifth from either end of the row. How many trees are there in
the row ?
a) 8 b) 9 c) 10 d) 11

Q 20. ​In a queue, Amrit is 10th from the top while Mohit is 25th from bottom and Meena is just in
the middle of the two. If there be 50 persons in the queue. What position does Meena occupy
from the top ?
a) 18th b) 19th c) 20th d) 15th

Q21. In a row of girls, Shilpi is eighth from the left and Rita is seventeenth from the right. If they
interchange their positions, Shilpi becomes fourteenth from the left. How many girls are there in
the row?
a) 34 b) 35 c) 30 d) 37

Plutus Academy-(93)
Q22. In a row of thirty five children, A is fifteenth from the right end and there are ten children
between A and B. What is B's position from the left end of the row?
a) 15 b) 5 c) 30 d) Data inadequate

Q23.​In the series given below,count the number of 9s,each of which is not immediately
preceded by 5 but is immediately followed by either 2 or 3.How many such 9s are there?
1 9 2 6 5 9 3 8 3 9 3 2 5 9 2 9 3 4 8 2 6 9 8
a) One b)Three c)Five d)Six

Q24. In a row of girls Kamala is 11th from the front. Neelam is 3 places ahead of Soni who is
22nd from the front. How many girls are there between Kamala and Neelam in the row?
a) Six b) Eight c) Seven d) Cannot be determined

Q25 ​In a row of 40 students facing North, Kaitan is 6th to the left of Suman. If Suman is 30th
from the left end of the row, how far is Kaitan from the right end of the row?
​a) 17 b) 16 c) 15 d) 26

Q26. In a row of Girls, there are 16 girls between Priya and Natasha. priya is 32nd from the lft
end of the row. If Priya is nearer than Natasha from the right end of the row, then how far away
is natasha from the left end of the row?
a) 14th b) 15th c) 16th d) data inadequate

Q27​. If Ankit finds that he is twelfth from the right and forth from the left , how many boys should
be added to the queue so that there are 28 boys in the line?
a) 12 b) 13 c) 14 d) 20

Q28​. Mohan is older than Prabir, Suresh is younger than Prabir. Mihir is older than Suresh but
younger than Prabir. Who among the four is the youngest?
a) Prabir b) Mihir c) Mohan d) Suresh

Q29. In a class of 10 girls and 20 boys, Jayanti’s rank is 4 among the girls and 18 in the class.
What is Jayanti’s rank among the boys in the class?
a) Cannot be determined b) 16 c) 14 d) 15

Q30​. Among M, N, P, R and T each one has secured different marks in an examination. R
secured more marks than M and T. N secured less marks than P. Who among them secured
third highest marks?
a) N b) R c) M d) None

Q31. ​Among A, B, C, D and E each having a different weight, D is heavier than A and E and B
is lighter than C. Who among them is the heaviest?
a) D b) B c) C d) Data inadequate

Plutus Academy-(94)
Q32. ​In a class, among the passed students, Amisha is twenty-second from the top and Sajal,
who is 5 ranks below Amisha, is thirty-fourth from the bottom. All the students from the class
have appeared for the exam. If the ratio of the students who passed in the exam to those who
failed is 4 : 1 in that class, how many students are there in the class?
a) 60 b) 75 c) 90 d) Data inadequate

Q33 In a row of girls, there are 16 girls between Priyanka and Nita. Priyanka is thirty-second
from the left end of the row. If Priyanka is nearer than Nita to the right end of the row, then how
far away is Nita from the left end of the row?
a) Data inadequate (b) 14th (c) 15th (d) 16th

Q34. ​In a row of boys, if A who is tenth from the left and B who is ninth from the right
interchange their positions, A becomes fifteenth from the left. How many boys are there in the
row?
a) 23 b) 27 c) 28 d) 31

Q35. In a row of 40 boys, Satya was shifted 10 places to the right of Rohit and Kewat was
shifted 10 places to the left of Vikas. If Vikas was twenty-sixth from the left and there were three
boys between Kewat and Satya after shifting, what was the position of Rohit in the row?
a) 10th from the right end b) 10th from the left end c) 39th from the right end d)
Data inadequate
 
Answer Key- 
1.Sol-​Total student=50 
A’s position from top=17 
50-17=33 
33+1=34th (A’s rank from the bottom).  
 
2.Sol​-A’s position from the top=17 
A’s position from the bottom=33 
Total  =50 
Total student=50-1=49 
 
3.​A’s position from the top=15 
S’s position from the bottom=15 
total=30 
50-30=20 
4.Solution​-Total students=50 
Both positions=35+35=70 
70is more than 50,so it is overlapping case.then 
70-50=20 

Plutus Academy-(95)
20-2=18 
5.  
(i)S rank=15(before interchange) 
A rank=15(after interchange) 
15+15=30-1(29)total student 
 
(ii)Amit rank from left is 11 before interchange 
=29-11=18+1=19 
19 rank from right of Amit 
 
(iii)total number of student between A and S=3.there are two method to find this. 
(a)29-(15+11)=3 
(b)11-----15.there are only 3 student i.e 12,13,14. 
6.Answer-P 
7.Answer-Lokita 
8.Answer-None of these as no of boys are-41 
9.Answer-Sourav 
10.Ans-Kunal 
11.Ans-Sourav 
12.Ans-Anuj 
13.Ans-Harish 
14.Ans-Little Pigeon->Crow->Pigeon->Big Crow->Eagle 
15.Answer=13 
16. 32
17. d
18. 25
19. b
20. a
21. c
22. d
23. b
24. d
25. a
26 c
27 b
28 d
29 d
30 d
31 d
32 b
33 c
34 a
35 d

Plutus Academy-(96)
SYLLOGISM 
 
In Syllogism Questions, Some statements are given followed by some conclusions. We have to 
examine all statements carefully and check that given conclusions follows or not.  
 
Ex- 
 
Statements- 
------------------------------ 
------------------------------ 
------------------------------ 
Conclusions- 
----------------------------- 
----------------------------- 
----------------------------- 
 
Types of Statement- 
 
1) According to Positivity or Negativity- 
a) Positive type statements- Positive types statements include below types- 
  i) All and ii) Some 
b) Negative type statements- Negative types statements include below types- 
i) Some not and ii) No/All not 
2) According to truthness 
a) Definitely true or 100% true 
b) Possibility 
c) Definitely false or 100% false 
 
A) ​Syllogism with Definitely truth statements- 
 
In Normal Situation, you will see 4 types of statement in the questions. 
1) All A are B 
2) Some A are B 
3) No A are B  
4) Some A are not B 
 
Now we will see according to these statements which conclusions will follow or true by using 
venn diagram . 
 
 
 
 

Plutus Academy-(97)
1. Statement: All A are B

Conclusions: All A are B ✔ 


Some A are B ✔ 
Some B are A ✔ 
Some A are not B ×
Some B are not A ×
All B are A ×
No A are B ×
No B are A ×

2. Statement: Some A are B

Conclusions: Some A are B ✔ 


Some B are A ✔
Some A are not B × 
Some B are not B ×
All A are B ×
All B are A × 
No A are B × 
No B are A × 

Plutus Academy-(98)
3. Some A are not B

Conclusions:  Some A are not B ✔ 

4. No A are B or All A are not B

Conclusions: No A are B ✔ 
No B are A ✔ 
Some A are not B ✔
Some B are not A ✔
Some A are B× 
Some B are A ×
All A are B ×
All B are A ×

There are certain statements which consist of ‘atleast some’, ‘few’, ‘many’, ‘more’ , ‘very few’, any 
percentage from 1 to 99 can be converted into ‘​some​’ type stattement. 

Also, you might find a statement ​“Only A are B”​ which can be converted into ​“All A are B” 

Plutus Academy-(99)
(B) Conditions for ‘Either- Or’
1) Requires same subject and predicate in two different conclusion.
2) Relation must not be clear between the subject and predicate(doubtful conclusion).
3) One conclusion must be positive and another one must be negative..
4) ‘Either-or’ is not allowed between ‘All’ type and ‘No’ type conclusion
5) Cases in which we can say either can come- 
a) Some (+ statement) and No (- negative statement) 
b) All (+ statement) and Some not (- negative statement)
c) Some(+ statement) and Some not (- negative statement)

Statements:  Some number are alphabets. 


No Alphabets are vowel. 
Conclusion: Some number are not vowel. ×
Some number are vowel.​× 
Individually they are wrong but forms complementary pairs hence either will follow. 

(C) Condition for “No Statement”-


In No statements we expand the figure to check whether a figure can enter in other figure or not. 
Example: 
Statements: Some number are alphabets 
No alphabets are vowel 

Conclusions: No vowel are numbers ×


No number are vowel ×
All vowels are numbers ×
All numbers are vowel × 

Plutus Academy-(100)
Previous year Questions:- 
Directions:  ​In each of the following questions below are given some statements followed by
some conclusions. You have to take the given statements to be true even if they seem to be at
variance with commonly known facts. Read all the conclusions and then decide which of the
given conclusions logically follows:

Q1.  Statements: All building are rains 


All paper are building 
All dogs are paper 
Conclusions:  
A. All dogs are rains 
B. Some papers are rains 
C. Some rains are building 
D. Some rain are papers (Bank PO 2005) 
 
Q2. Statements: All pencils are birds 
All birds are skies 
All skies are hills 
Conclusions:  
A. All pencils are hills 
B. All hills are birds 
C. All skies are pencils 
D. All birds are hills (SBI PO 2005) 
 
Q3. Statements: Some fruits are flowers 
No flower is a boat 
All boats are rivers 
Conclusions: 
A. Some fruits are river 
B. Some river are boats 
C. Some river are fruits 
D. Some flower are fruits (SBI PO 2002) 
 
Q4. Statements: Some numbers are digits 
All digits are alphabets 
No alphabet is vowel 
All consonants are vowels 
Conclusions: 
A. No digit is a vowel 
B. No alphabet is a consonant 
C. No vowel is a number 
D. All vowels are numbers. (IBPS PO 2013) 

Plutus Academy-(101)
Q5. Statements: Some flowers are skies 
Some skies are rooms 
Some room are window 
Conclusions:  
A. Some windows are skies 
B. Some rooms are flowers 
C. No sky is window 
 
Q6. Statements: No group is people 
All people are animals 
All animals are plants 
All plants are roots 
Conclusions:  
A. No group is plant 
B. All people are plants 
C. At least some group are plants 
D. All plants are groups (IBPS PO 2013) 
 
Q7. Statements: All documents are files 
Some files are papers 
All papers are certificates 
No certificate is a manuscript 
Conclusions:  
A. At least some documents are papers 
B. No manuscript is a document 
C. At least some certificates are files 
D. No paper is a document (IBPS PO 2013) 
 
Q8. Statements: Some jobs are vacancies 
All jobs are works 
No work is trade 
All professions are trades 
Conclusions:  
A. At least some works are jobs 
B. No vacancy is a trade 
C. No professions is a work 
D. At least some works are professions (IBPS PO 2013) 
 
Q9. Statements: All marks are grades 
No grade is a score 
All letters are scores 
All scores are characters 
 

Plutus Academy-(102)
Conclusions:  
A. At least some letters are grades 
B. At least some scores are grades 
C. At least some characters are marks 
D. Some characters are letters (IBPS PO 2013) 
 
Q10. Statements: Some spoon are bowls 
All bowls are knives 
All knives are forks  
Conclusions:  
A. All spoon are forks 
B. All bowls are forks 
C. Some knives are bowls 
D. Some forks are spoon (Bank PO 2006) 
 
Q11. Statements: Some tigers are lions 
Some lions are rabbit 
Some rabbits are horses 
Conclusions:  
A. Some tigers are horses 
B. Some rabbits are tigers 
C. Some horses are lions 
D. All horses are rabbits (RBI 2004) 
 
Q12. Statements: No Man is sky 
No sky is road 
Some men are roads 
Conclusions:  
A. No road is Man 
B. No road is sky 
C. Some skies are men 
D. All roads are men (Bank PO 2003) 
 
Q13.  Statements: All jungles are tigers 
Some jungles are horses. 
Conclusions:  
A. Some horses are jungles 
B. No horse is jungle. (Bank PO 2005) 
 
Q14.  Statements: All huts are mansions 
All mansions are temples  
Conclusions:  
 

Plutus Academy-(103)
A. Some Temple are huts 
B. Some temples are mansions 
C. All huts are temples 
D. All temple are huts (RBI 2003) 
 
Q15. Statements: All pens are chalks  
All chairs are chalks 
Conclusions:  
A. some pens are chairs 
B. some chalks are pens  (Bank PO 2005) 
 
Q16. Statements: No bat is ball 
No ball is wicket 
Conclusions:  
A. No bat is wicket 
B. All wickets are bats (MAT 2003) 
 
Q17. Statements: No Man is Donkey 
A is a Man 
Conclusions:  
A. A is not donkey 
B. All men are not A  
 
Q18. Statements: All rods are bricks 
Some bricks are ropes 
All ropes are doors 
Conclusion:  
A. Some rods are doors 
B. Some rods are not doors 
C. Some doors are bricks 
D. All doors are ropes (Bank PO 2000) 
 
Q19. Statements: All A are B 
All B are C 
No C is D 
Conclusion:  
A. No D is A 
B. No A is B  
 
Q20. Statements: All building are houses 
No house is apartment 
All apartments are flats 
 

Plutus Academy-(104)
Conclusion:  
A. No Flat is a house 
B. All apartments are flats (Allahabad Bank PO 2011) 
 
Q21. Statements: Some leaves are fruits 
All branches are fruits 
Some roots are branches 
Conclusion:  
A. Some roots are fruits 
B. Some branches are leave 
C. No leaf is branch 
D. Some leaves are roots (Bank PO) 
 
Q22. Statements: No Tree is fruit 
All fruits are stones 
All stones are rains 
Conclusion:  
A. No stone is tree 
B. No rain is tree 
C. Some rain are fruits 
D. Some rain are tree (Bank PO ) 
 
 
 
Q23. Statements: All Petals are flower 
Some flower are buds 
Some buds are leaves 
All leaves are plants 
Conclusion:  
A. Some petals are not buds 
B. Some flower are plants 
C. No flower is plant (Bank PO) 
 
Q24. Statements: No Book is paper 
All paper are red 
Conclusion:  
A. Some books are red 
B. Some red are paper 
C. Some red are not paper 
D. Some paper are white (Bank PO) 
 
Q25. Statements: Some cars are bikes 
No bike is fast 

Plutus Academy-(105)
Conclusion:  
A. Some bikes are cars 
B. Some fast are cars 
C. Some cars are not bikes 
D. All fast are cars (Bank PO)  
 
 
 
Syllogism with Possibility

In this type of questions, conclusions will be in possibility. we have to check these conclusions
(true or Not)as we check conclusions of “No Case”.

The possibility conclusions will contain below keywords-


Can be, can never be, may be, is being etc.

‘Can be’/ “is a possibility”​/ “​may be”​ type conclusions only follows for conclusion where
relation is not clear between the subject and predicate.

‘Can never be’ ​type conclusion only follows for a conclusion that is definitely false.(100% false
conclusion)

Tricks:-
1) If Statement is 100% true then conclusion can’t be 100% false.
2) Some Case:
Statement: Some A are B
Conclusions: Some A are B is a possibility X
Some B are A is a possibility X
Some A are not B is a possibility ✔
Some B are not A is a possibility ✔
All A are B is a possibility ✔
All B are A is a possibility ✔
3) All Case:
Statement: All A are B
Conclusions: All B are A is a possibility X
Some B are not A is a possibility ✔

4) Not Case:
Statement: No conclusion with possibility follow

Plutus Academy-(106)
Previous year Question and Examples
Q1. Statements: All gliders are parachutes
No Parachutes is an airplane
All airplane are helicopters
Conclusions: No helicopter is a glider
All parachutes being helicopter is a possibility
No glider is an airplane
All gliders being helicopter is a possibility (Bank PO)

Q2. Statements: Some mails are chtas


All updates are chats
Conclusions: All mails being updates is a possibility
No update is a mail (Bank PO)

Q3. Statements: No Stone is metal


Some metals are papers
All papers are glass
Conclusions: No glass is metal
At least some glasses are metals
All stone being glass is a possibility
No stone is paper (Bank PO)

Q4. Statements: Some exams are tests


No exam is a question
Conclusions: No question is a test
Some tests are definitely not exams (Bank PO)

Q5. Statements: All forces are energies


All energies are power
No power is heat
Conclusions: Some forces are definitely not power
No heat is force
No energy is heat
Some force being heat is a possibility (Bank PO)

Q6. Statements: No note is coin


Some coins are metals
All plastics are notes
Conclusions: No coin is plastic
All plastic being metal is a possibility
No metal is plastic
All notes are plastics (Bank PO)

Plutus Academy-(107)
Q7. Statements: All building are houses
No house is an apartment
All apartments are flats
Conclusions: No flat is a house
No building is an apartment
All building being flat is a possibility
All apartments being buildings is a possibility (Allahabad Bank PO)

Q8. Statements: All TV are LCD


Some LCD are LED
Conclusions: Some LED are TV
All TV being LED is a possibility (Bank PO)

Q9. Statements: Some parrots are peacocks


No peacocks are sparrows
Conclusions: All sparrow being a parrot is a possibility
At least some sparrow are peacocks

Q10. Statements: Some planes are ships


No ship is a radar
All cars are radars
Conclusions: No car is a ship
All radars being planes is a possibility (NIACL AO Exam 2015 )

Plutus Academy-(108)
REVERSE SYLLOGS
In Reverse syllogs, two conclusion are given followed by two or more statement. You have to 
identify which statements can the conclusions be derived from. 

Tip: These questions should be tackled by omitting the options.  

Direction (1-2): ​Each question consists of some conclusion followed by five statements. 
Consider the given conclusion to be true even if they seem to be at variance with commonly 
known facts. Read the conclusions and then decide from which statement, the conclusion can 
be possible. 

1. Conclusion: 

No toxic is injection. 
Some injection are glucose. 
Statement: 

(a) All toxic are tablet. No tablet is glucose. Some glucose are medicine. All medicine are 
injection 
(b) Some toxic are tablet. All tablet are injection. Some injection are glucose. All glucose are 
medicine 
(c) All tablet are toxic. No injection is toxic. Some glucose are injection. All glucose are 
medicine. 
(d) Some injection are toxic. All injection are tablet. Some toxic are glucose. All glucose are 
medicine 
(e) Some medicine are toxic. Some toxic are injection. Some injection are tablet. All tablet are 
glucose 
 
2.​Conclusion: 
Some clock are watch. 
Some wheel are buses. 
Statement: 
(a) All watch are clocks. Some clocks are tyres. Some tyres are wheels. Some wheels are buses. 
(b) Some watch are wheel. All wheel are tyre. Some tyre are buses. All buses are clock 
(c)Some buses are tyre. Some tyre are clock. All clock are watch. Some watch are wheel 
(d)All tyre are buses. Some buses are wheel. Some wheel are clock. No clock is watch. 
(e)Some buses are watch. Some watch are tyre. Some tyre are clock. Some clock are wheel 
  
Direction (3-10): ​In each group of questions below are two conclusions followed by five set of 
statements. You have to choose the correct set of statements that logically satisfies given 
conclusions. Given statements to be true even if they seem to be at variance from commonly 
known facts. 

Plutus Academy-(109)
3. Conclusions: 

All dogs being monkeys is a possibility. 


Some Monkeys are Cats is a possibility 
Statements: 

1. All dogs are cats. No cat is Pig. All Pigs are Monkeys. 
2. No Cat is dog. No dog is Monkey. No Monkey is pig. 
3. All dogs are cats. No dog is Pig. No cat is monkey. 
4. All Cats are dogs. No cat is Pig. No dog is Monkey. 
5. All dogs are cats. No cat is Monkey. All Pigs are Monkeys. 
A. Only Statements – 1 
B. Only Statements – 2 
C. Only Statements – 3 
D. Only Statements – 4 
E. Only Statements – 5 
 
4. Conclusions: 
No teacher is a lawyer. 
All doctors are students. 
Statements: 
1: All Students are teachers. All teachers are doctors. No doctor is a lawyer. 
2: All doctors are teachers. All teachers are students. Some Students are lawyers. 
3: All doctors are teachers. No teachers is student. Some Students are lawyers. 
4: All doctors are teachers. All teachers are students. No Student is a lawyer 
5: All doctors are teachers. No teachers is student. No Student is a lawyer. 
A. Only Statements – 1 
B. Only Statements – 2 
C. Only Statements – 3 
D. Only Statements – 4 
E. Only Statements – 5 
 
5. Conclusions: 
At least some hills being lake is a possibility. 
Some mountains are hills. 
Statements: 
1:All Hills are Mountains. All hills are rivers. No hill is a lake. 
2: All Hills are Mountains. All Mountains are rivers. No river is a lake. 
3: Some Hills are Mountains. Some hills are rivers. No hill is a lake. 
4: Some Hills are Mountains. All Mountains are rivers. No hill is a lake. 
5: All Hills are Mountains. Some hills are rivers. No river is a lake. 
A. Only Statements – 1 
B. Only Statements – 2 

Plutus Academy-(110)
C. Only Statements – 3 
D. Only Statements – 4 
E. Only Statements – 5 
 
6. ​Conclusions: 
Atleast some beans are carrot. 
Some brinjals being carrot is a possibility 
Statements: 
1: All carrots are beans. Some beans are apples. No brinjal is Carrot. 
2: No carrots is beans. Some beans are brinjals. No brinjal is apple. 
3: All carrots are beans. Some beans are brinjals. No brinjal is apple. 
4: No carrots is beans. Some beans are brinjals. No brinjal is apple. 
5: All carrots are beans. Some beans are apples. No brinjal is carrot. 
 
A. Only Statements – 1 
B. Only Statements – 2 
C. Only Statements – 3 
D. Only Statements – 4 
E. Only Statements – 5 
 
7. Conclusions: 
Atleast some beans are carrot. 
Some brinjals being carrot is a possibility 
Statements: 
1: All carrots are beans. Some beans are apples. No brinjal is Carrot. 
2: No carrots is beans. Some beans are brinjals. No brinjal is apple. 
3: All carrots are beans. Some beans are brinjals. No brinjal is apple. 
4: No carrots is beans. Some beans are brinjals. No brinjal is apple. 
5: All carrots are beans. Some beans are apples. No brinjal is carrot. 
 
A. Only Statements – 1 
B. Only Statements – 2 
C. Only Statements – 3 
D. Only Statements – 4 
E. Only Statements – 5 
 
8. Conclusions: 
All Donkeys being Tigers is a possibility 
All Monkeys being Tigers is a possibility 
Statements: 
1: All monkeys are Donkeys. No Donkey is Tiger. All elephants are Tigers 
2: All monkeys are Donkeys. No Donkey is elephant. All elephants are Tigers 
3: Some monkeys are Donkeys. No Donkey is Tiger. All elephants are Tigers 

Plutus Academy-(111)
4: No monkey is Tiger. No Donkey is elephant. All elephants are Tigers 
5: Some monkeys are Donkeys. No Donkey is elephant. No monkey is Tiger 
 
A. Only Statements – 1 
B. Only Statements – 2 
C. Only Statements – 3 
D. Only Statements – 4 
E. Only Statements – 5 
 
9. Conclusions: 
Some covers are pillows. 
All sheet being cover is a possibility. 
Statements: 
Statements – 1: No pillows is cover. Some covers are bed. No pillow is sheet. 
Statements – 2: Some pillows are covers. Some covers are bed. No Sheet is cover. 
Statements – 3: All pillows are covers. Some covers are bed. No pillow is sheet. 
Statements – 4: Some pillows are covers. All covers are bed. No Sheet is cover. 
Statements – 5: No pillows is cover. All covers are bed. No pillow is sheet. 
A. Only Statements – 1 
B. Only Statements – 2 
C. Only Statements – 3 
D. Only Statements – 4 
E. Only Statements – 5 
 
10. Conclusions: 
All tanks are gingers. 
Some gingers are not fishes 
Statements: 
1: Some tanks are jars. Some jars are ginger. No fish is jar 
2: All gingers are jars. Some jars are tanks. No fish is jar 
3: All tanks are jars. Some jars are ginger. Some fishes are ginger 
4: All tanks are jars. All jars are ginger. No fish is jar 
5: All tanks are jars. All jars are ginger. Some fishes are ginger 
A. Only Statements – 1 
B. Only Statements – 2 
C. Only Statements – 3 
D. Only Statements – 4 
E. Only Statements – 5

 
 
For video solutions: ​https://www.youtube.com/watch?v=Q3eBVDC0OYQ 
Reverse syllogs: ​https://www.youtube.com/watch?v=dPoV2ufeTmo&t=11s 

Plutus Academy-(112)
Seating Arrangement 
 
In this type of question, some clues regarding sitting or placing sequence of any items is 
given.The item can be some person,place,Animal or any Noun.The sequence of arranging the 
item can be in any format for ex-circle,single Row,parallel 
row,square,rectangle,pentagon,hexagon,octagon etc. format. 
 
In sitting arrangement,We have three types/forms of question- 
1)Line/Row-Single or Parallel 
2)Circular table 
3)Square/Rectangle/Pentagon/Hexagon/Octagon/Decagon table 
 
Q1. ​Seven persons A,B,C,D,E,F and G are standing in a straight line. 
D is to the right of G. 
C is between A and B. 
E is between F and D. 
There are three persons between G and B.(SSC CGL 2015) 
Who is on the extreme left? 
a)G b)A c)B d)D 
Ans-G 
Pattern-Left<-G->D->E->F->B->C->A->Right 
 
Q2.​Six girls are standing in such a way that they form a circle,facing the centre.Subbu is to the 
left of pappu,Revathi is between Subbu and nisha,Aruna is between pappu and Keerthana.who is 
to the left of Pappu?(SSC CGL 2013) 
a)Subbu b)Keerthana c)Nisha d)Aruna 
Ans-Subbu 
Pattern= Subbbu<-Pappu<-Aruna<-Keerthana<-Nisha<-Revathi 

Plutus Academy-(113)
Q3.A group of friends are sitting in an arrangement one each at the corner of an octagon.All are 
facing the centre.Mahima is sitting diagonally opposite Rama,who is on sushma’s right.Ravi is 
next to Sushma and opposite girdhar,who is on Chandra’s left.Savitri is not on mahima’s right 
but opposite shalini.Who is on Shalini’s right?(SSC CGL 2010) 
a)Ravi b) Mahima c) Girdhar d) Rama 
Ans-Ravi 

 
Q4.Six person A,B,C,D,E,F are sitting in 2 rows,3 in each.If E is not at any end,D is second to left 
of F,C is neighbour of E and is sitting diagonally opposite to D and B is neighbour of F.Find who 
will be opposite to B? 
a)A b)E c)C d)D 
 
Ans-E 

 
 
Q5.​Four girls are sitting on a bench to be photographed.Shikha is to the left of Reena.Manju is 
to the right of reena.Rita is between Reena and Manju.who would be second from the left in the 
photograph?(Bank PO 1994) 
a)Reena 
b)Shikha 
c)Manju 

Plutus Academy-(114)
d)Rita 
Answer-D(Rita) 
Right-------Manju->Rita->Reena->Shikha-------left 
According to the given persons we have taken Left and Right and Answering the questions 
according to our perception  
 
Q 6 Read the following information carefully and answer the questions given below it: 
(Bank PO 1994) 
(A)There are five friends. 
(B)They are standing in a row facing south. 
(C)Jayesh is to the immediate right of alok. 
(D)Pramod is between bhagat and Subodh. 
(E)Subodh is between Jayesh and Pramod. 
Right--------Bhagat->Pramod->Subodh->Jayesh->alok--------Left 
(As they are facing south) 
 
Q2.Who is at the extreme left end? 
a)Alok 
b)Bhagat 
c)Subodh 
d)Data inadequate 
e)None of these 
Answer-(a) 
 
Q3.Who is in the middle? 
a)Bhagat 
b)Jayesh 
c)Pramod 
d)Subodh 
e)Alok 
Answer-(d) 
 
Q4.To find answers to the above two questions,which of the given statements can be dispensed 
with? 
a)None 
b)A only 
c)B only 
d)C only 
e)D only 
Answer-(b) 
 

Plutus Academy-(115)
Q7 .​Five persons A,B,C,D and E are sitting in a row facing you such that D is on the left of C and 
B is on the right of E.A is on the right of C and B is on the left of D.If E occupies a corner 
position,then who is sitting in the centre? (CBI 1995) 
a)A 
b)B 
c)C 
d)D 
Answer-(d) 
Right--------A--C--D--B--E----Left 
 
Q 8. ​Read the following information carefully and answer the questions given below it: 
In a seven storey building,having floors numbered one to seven,P,Q,R,S,T and V each live on a 
different floor.(the ground floor is numbered floor no 1,the floor above it floor no 2 and so 
on).One of the floors in the building is vacant.P lives on the fifth floor. No floor below fifth floor 
is vacant ALSO no odd numbered floor is vacant.Only S lives between T and V.T does not live on 
an odd numbered floor.T does not live on a floor immediately above or immediately below R’s 
floor.Q does not live on the bottom most floor.(UBI PO 2010) 
Sol-   

 
 
Q1)Who lives on the topmost floor? 
a)Q 
b)S 
c)R 
d)V 

Plutus Academy-(116)
e)No one as it is vacant. 
Answer-Q 
 
Q2)On which of the following floors does R live? 
a)Seventh 
b)Sixth 
c)Third 
d)Second 
e)First 
Answer-First 
 
Q3)Which of the following floors is vacant? 
a)Seventh 
b)Sixth 
c)Third 
d)Second 
e)Fourth 
Answer-Sixth 
 
Q 9 . Study the following information carefully to answer the given questions: 
Twelve people are sitting in two parallel rows containing six people each.in such a way that 
there is an equal distance between adjacent persons.In row-1,P,Q,R,S,T and V are seated and all 
of them facing south.In row-2 A,B,C,D,E and F are seated and all of them are facing north. 
Therefore, in the given seating arrangement each member seated in a row faces another 
member of the row. 
A sits third to right of D,Neither A nor D sits at extreme ends.T faces D.V does not face A and V 
does not sit at any of the extreme ends.V is not an immediate neighbor of T.B sits at one of the 
extreme ends.Only two persons sit between B and E.E does not face V.Two persons sit between 
R and Q.R is not an immediate neighbour of T.C does not face V.P is not an immediate 
neighbour of R. (Corporation Bank PO 2011) 

 
Q1) Who amongst the following sit at extreme ends of the rows? 
a)B,E 
b)S,T 

Plutus Academy-(117)
c)P,R 
d)B,F 
e)none of these 
Ans-(c)P,R 
 
Q2) Who amongst the following faces A? 
a)R 
b)T 
c)P 
d)Q 
e)S 
Ans-(e)S 
 
Q3)How many persons are seated between T and S? 
a)1 
b)2 
c)3 
d)4 
e)5 
Ans-(b)2 
 
Q4)P is related to V in the same way as C is related to F.To which of the following is E related 
to,following the same pattern? 
a)B 
b)D 
c)C 
d)A 
e)none of these 
Ans-(a)B  
 
Circular Table Seating Arrangement 
 
Q1. Study the following information carefully and answer the given questions:- 
A,B,C,D,E,F,G and H are sitting around a circle,facing the centre.A sits fourth to the right of H 
while second to the left of F.C is not the neighbour of F and B.D sits third to the right of C.H 
never sits next to G. (Bank PO 2008) 

Plutus Academy-(118)
 
 
Q1.Who amongst the following sits between B and D? 
Q2.Which of the following pairs sit between H and G? 
Q3.Who is immediate right of A? 
Q4.Who sits second to the right of B? 
Q5.Which is the position of B with respect of C? 
 
Q6.Four of the following are alike in a certain way based on their position in the sitting 
arrangement & so form the group.Which is the one that does not belong to that group? 
AE HF BD GE CH 
Answer-1-F, 2-CE, 3-D, 4-E, 5-B is third to the left of C/B is fifth to the right of C, 
6-AE 
_________________________________________________________________________ 
Study the following information carefully and answer the given questions:- 
A,B,C,D,E,F,G and H are sitting around a circular table facing the centre.Each one of them has a 
different profession viz.Doctor,Engineer,Architect,Pilot,Banker,Teacher,Businessman and 
Politician. 
The politician sits third to right of G.C is an immediate neighbour of G.Architect sits second to 
right of C.B sits third to right of H.H is neither a politician nor an architect.Only one person sits 
between C and the teacher.A and F are immediate neighbours of each other.Neither A nor F is a 
politician.Doctor sits second to right of A.Two people sit between D and the engineer.D is not a 
politician.Pilot is not an immediate neighbour of the politician.Banker sits second to left of A. 
 
 

Plutus Academy-(119)
 
 

 
Q7.Who amongst the following is a businessman? 
Q8.What is the position of F with respect to the politician? 
Q9.who sits exactly between the teacher and the engineer? 
Q10.What is the profession of G? 
 
Q11.Four of the following are alike in a certain way based on the sitting arrangement and thus 
form a group.which is the one that does not belong to that group? 
a)G-Doctor b)E-Architect c)H-BusinessMan d)E-Politician e)D-Pilot 
 
Q12.What is the profession of E?  
 
Q13.Which of the following is true with respect to the given seating arrangement? 
a)D is an immediate neighbour of G 
b)G is a banker 
c)The banker and the teacher are immediate neighbours of each other 
d)Pilot sits exactly between architect and the businessman. 
e)Doctor sits second to the right of the businessman  
 

Plutus Academy-(120)
Answers-7-A,  
8-F is Second to the right of politician/F is sixth to the right of politician 
9-H(Doctor) sits between Teacher and Engineer.  
10.Pilot 
11.E-Politician 
12.Politician 
13.Doctor sits second to the right of the businessman 
 
Study the following information carefully 
Eight colleagues, A,B,C,D,E,F,G and H are sitting around a circular table facing centre not 
necessarily in the same order.Each one of them holds a different post viz. Manager,Company 
Secretary,Chairman,President,Vice-President,Group Leader,Financial Advisor and Managing 
Director. 
A sits third to right of the Managing Director.Only two people sit between the Managing Director 
and H.Vice President and company secretary are immediate neighbour of each other.Neither A 
nor H is a Vice President or a Company Secretary.Vice President is not an immediate neighbour 
of the Managing Director.Manager sits second to the left E.E is not an immediate neighbour of 
H.the Manager is an immediate neighnour of both group Leader and the Financial 
Advisior.Financial Advisor sits third to right of B.B is not the Vice President.C sits to the 
immediate right of Chairman.A is not the chairman.F is not an immediate neighbour of A.G is 
not an immediate neigbhour of the Manager.  
 
14.Who among the following sits third to the left of E? 
15.Who among the following is the president of the company ? 
16.Four of the following five are alike in a certain way based on the given arrangement and thus 
form a group.Which is the one that does not belong to that group? 
a)F-Chairman 
b)G-President 
c)D-Manager 
d)A-Financial Advisor 
e)B-Managing Director 
17.Who amongst the following is the Group Leader? 
18.Who amongst the following sit exactly between the Managing Director and H? 
19.Which post does the B hold in the company? 
Answer- 
14-D(Financial Advisor),15-A,16-B-Managing Director,17-F,18-EF,19-Company secretary 
 

Plutus Academy-(121)
 
Study the following information carefully and answer the questions given below. 
Eight persons -J,K,L,M,P,Q,R and S are sitting around a circular table at equal distances between 
each other,but not necessarily in the same order.Some of the people are facing the centre while 
some others are facing outside(i.e. In a direction opposite to the centre.(Note:-Facing the same 
direction means if one faces the centre then the other also faces the centre and vice 
versa.Facing the opposite directions means if one faces the centre then the other faces outside 
and vice-versa.) 
(NIACL AO -online 2015) 
K sits second to the right of P.K is an immediate neighbour of J.P faces towards centre.P is to 
the immediate left of M.Neither K nor L is an immediate neighbour of M.S sits second to the 
right of M.L is the immediate right of S.R sits third to the right of Q.L and Q face the same 
direction as that of M.Q sits second to the right of J.Both the immediate neighbours of Q face 
the same direction.Both the immediate neighbour of M face the same direction. 
 

Plutus Academy-(122)
 
Q20.How many persons in the given arrangement face outside? 
Q21.Who is sitting third to the left of L? 
Q22.Four of the following five are alike in a certain way based on the given seating arrangement 
and hence they form a group.Which is the one that does not belong to that group? 
1)P 2)Q 3)J 4)L 5)S 
Q23.who sits exactly between K and P if one moves clockwise from K? 
Q24.What is the position of R with respect to Q? 
Q25.Who is sitting second to the left of S? 
Answer- 
20-Five, 
21-M is sitting third to the left of L, 
22-Except P,all others face outside, 
23-Q sits exactly between K and P when one moves clockwise from L, 
24-R is the third to the right of Q, 
25-M is sitting second to the left of S. 
 
Study the following information carefully and answer the questions given below. 
Eight Person E,F,G,H,I,J,K, and L are seated around a square table-two on each side.there are 
three lady members and they are not seated next to each other.J is between L and F.G is 
between I and F.H, a lady member,is second to the left of J.F, a male member,is seated opposite 
E, a lady member.There is a lady member between F and I.(Bank PO 1995) 
 
Q26.Who among the following is seated between E and H? 
a) F b) I c) J d) Cannot be determined e) None of these 
 
Q27.How many persons are seated between K and F? 

Plutus Academy-(123)
a)1 b)2 c)3 d)cannot be determined e)None of these 
 
Q28.Who among the following are the three lady members? 
a)E,G,J b)E,H,G  c)G,H,J d)cannot be determined e)none of these  
 
Q29.Who among the following is to the immediate left of F? 
a)G b)I c)J d)cannot be determined e)none of these 
 
Q30.Which of the following is true about J? 
a)J is male b)J is Female c)Sex of J cannot be determined d)Position of J cannot be 
determined e)none of these.   

 
 
 
 
Answer- 
26-E 
27-C 
28-B 
29-C 
30-A 
 
Study the following information carefully and answer the given questions:- 
Eight friends, Meenal,Rumia, Shikha,Ali,Peter,Harleen,Ketan and Bharat are sitting around a 
rectangular table in such a way that four of them sit at four corners of the rectangle while four 
sit in the middle of each of the four sides.The ones who sit at the four corners face the centre 
while those who sit in the middle of the sides face outside. 

Plutus Academy-(124)
 
 
Bharat sits second to the right of Shikha. Bharat does not sit at any of the corners.Meenal sits 
third to the right of peter.Peter is not an immediate neighbour of shikha.Rumia and Ketan are 
immediate neighbour of each other but Rumia does not sit at any of the corners of the 
table.Harleen is neither an immediate neighbour of Peter nor Shikha.(Allahabad Bank PO 2011)

 
 
Q31.Four of the following five are alike in a certain way and so form a group. Which is the one 
that does not belong to that group? 
1)Peter2)Rumai 3)Harleen 4)Shikha 5)Bharat 
 
Answer-(3) 
 
Q32.Who sits third to the left of Ali? 
1)Bharat 2)Rumia 3)Shikha 4)Peter5)Cannot be determined 
 
Answer-(1) 
 
Q33.What is the position of peter with respect to Meenal? 
1)To immediate Left 
2)Second to the left 
3)Third to the left 
4)Third to the right 
5)Second to the right 
 
Answer-(4) 
 
Q34.Who amongst the following sits second to the right of Ketan? 
1)Shikha 2)Ali 3)Bharat 4)Harleen 5)Meenal 

Plutus Academy-(125)
 
Answer-(4) 
 
Q35.Who amongst the following represent the immediate neighbours of Harleen? 
1)Meenal,Ketan 
2)Bharat,Rumia 
3)Bharat,Meenal 
4)Ali,Rumia 
5)Ali,Ketan 
 
Answer-(5) 
Q36.Who amongst the following is an immediate neighbour of Meenal? 
1)Rumia 
2)Ali 
3)Ketan 
4)Harleen 
5)Shikha 
Answer-(5) 
 
Directions(1-5): Study the following information carefully and answer
the given questions.
M, N, O, D, E, F, G and H are eight members of a family. They belong to three different
generations. There are three married couples. All of them are sitting around a circular table,
facing the centre but not necessarily in the same order.
·​ ​D and F are a married couple. D, the wife of F, sits second to the left of her husband.
·​ ​O and M are offspring of D. M is not an immediate neighbour of his mother.
· ​There is only one person sitting between O and his niece G, but that person is not G’s
father.
· ​E, a bachelor, sits third to the right of his uncle M, but neither to the opposite nor to the
immediate left of his father.
·​ ​G is not an immediate neighbour of her aunt N. No three females are sitting together.
·​ ​O and his sister-in-law are immediate neighbour.

1. Who among the following is G’s father?


a)​ ​F
b)​ M ​
c)​ H

d)​ N ​
e)​ ​Can’t be determined

2. ​How many females are there in the family?


a)​ ​Two
b)​ ​Three
c)​ ​Five
d)​ ​Can’t be determined

Plutus Academy-(126)
e)​ N
​ one of these

3. ​What is the position of D with respect to her granddaughter?


a)​ ​Second to the left
b)​ ​Fourth to the right
c)​ ​Third to the right
d)​ ​Fourth to the left
e)​ ​Immediate left

4. Who among the following is the aunt of E?


b)​ ​D
c)​ G

d)​ H ​
e)​ ​Can’t be determined

5. Which of the following represents a group of male members of the


family?
a)​ ​EN
b)​ ​FG
c)​ ​OG
d)​ ​FM
e)​ ​Can’t be determined

1. b)​ ​ M
2. e)​ ​None of these
3. c)​ ​Third to the right
4. a)​ ​N
5. d)​ ​FM 
 
 

Plutus Academy-(127)
EXERCISE 
Q1: Study the given information carefully to answer the given question. 
Eight  people-  A,  B,  C,  D,  W,  X,  Y and Z are sitting in a circle facing the centre. All eight people are 
from  different  place  i.e.  Okhla,  Dwarka,  Lajpat  Nagar,  Chanakyapuri,  Saket, Mehrauli, Rohini and 
Karol  Bagh  but  not  necessarily  in  the  same  order.  W  is  sitting  third  to  the  left  of  Y.  The  person 
who  is  from  Dwarka is to the immediate right of W and W is not from Okhla. B is sitting fourth to 
the  right  of  Z.  Z  is  not  the  neighbor  of  Y.  Neither  B  nor  Z  is  an  immediate  neighbor  of  W.  X  is 
from  Chanakyapuri  and  is  sitting  third  to  the  right  of  the  person  from  Dwarka.The  person  from 
Mehrauli  is  sitting  second  to  the  left  of  person  from  Chanakyapuri.  The  person  from  Rohini  is 
sitting  second  to  the  left  of  W.  A  who  is  Lajpat  Nagar  is  sitting  exactly  between  X  and  Z.  The 
person  from  Saket  is  sitting  second  to  the  right  of  the  person  from  Lajpat  Nagar.  C  is  sitting 
third to the left of X.​(​IBPS PO 2016 Prelims​) 
1. Who amongst the following persons belongs to Okhla? 
(a) Y (b) D (c) C (d) B (e) None of these 
2. What is A’s position with respect to B? 
(a)  Third  to  the  right  (b)  Second  to  the  right  (c)  Third  to  the left (d) Second to the left (e) Fourth 
to the right 
3.  How  many  people  are  sitting  between  Z  and  C  when  counted  in  an  anticlockwise  direction 
From C? 
(a) One (b) Two (c) Three (d) Four (e) None 
4.  Four  of  the  following  five  pairs  are  alike  in  a  certain  way  based  on  their  positions  in  the 
above arrangement and so from a group. Which of the following does not to the group? 
(a) B-Rohini (b) Z – Mehrauli (c) D – Okhla (d) Y – Saket (e) X – Dwarka 
5. Which of the following statements is false according to the above mentioned arrangement? 
(a) C is to the immediate right of the Karol Bagh 
(b) The person from Lajpat Nagar is third to the right of the person from Mehrauli. 
(c) The person from Dwarka is sitting exactly between the Karol Bagh and the Saket. 
(d) D is neither from Chanakyapuri nor from Karol Bagh. 
(e) There are only three people between A and C. 
 
Q2. Study the given information carefully to answer the given question. 
S,  T,  U,  V,  W,  X,  Y  and  Z  are  sitting  in  a  straight  line  equidistant  from  each  other  (but  not 
necessarily  in  the  same  order).  Some  of  them  are  facing  south  while  some  are  facing  north. 
(Note  :  Facing  the  same  direction  means,  if  one  is  facing  north  then  the  other  also  faces  north 
and  vice-versa.  Facing  the  opposite  directions means, if one is facing north then the other faces 
south and vice-versa) S faces north. Only two people sit to the right of S. T sits third to the left of 
S.  Only  one  person  sits  between  T  and  X.  X  sits  to  the  immediate  right  of  W.  Only  one  person 
sits between W and Z. Both the immediate neighbors of T face the same direction. U sits third to 
the  left  of  X.  T  faces  the  opposite  direction  as  S.  Y  does  not  sit  at  any  of  the extremes ends of 
the line. V faces the same direction as W. Both Y and U face the opposite direction of Z.  
(​IBPS PO 2016 Prelims​) 
1. How many persons in the given arrangement are facing North? 
(a) More than four (b) Four (c) One (d) Three (e) Two 
2.  Four  of  the  following  five  are  alike  in  a  certain  way,  and  so  form  a  group.  Which  of  the 
following does not belong to the group? 
(a) W, X (b) Z, Y (c) T, S (d) T, Y (e) V, U 
3. What is the position of X with respect to Z? 

Plutus Academy-(128)
(a)  Second  to  the  left  (b)  Third  to  the  right (c) Third to the left (d) Fifth to the right (e) Second to 
the right 
4. Who amongst the following sits exactly between Z and W? 
(a) T (b) Y (c) X (d) W (e) U 
5. Who is sitting 2nd to the right of T? 
(a) Z (b)V (c)X (d)W (e)None of these. 
 
Q3. Study the following information and answer the given questions: (​IBPS PO 2016 Prelims​) 
A,  B,  C,  D,  E,  F,  G  &  H  are  eight  friends  and  sitting  around  a  circular  table  but  not  necessarily  in 
same  order.  Some of them are facing inside and some of them are facing outside. A sits third to 
right  of  H.  There  is  two  people  sits  between  H  and  B.  C  sits  second  to  left  of  B.  There  is  three 
people  sits  between  B  and  E.  D  is  second  to  left  of  F,  who  is  not  immediate  neighbor  of  A. 
Immediate  neighbours  of  H faces same direction as H. F sits third to left of A, who faces centre. 
The immediate neighbors of A face opposite to the direction of A. 
 
Q1. Who is sitting third to right of F? 
(a) C 
(b) B 
(c) A 
(d) E 
(e) D 
 
Q2. Who is facing inside? 
(a) AD 
(b) AGH 
(c) AB 
(d) ADC 
(e) None of these 
 
Q3. Who sits opposite to H? 
(a) A 
(b) D 
(c) F 
(d) E 
(e) G 
 
Q4. How many people are siting between C and B, when counted from left of C? 
(a) Two 
(b) Three 
(c) One 
(d) Four 
(e) Five 
 
Q5. Who is sitting exactly between D and C when counted from right of D? 
(a) HE 
(b) CE 
(c) FH 
(d) BF 

Plutus Academy-(129)
(e) DF 
 
Q4: Study the following information carefully and answer the questions:  
(​IBPS PO 2016 Prelims​) 
Ten  persons  are  sitting  in  2 parallel rows containing 5 persons in each row. In 1st row M, N, O, P 
and  Q  are  seated  and  are  facing  south.  In  2nd  row,  U,  V,  X,  Y  and  Z  are  seated  and  are  facing 
north.  Therefore  in  the  given  seating  arrangement,  each  member  seated  in a row faces another 
member  of  the  other  row.  They  like  different  colours  Red,  Orange,  Blue,  Brown,  Black,  White, 
Yellow,  Pink,  Peach,  and  Grey  (not  necessarily  in  same  order).  M  doesn’t  like  brown and P likes 
black.  Y  sits  third  to  the left of U, who likes yellow. M faces immediate neighbour of Y, who likes 
orange.  The  one  who  likes  peach  sits  at  extreme  end.  O  sits  second  to  the  right  of  M.  The  one 
who  likes  red  faces the one who likes pink but M doesn’t like pink. Only one person sits between 
N  and  P.  V  and  Z  are  immediate  neighbours.  Z  does  not  face  M  and  N,  who  doesn’t  like  grey. 
The one who faces U likes white. The one who faces an immediate neighbour of Y likes brown. 
 
Q1. How many persons are seated between N and the one who likes white? 
(a) None 
(b) One 
(c) Two 
(d) Three 
(e) None of these 
 
Q2. Who amongst the following faces P? 
(a) U 
(b) The one who likes pink 
(c) X 
(d) N 
(e) The one who likes grey 
 
Q3. Which of the following is true regarding M? 
(a) N and X are immediate neighbours of M 
(b) M sits at one of the extreme ends of the line. 
(c) M likes black. 
(d) P sits immediate left of M 
(e) None of these 
 
Q4. Who amongst the following pair sits exactly in the middle of the rows? 
(a) M, Z 
(b) P, Y 
(c) None of these 
(d) U, N 
(e) M, V 
 
Q5. V likes which of the following colour? 
(a) Brown 
(b) Pink 
(c) Black 
(d) White 

Plutus Academy-(130)
(e) None of these 
 
Q5:  Study  the  following  information  carefully  and  answer  the  question  given  below:  (​IBPS  PO 
2016 Prelims​) 
Eight  people  viz.  P,  Q,  R,  S,  T,  U,  V  and  W  are  sitting  in  a  straight  line.  They  all  are  facing  north. 
Each  one  of  them has a different age i.e. 8, 11, 15, 17, 21, 25, 27 and 30 year, but not necessarily 
in the same order. The following conditions are given below. 
P  sits  one  of  the  extreme  ends  of  the  row.  There  are  three  person  sits  between  P  and  the  one, 
who is 11 year old. R sits third to the left of the person, who is 17 year old. The person, who is 17 
year  old,  is  not  immediate  neighbour  of  the  person,  who  is  11 year old. R is not 11 year old. P is 
not 17 year old. 
Q  is 15 year old. S is not the youngest person. R is older than P but his age is a not whole square 
number.  There  are  as  many  person  sits  between  P and R is same as the person sits between W 
and the person, who is 30 year old. 
T  sits  immediate  left  of  the  person,  who  is  8  year  old.  Q  sits  third  to  the  left  of  S.  Youngest 
person  sits  extreme  right  end  of  the  row.  Persons,who  sits  left  of  W  are  younger  than  W.  U  is 
older than V.Q does not sit at extreme left end. 
 
Q1. Who among the following person sits extreme right end of the row? 
(a) U 
(b) P 
(c) V 
(d) Q 
(e) None of these 
 
Q2. How many persons are older than S? 
(a) 3 
(b) 5 
(c) 0 
(d) 6 
(e) 4 
 
Q3.  How  many  persons are sits between the person, who is 25 year old and the person, who sits 
immediate right of U? 
(a) None 
(b) Three 
(c) Five 
(d) Six 
(e) One 
 
Q4. Which of the following statement is true according to the given conditions? 
(a) R sits third to the right of P. 
(b) R sits immediate left of S. 
(c) All are true 
(d) T is 17 year old. 
(e) W sits immediate right of the person, who is 15 year old. 
 
Q5. Who among the following persons sits extreme ends of the row? 

Plutus Academy-(131)
(a) W, The person who is 21 year old. 
(b) The person, who is 21 year old and V. 
(c) P and The person, who is 15 year old. 
(d) None of these 
(e) Q and The person, who is 8 year old. 
 
Q  6  Direction  (1-5):  ​Read  the  given  information  carefully  and  answer  the  given  questions. 
Twelve  friends  are  sitting  in  two  parallel  rows  containing  six  people  each  in  such  a  way  that 
there  is  equal  distance  between  adjacent  persons.  In  row 1 A, B, C, D, E and F are seated and all 
of  them  are  facing north and in row 2 P, Q, R, S, T and U, are seated and are facing south, but not 
necessarily  in  the  same  order.  There  are  four  persons  sits  to  the  right  of  R.  R’s  daughter  is 
immediate  neighbour  of  the  one,  who  faces  R.  There  are  one  person  sits  between  R’s  son  and 
R’s  daughter.  Q  is  married  to  F.  R’s  son-in-law  sits  third  to  the  left  of  the  one,  who  faces  the 
person,  who  is  immediate  left  of  A’s  brother.  Two  persons  sit  between  S  and T. R’s brother sits 
second  to  the  right  of  R’s  sister.  S  does  not  sit one of the extreme ends. A is brother-inlaw of B. 
A’s  father  sits  second  to  the  right of A’s brother. E is married to S. only one person sits between 
A’s  brother  and  R’s mother. R’s son-in-law does not sit one of the extreme ends. A’s sister-in-law 
faces  S.  A’s  sister-in-law  sits  third  to  the  left  of  R’s  daughter.  P  faces  E,  who  sits  one  of  the 
extreme left end. B sits third to the right of E. Q sits immediate right of the person, who faces R’s 
wife.  R’s  mother  sits  third  to  the  left  of  the  one,  who  faces  the one, who sits immediate right of 
R’s  son  in-law.  R’s  brother-in-law  sits  fourth  to  the  left  of  R’s  wife.  C  sits  diagonally  opposite to 
the one, who is immediate right of U. A is son-in-law of D.(IBPS PO MAINS 2016) 
 
1. Who sits immediate left of the one, who is sister-in law of A?  
(a) B (b) C’s brother (c)Q’s brother (d)E’s brother-in-law (e)  
 
Q 2. Who is grandmother of B?  
(a)  T  (b)  D’s mother-in-law (c)Person sits one of the extreme left end. (d)E’s mother-in-law (e) All 
are true 
 
3. Who sits second to the left of the one, who faces the person, who sits second to the left of P?  
(a) R’s brother (b) Q (c) None are true (d)A (e) S’s husband  
 
4. Who is the niece of P?  
(a) B (b) T’s son (c)Q’s wife (d)Q’s sister-in-law (e)  
 
Q5.  If  P  related  to  A’s  sister-in-law  in  the  same  way  as  Q  related  to  R’s  daughter.  Which  of  the 
following is U related to, following the same pattern?  
(a)R’s brother (b) C’s brother (c)F (d)A (e) None of these  

Q  7  Direction (1-3): Study the information carefully and answer the questions given below. Eight 
members  A,  B,  C,  D,  E,  F,  G  and  H  sitting  in  a  row  facing  north  But  not  necessary  in  the  same 
order.  No  two  successive  members  are  sitting  together  according  to  alphabetical  order.  For 
Example:  A does not sit with B. similarly B does not sit with C and so on. They also have a hobby 
like  Playing  game,  Watching  TV,  Singing,  Dancing,  Online  surfing,  Chatting,  Acting  and Cooking, 
but  not  necessary  in  the same order. Either A or H sits at the extreme end of the row. A is sitting 
third  to  the  left  of  the  person  whose  hobby  is  online  surfing.  B  sits  second  to  the  right  of  the 
person  who  likes  acting.  C  sits  second  to  the  right  of  F.  G  who  likes  watching  TV  is  sitting 

Plutus Academy-(132)
second  from  the  right  end  of  the  row.  The  person  whose  hobby  is  dancing  is  immediate 
neighbor  of  F  who  likes  Playing  Games.  The  person  whose  hobby  is  acting  is  not  sitting 
adjacent  to  the  person  whose  hobby  is  online  surfing.  C’s  hobby  is  neither  acting  nor  online 
surfing.  D  and  E  do  not  sit  any  extreme  end  of  the  row.  One  of  the  immediate  neighbors  of  H 
likes  Cooking  and  the  one  whose  hobby  is  singing  sits  left  end of the row. Neither H nor D likes 
Acting. ​(SBI PO MAINS 2017) 
Q1.  Which  of  the  following  member  is  sitting  sixth  to  the  right  of  the  second from the right end 
of the row?  
(a) E  
(b) The one whose hobby is acting  
(c) A  
(d) The one whose hobby is cooking  
(e) None of these.  
 
Q2.  A  is  related  to  B  and  F  is  related  to  E  in  the  same  way  how  is  the  member  whose  hobby  is 
dancing related to which of the following?  
(a) The one whose hobby is acting  
(b) G 
(c) The one whose hobby is cooking  
(d) C  
(e) B  
 
Q3. Which of the following combination is not true?  
(a) A-Singing (b) E-Acting (c) G-Watching TV (d) B-Playing games (e) D-Dancing  

Q  8  Direction  (1-2):  Study  the  information  carefully  and  answer  the  questions  given  below. 
There  are  two  square  fields  of  different  size  such  that  the  larger  one  is  surrounding  smaller 
field.  Four  gates  are  there  for  each field in the middle of the sides. Eight people A, B, C, D, E, F, G 
and  H are standing at different gates but not necessary in the same sequence. The persons who 
are  on  the  sides  of  larger  park  facing  center  and  The  persons  who  are  at  side  of  smaller  park 
facing  outside  such  that  inner  sides  persons  and  outer  sides  persons  are  facing  each  other. 
There  is  one  person  standing  between  B  and  D.  C  faces  B. A is to the immediate right of C. G is 
not the immediate neighbor of D. G faces neither D nor F. One person is standing between H and 
F. E is facing the center ​(SBI PO MAINS 2017) 
 
Q1. Which of the following persons are facing to each other?  
(a) BD (b) EB (c) FH (d) DE (e) AH 
 
Q2.  Four  of  the  following  five  are  alike  in  a  certain  way  based  from  a  group  which  one  of  the 
following does not belong to that group?  
(a) EF (b) CH (c) DA (d) FC (e) BH 
 
Q 9 Direction (1-5):​ ​Eight Friends Parkash, Qartar, Ram, Sooraj, Takshay, Vanketesh, Wasir and 
Zubaid out of whom one is a pilot, professor, businessman, doctor, lawyer, banker, cricketer or 
an architect (but not necessarily in the same order) are sitting around a circular table facing the 
center. Sooraj who is a banker sits third to right of Zubaid. The professor and the architect are 
immediate neighbors of each other. Neither the professor nor the architect is an immediate 

Plutus Academy-(133)
neighbor of either Zubaid or Sooraj.Cricketer and the pilot are immediate neighbors of each 
other. Neither Zubaid nor Wasir is a pilot. The one who is a professor sits second to the right of 
Takshay who is a lawyer. Vanketesh who is a cricketer is not an immediate neighbor of the 
banker. Only Ram sits between the professor and the doctor. Parkash sits third to the right of 
the pilot. (IBPS CLERK MAINS 2016) 
 
1. Which of the following pairs represents the immediate neighbors of the doctor?  
a) Professor - Businessman  
b) Pilot - Professor  
c) Cricketer - Businessman  
d) Lawyer - Architect  
e) None of The Above 
  
2. What is the position of the businessman with respect to the pilot? 
a) Third to the left  
b) Second to the left  
c) Immediately to the right  
d) Fourth to the right  
e) Second to the right  
 
3. Who sits third to the right of the professor?  
a) The Banker  
b) Qartar  
c) The Cricketer  
d) The Lawyer  
e) None of The Above  
 
4. Which of the following is true regarding Ram?  
a) He is a doctor  
b) He is an immediate neighbor of the Pilot  
c) Ram sits between Qartar and Takshay  
d) He is an immediate neighbor of the professor  
e) None is true  
 
5. How many people sit between the Banker and Wasir when counted in anti - clockwise 
direction from the banker?  
a) None b) One c) Two d) Three e) None of The Above 

Q 10 Direction (1-5):​ ​ Read the following information carefully to answer the given questions. 
Akram, Bakhtawar, Changez, Dayud, Ekam, Farah, Gulrej and Humayun are seated in straight line 
facing north. 
Changez sits fourth to the left of Gulrej. Dayud sits second to right of Gulrej. Only two people sit 

Plutus Academy-(134)
between Dayud and Akram. Bakhtawar and Farah sits adjacent to each other. 
Humayun is not an immediate neighbor of Dayud.(IBPS CLERK MAINS 2016) 
 
1. Who among the following sits exactly in the middle of the persons who sit fifth from the left 
and the person who sits sixth from the right? 
a) Changez 
b) Humayun 
c) Ekam 
d) Farah 
e) Dayud 
 
2. Who among the following sits third to the right of Changez? 
a) Bakhtawar 
b) Farah 
c) Akram 
d) Ekam 
e) None of The Above 
 
3. Which of the following represents persons seated at the two extreme ends of the line?  
a) Changez and Dayud b) Akram and Bakhtawar c) Bakhtawar and Gulrej  
d) Dayud and Humayun e) None of The Above  
 
4. What is the position of Humayun with respect to Changez?  
a) Third to the Left b) Immediate Right c) Second to Right  
d) Fourth to Left e) Immediate Left 
 
5. How many persons are seated between Akram and Ekam?  
a) One b) Two c) Three d) Four e) None of The Above 
 
Q 11 Direction (1-6):​ ​Study the following information carefully and answer the questions given 
below: Eight friends – E, F, G, H, J, K, L and M are sitting around a circular table facing the 
centre. Each of them is wearing dress of different cartoon characters i.e. Nobita, Osward, 
Popeye, Yogi, Simpon, Minion, Scrooge and Dexter but not necessarily in the same order. F is 
sitting second to the left of K. The one who is wearing dress of Dexter is an immediate 
neighbour of K. There are only three people sits between the the one who is wearing dress of 
Dexter and E. Only one person sits between the one who is wearing dress of Yogi and E. The one 
who is wearing dress of Osward is to the immediate right of the one who is wearing dress of 
Yogi. M is second to the right of K. H is wearing dress of Dexter. G and J are immediate 
neighbours of each other. Neither G nor J is wearing dress of Yogi. The one who is wearing 
dress of Simpon is to the immediate left of F. The one who is wearing dress of Minion is second 
to the right of the one who is wearing dress of Osward. The one who is wearing dress of 
Scrooge is an immediate neighbour of the one who is wearing dress of Yogi. G is second to the 
right of the one who is wearing dress of Nobita. (BOB PRELIMS 2016) 
 

Plutus Academy-(135)
1. Who is sitting second to the right of E?  
(a) The one who wearing dress of Minion  
(b) G  
(c) The one who wearing dress of Yogi  
(d) F  
(e) K  
 
2. Who amongst the following is wearing dress of Scrooge?  
(a) F (b) L (c) M (d) K (e)None of these  
 
3. Four of the following five are alike in a certain way based on the given arrangement and 
hence form a group. Which of the following does not belong to group?  
(a) Nobita – H  
(b) M-Popeye  
(c) J - Yogi  
(d) Simpon – L  
(e) Minion- K  
 
4. What is the position of L with respect to the one who is wearing dress of Dexter?  
(a) Third to the left (b) Second to the right (c) Second to the left  
(d) Third to the right (e) Immediate right  
 
5. Which of the following statements is true accordingly the given sitting arrangement?  
(a) The one who is wearing dress of Minion sits second to the left of the one who is wearing 
dress of Popeye  
(b) E is an immediate neighbour of the one who is wearing dress of Simpon  
(c) H sits exactly between F and the one who is wearing dress of Simpon  
(d) Only four people sit between the one who is wearing dress of Osward and F (e) All of the 
given statements are true  
 
6. Who amongst the following is wearing dress of Nobita?  
(a) E (b) L (c) M (d) K (e)None of these

Q  12 Direction (1-5): ​Study the lowing information to answer the given questions S, T, U, V, W, X, 
Y  and  Z  are  sitting in a straight line equidistant from each other (but not necessarily in the same 
order).  Some  of  them  are  facing  south  while  some  are  facing  north.  (Note  :  Facing  the  same 
direction  means,  if  one  is facing north then the other also faces north and vice-versa. Facing the 
opposite  directions  means,  if  one  is  facing  north  then  the  other  faces  south  and  vice-versa)  S 
faces  north.  Only  two people sit to the right of S. T sits third to the left of S. Only one person sits 
between  T  and  X.  X  sits  to  the  immediate  right  of  W.  Only  one  person  sits  between  W  and  Z. 
Both  the  immediate  neighbors  of  T  face  the  same  direction.  U  sits  third to the left of X. T faces 
the  opposite  direction  as  S.  Y  does  not  sit  at  any  of  the  extremes  ends  of  the  line.  V  faces  the 
same direction as W. Both Y and U face the opposite direction of Z. ​(BOB 2016) 
 
1. How many persons in the given arrangement are facing North?  
(a) More than four (b) Four (c) One (d) Three (e) Two  
 

Plutus Academy-(136)
2.  Four  of  the  following  five  are  alike  in  a  certain  way,  and  so  form  a  group.  Which  of  the 
following does not belong to the group?  
(a) W, X (b) Z, Y (c) T, S (d) T, Y (e) V, U  
 
3. What is the position of X with respect to Z?  
(a) Second to the left (b) Third to the right (c) Third to the left  
(d) Fifth to the right (e) Second to the right  
 
4. Who amongst the following sits exactly between Z and W?  
(a) T (b) Y (c) X (d) W (e) U  
 
5. Who is sitting 2nd to the right of T?  
(a) Z (b)V (c)X (d)W (e)None of these.  
 
6. What is the position of W with respect to S?  
(a) Second to the left (b) Third to the right (c) Third to the left  
(d) Fifth to the right (e) Second to the right

Q  13  Direction  (1-5): ​Study the following information and answer the given questions: A, B, C, D, 


E,  F,  G  &  H are eight friends and sitting around a circular table but not necessarily in same order. 
Some  of  them  are  facing  inside  and  some  of  them  are  facing  outside.  A  sits  third to right of H. 
There  is  two  people  sits  between  H  and  B.  C  sits  second  to  left  of  B.  There  is three people sits 
between  B  and  E.  D  is  second  to  left  of  F,  who  is  not  immediate  neighbor  of  A.  Immediate 
neighbours  of  H  faces  same  direction  as  H.  F  sits  third  to  left  of  A,  who  faces  centre.  The 
immediate neighbors of A face opposite to the direction of A. ​(IBPS PO Prelims 2017) 
 
1. Who is sitting third to right of F?  
(a) C (b) B (c) A (d) E (e) D  
 
2. Who is facing inside?  
(a) AD (b) AGH (c) AB (d) ADC (e) None of these  
 
3. Who sits opposite to H?  
(a) A (b) D (c) F (d) E (e) G  
 
4. How many people are siting between C and B, when counted from left of C?  
(a) Two (b) Three (c) One (d) Four (e) Five  
 
5. Who is sitting exactly between D and C when counted from right of D?  
(a) HE (b) CE (c) FH (d) BF (e) DF  
 
Q  14  Direction  (1-5):  Study  the  following  information  carefully  and  answer  the  questions:  Ten 
persons  are  sitting  in  2  parallel rows containing 5 persons in each row. In 1st row M, N, O, P and 
Q  are  seated  and  are  facing  south.  In  2nd  row,  U,  V,  X,  Y  and  Z are seated and are facing north. 
Therefore  in  the  given  seating  arrangement,  each  member  seated  in  a  row  faces  another 
member  of  the  other  row.  They  like  different  colours  Red,  Orange,  Blue,  Brown,  Black,  White, 
Yellow,  Pink,  Peach,  and  Grey  (not  necessarily  in  same  order).  M  doesn’t  like  brown and P likes 
black.  Y  sits  third  to  the left of U, who likes yellow. M faces immediate neighbour of Y, who likes 

Plutus Academy-(137)
orange.  The  one  who  likes  peach  sits  at  extreme  end.  O  sits  second  to  the  right  of  M.  The  one 
who  likes  red  faces the one who likes pink but M doesn’t like pink. Only one person sits between 
N  and  P.  V  and  Z  are  immediate  neighbours.  Z  does  not  face  M  and  N,  who  doesn’t  like  grey. 
The  one  who  faces  U  likes  white.  The  one  who  faces  an immediate neighbour of Y likes brown. 
(IBPS PO Prelims 2017) 
 
Q1. How many persons are seated between N and the one who likes white?  
(a) None (b) One (c) Two (d) Three (e) None of these  
 
Q2. Who amongst the following faces P?  
(a) U (b) The one who likes pink (c) X (d) N (e) The one who likes grey  
 
Q3. Which of the following is true regarding M?  
(a) N and X are immediate neighbours of M  
(b) M sits at one of the extreme ends of the line.  
(c) M likes black.  
(d) P sits immediate left of M  
(e) None of these  
 
Q4. Who amongst the following pair sits exactly in the middle of the rows?  
(a) M, Z (b) P, Y (c) None of these (d) U, N (e) M, V  
 
Q5. V likes which of the following colour?  
(a) Brown (b) Pink (c) Black (d) White (e) None of these  
 
 
SOLUTIONS: 
 

1.  
1.​(a) Y 
2.(a) Third to the right 
3.(d) Four 
4.(e) X – Dwarka 
5.(c) The person from Dwarka is sitting exactly between the Karol Bagh and the Saket. 
 
 

Plutus Academy-(138)
 
 
 

 
1.​(b) Four 
2.(d) T, Y 
3.(b) Third to the right 
4.(a) T 
5.(a) Z 
 

 
1. (d) E 
2. (c) AB 
3. (b) D 
4. (c) One 
5. (d) BF 
 
4.  
 

 
 
1. (c) Two 

Plutus Academy-(139)
2. (e) The one who likes grey 
3. (d) P sits immediate left of M 
4. (e) M, V 
5. (b) Pink 
 
5.  
 

 
 
 
1. (c) V 
2. (d) 6 
3. (b) Three 
4. (c) All are true 
5. (b) The person, who is 21 year old and V. 
 
6.  

 
1. c) 
2. e) 
3. e) 
4.d) 
5. b) 
 
7.  

 
1. D 
2. B 
3. D 
 

Plutus Academy-(140)
8.  

 
1. D 
2. E 
 
9. 
1. c) Cricketer - Businessman 
2. a) Third to the left  
3. c) The Cricketer  
4. d) He is an immediate neighbor of the professor  
5. c) Two 
 
10.  
1. d) Farah 
2. c) Akram 
3. d) Dayud and Humayun  
4. e) Immediate Left 
5. a) One 
 
 
11 

1. (b) 2. (d) 3. (c) 4. (b) 5. (a) 6. (a)

Plutus Academy-(141)
12

1. (b)
2. (d)
3. (b)
4. (a)
5. (b)
6. (a)

13 

 
1. (d) E
2. (c) AB
3. (b) D
4. (c) One
5. (d) BF

14 

Plutus Academy-(142)
1. (c) Two
2. (e) The one who likes grey
3. (d) P sits immediate left of M
4. (e) M, V
5. (b) Pink
 
 
 
 
 
 
 
 
 
 
 
 
 
 
 
 
 
 
 
 
 
 
 
 
 
 
 
 
 
 
 
 
 
 
 
 
 
 

Plutus Academy-(143)
Classification Test 
This type of Puzzle question consist a group of items. These item can be anything (like humans, 
cards, vehicle, place etc). Each item will have many characteristics regarding which we have 
many clues and remarks given in the question. We have to identify the each characteristic for 
each member of the group after examining the various clues. After proper arrangement, we will 
get a group whose data is organized. 
To solve this type of question we need to examine each clue carefully and maintain all the data 
related to given group in tabular form i.e. in Rows and column format. 
Eamplex 1. In a Hockey session, Team A defeated Team B twice. Team C defeated Team A 
twice,Team B defeated Team C twice,Team A defeated Team D twice & Team C defeated 
Team D twice.which Team has lost most number of times.   
Solution: 
Won  Defeat 
Team A  Team B 
Team A  Team B 
Team C  Team A 
Team C  Team A  
Team B  Team C 
Team B  Team C 
Team A  Team D 
Team A  Team D 
Team C  Team D 
Team C  Team D 
Clearly Team D loses most of the matches. 
Example 2. Study the following information and answer the questions given below it. 
(i) komal ,shivani and Reena are Intelligent. 
(ii)Komal,Rani and Seeta are hard working. 
(iii)Rani,Reena and Seeta are honest. 
(iv)Komal,shivani and Seeta are ambitious. 
Q1.Which of the following persons is neither hard working nor ambitious? 
a)Komal b)shivani c)Reena d)Rani 
Q2.Which of the following persons is neither honest nor hard working but is ambitious? 
a)Komal b)shivani c)Reena d)Rani 
Solution: Make a Tabular form after examined various clues. 
Student  Tyes of Student 
Name  Intelligent  Hardworking  Honest  Ambitiou

Komal  ✔  ✔    ✔ 
Shivani  ✔      ✔ 
Reena  ✔    ✔   
Rani    ✔  ✔   
Seeta    ✔  ✔  ✔ 

Plutus Academy-(144)
 
Solution 1: Clearly Reena is a person who is neither hard working nor ambitious. 
Solution 2: Clearly Shivani is a person who is neither honest nor hard working but is ambitious. 
 
Example 3. Read the following information carefully and answer the questions given below: 
(i) There are five types of cards viz. A,B,C,D and E.There are three cards of each type.These 
are to be inserted in envelopes of three colours-Red,Yellow and Brown.There are five 
envelopes of each colour.(SBI PO 2000). 
(ii) B,D and E type cards are inserted in red envelopes .A,B and C type cards are to be inserted 
in yellow envelopes and C,D and E type cards are to be inserted in brown envelopes. 
(iii)Two cards each of B and D type are inserted in red envelopes.  
 
Sol: Make a tabular form after examined various clues. 
Envelopes Color/Types  Types of Cards 
Red  B  D  E  B  D 
Yellow  A  B  C  A  A 
Brown  C  C  E  C  E 
 
Q Which of the following combinations of types of cards and the number of cards and colour of 
envelope is definitely correct? 
(a)A-2,B-2,C-1:yellow 
(b)C-2,D-1,E-2:Brown 
(c)C-1,D-2,E-2:Brown 
(d)B-2,D-2,A-1:Red 
(e)none of these 
Sol:(b) is correct. 
 
Q Which of the following combinations of colour of the envelope and the number of cards is 
definitely correct in respect of E-type cards? 
(a)Red-1:yellow-2 
(b)Yellow-1:Brown-2 
(c)Red-2:Brown-1 
(d)Red-2:yellow-1 
(e)none of these 
Sol:(e) as the correct combination in respect of E-type is Red-1:Brown-2 
 
Q Which of the following combinations of the type of cards and the number of cards is 
definitely correct in respect of yellow coloured envelopes? 
(a)A-1,E-1,D-2 
(b)A-2,B-1,C-2 
(c)A-3,B-1,C-1 
(d)B-1,C-2,D-2 
(e)none of these 

Plutus Academy-(145)
Sol-(c) 
 
Ex 4.Study the following information carefully and answer the questions given below:(Bank PO 
2004). 
(i)Eight persons A,B,C,D,E,F,G and H work in three different companies X,Y,Z. 
(ii)There are two ladies who work in different companies and their specialization is also 
different. 
(iii) Two of them have specialization in finance,another two have specialization in human 
Resources,two have specialization in Marketing, one's engineer and one of them is specialist 
in computer.   
(iv) D is a specialist in Human Resource working in company X while her friend G is a Finance 
specialist and works in company Z. 
(v)H is a Human REsource specialist who work with Marketing specialist B but does not work 
in company Y. 
(vi)The two persons with same specialization do not work together. 
(vii)Marketing specialist F works in company Y and his friend A who is finance specialist works 
in Company X with only one other specialist. 
(viii)In no company more than three persons work. 
(ix)C is an engineer and his sister works in Company Z. 
(x)No lady is an Engineer or Computer Specialist. 
 
Q1.In which company does C work? 
X b) Y  c) Z d)Either Y or Z e) None of these 
Q2.In which two companies do Human Resource specialists work? 
a)X and Y b)Y and Z c)X and Z  d)Data Inadequate e)None of these 
Q3.The two ladies are: 
a)B and D b) D and G c)D and H d)Either a or b e)Data inadequate 
Q4.Who is Computer specialist? 
a)C b)E c)H d)Data inadequate e)None of these 
Solution:-There are 2 Female and 5 Males. 
persons= 2-F.S, 2-M.S, 2-M.S, 1-Eng., 1-computer 
Persons  Specialist  Company  M/F 
A  Finance Specialist  X  M 
B  Marketing Specialist  Z  M/F 
C  Engineer  Y  M 
D  H R Management  X  F 
E  Computer  Y  M 
F  Marketing Specialist  Y  M 
G  Finance Specialist  Z  M/F 
H  H R Specialist  Z  M 
Sol1-C works in Company Y. 
Sol2.Human resource specialist work in X and Z company. 
Sol3.The two ladies are:D and Either B or G. 

Plutus Academy-(146)
Sol4.E is computer specialist. 
 
Ex5. Study the following information carefully and answer the questions given below: 
(i) A,B,C,D,E and F are six members of a group in which there are three female members. 
Females work in three departments -Accounts, Administration and Personnel and sit on three 
different floors-Ist, IInd and IIIrd.Persons working in the same department are not on the same 
floor.On each floor two person work. 
(ii)No two ladies work in the same department or on the same floor.B and E work in the same 
department but not in personnel work in Administration and A are on the Ist and IIIrd floors 
respectively and work in the same department. D ,a lady does not work on the IInd Floor.C, a 
man, works on Ist floor. (Bank PO 2001) 
 
Q1)Which of the following groups of persons are females? 
a)ADF b)CDF c)DEF d)Data inadequate e)None of these 
Q2)Which of the following pairs work on IInd floor? 
a)AE b)BD c)BF d)CF e)None of these 
Q3)Which of the following pairs of persons work in administration? 
a)BD b)CD c)CE d)Data Inadequate e)None of these 
Q4)F works in which department? 
a)Administration b)Accounts c)Personnel d)Accounts or Personnel e)None of 
these 
Solution: 
III  A  Accounts  M  D  Admin  F 
II  B  Accounts  M  F  Personnel  F 
I  C  Personnel/Adm M  E  Accounts  F 
in 
1) D,F and E are female members. 
2) B and F worked on IInd Floor. 
3) Data is adequate. 
4) F works in personnel. 
 
 
 
 
 
 
 
 
 
 
 
 

Plutus Academy-(147)
Comparison Type Puzzle 
 
In this type of Questions,we will do comparisons based on certain quality between the objects. 
Based on Comparisons we will arrange them in some order(Ascending,Descending etc.) and 
answers the questions accordingly.  
 
Study the following information and answer the following questions:- 
Ex1-A Carpenter have five wooden table J,K,L,M and N,each having a different weight. 
1. J weighs twice as much as K. 
2. K weighs four and a half times as much as L. 
3. L weighs half as much as M. 
4. M weighs half as much as N. 
5. N weighs less than J but more than L.  
Q1 Which of the following is the lightest in weight? 
A)J B)K C)L D)M E)N 
 
Q2.N is lighter in weight than which of the other two articles? 
a)J,K b) M,L c) J,L d)M,K e)K,N 
 
Q3.E is heavier than which of the following two articles? 
A)M,K B)M,L C)J,L D)J,K E)none of these 
 
Q4.Which of the following articles is the heaviest in weight? 
A)J B)K C)L D)M E)N 
 
Q5.Which of the following represents the descending order of weights of the articles? 
A)J,K,N,M,L 
B)K,M,NJ,L 
C)N,L,M,J,K 
D)L,J,D,K,N 
E)J,K,M,N,L 
 
Q6.Which of the above given statements is not necessary to determine the correct order of 
articles according to their weights? 
A)1 B)2 C)3 D)4 E)5 
Answer- 
J->K->N->M->L 
1)C 2)A 3)B 4)A 5)A 6)E 
 
Ex-2 There are five friends-P , Q , R , S and T . P is shorter than Q but taller than T. R is 
tallest.S is a little shorter than Q and little taller than P. 
Q1. Who is the shortest? 

Plutus Academy-(148)
a) T b) P c) S d)Q e) None of these 
Q2. If they stand in order of their heights,who will be in the middle? 
a) Q b) T c) P d) S e) None of these 
Q3. If they stand in the order of increasing heights,who will be the second? 
a) S b) P c) T d) Q e) None of these 
Q4. Who is second tallest? 
a) S b) P c) T d) Q e) None of these 
Answer- R->Q->S->P->T 
1) T 2) S 3) P 4) Q 
 
Q. A Toys Manufacturing Company launches nine different soft toys-Teddy, Panda, Tweety, 
Pikachu, Lion, Monkey, Mickey Mouse, Doraemon, Doll. in delhi. The Soft toys were launched 
one after the other over the period of 6 months in 2010. The order in which the products were 
launched is as follows:- 
1. Monkey is launched before both Doraemon and Teddy. 
2. Teddy gets launched after doll. 
3. Pikachu gets launched before monkey but after Panda. 
4. Tweety gets launched after monkey. 
5. Panda gets launched before Mickey Mouse. 
Q1.Which one of the following could be true? 
1)Y is the second product to be launched 
2)R is the third product to be launched  
3)Q is the fourth product to be launched 
4)S is the fifth product to be launched 
Answer-(4) 
 
Previous Exam Question- 
Study the following information and answer the following questions:- 
Each of five friends A,B,C,D and E scored different marks in an examination.Maximum marks of 
the examination are 100.B scored more than E but less than C.Only two people scored more 
than A.E did not score the minimum marks.The one who scored second highest marks scored 
87%.E scored 72% marks. (IDBI Bank 2012)  
Q1.How many people scored more than E in the examination? 
a)Noneb)One c)Two d)Three e)Cannot be determined. 
 
Q2.Who amongst the following possibly scored 81% marks? 
a)C b)D c)B d)A e)Either C or B 
 
Q3.Which of the following percentages may correctly represent C’s percentage in the exam? 
a)65% b)74% c)87% d)83% e)89% 
 
Answer-C->B->A->E->D 
1) D, 2) D, 3)E 

Plutus Academy-(149)
Family Based Problems 
 
In this type of Questions, some clues are given regarding members of a family and their 
features(for e.g. their like or dislike,profession,qualities,dresses,preferences,belonging city 
name etc.).The candidate is required to examine all clues carefully and answer the questions 
based on clues. 
To solve these questions we will use tabular form to maintain all data regarding Members of a 
Family 
 
Read the following information carefully and answer the questions given below- 
A. In a Family of Six persons,there are people from three generations.Each person has 
separate profession and also like different colours.There are two couple in the family. 
 
B. Rohan is a C.A and his wife neither is a doctor nor likes green colour. 
C. Engineer likes red colour and his wife is a teacher. 
D. Mohini is Mother-In-Law of Sunita and she likes orange colour. 
E. Vinod is grandfather of Tanmay and Tanmay,who is a principal, likes black colour. 
F. Nanu is GrandDaughter of Mohini and she likes blue colour.Nanu’s mother likes white 
colour. (SBI PO 1995) 
Q1.Who is an Engineer? 
1)Nanu 2)Mohini 3)Sunita 4)Cannot be determined 5)None of these 
Answer-(5)Vinod 
 
Q2.What is the profession of Sunita? 
1)Engineer 2)Doctor 3)Teacher 4)Cannot be determined 5)None of these 
Answer-(4) 
 
Q3.Which of the following is the correct pair of two couples? 
1)Mohini-Vinod and Rohan-Sunita 
2)Vinod-Mohini and Rohan-Nanu 
3)Rohan-Sunita and Tanmay-Nanu 
4)Cannot be determined 
5)None of these 
Answer-(1) 
 
Q4.How many ladies are there in the family? 
1)Two  2)Three 3)Four 4)Cannot be determined 5)None of these 
Answer-(2) 
 
Q5.Which colour is liked by CA? 
1)Green 2)White 3)Either White or Green 4)cannot be determined
5)None of these 

Plutus Academy-(150)
 
Answer-(1)Green 
Solution- 
 

 
 
 
Selection based Puzzle 
 
In this type of questions,some clues are given regarding the selection of items of the group.we 
have to make selection of items after examining the such clues and answer the questions 
accordingly. 
Eight student A,B,C,D,E,F,G and H are planning to enjoy car racing.There are only two cars and 
following are the conditions. 
1) One car can accommodate maximum five and minimum four students. 
2) A will sit in the same car in which D is sitting but H is not in the same car. 
3) B and C can’t sit in the same car in which D is sitting. 
4) F will sit in the car of four people only along with A and E but certainly not with G. 
(Bank PO 1996) 
Q1.If H and G are sitting in the same car, who are other two students sitting in the same car? 
1)B and C 2)C and D 3)E and B 4)None of these 5)None of these 
 
Q2.If E and A are sitting in the same car,which of the following statements is true? 
1)Five students are sitting in the same car’ 
2)B is sitting in the same car 
3)F is not sitting in the same car 
4)G is not sitting in the same car 
5)None of these 
 
Q3.Which of the following statements is superfluous for the above sitting arrangements? 
1)only (i) 2)only(ii) 3)only(iii) 4)only(iv) 5)none of these 
 

Plutus Academy-(151)
Solution= 
Car I-A,D,F,E 
Car II-H,B,C,G 
 
Q1-(1)BC Q2.(4)G is not sitting in the same car Q3.(1)only(i) 
 
Sequential order of things 
In this type of questions,some clues are given regarding the occurrence of the events done by 
objects.We have to Examine the clues carefully to frame the right sequence of events. After that 
based on this sequence we will give answer to the given questions.  
 
 
Previous Exam Questions- 
Study the following information carefully and answer the questions given below- 
 
Six plays A,B,C,D,E and F are to be staged on six day of the week starting from monday and 
ending on saturday.Play C is staged on a tuesday.Plays A,F and B are staged one after the 
other in the same order.Play D is not staged on a Monday or a Wednesday. 
(PNB MT 2010) 
 
Q1.How many plays are staged after play A is staged. 
a)One b)Two c)Three d)Four e)Cannot be determined 
 
Q2.Four of the following five form a group based on the days that they are staged.Which one of 
them does not belong to that group? 
a)EC b)FD c)CA d)AF e)BD 
 
Q3.Which play is staged immediately before the day play E is staged? 
a)B b)A c)F d)D e)there is no such play E is staged 
 
Q4.If play D was staged on a Monday,Which of the following plays would definitely be staged on 
a Saturday(all the other conditions given above remain the same)? 
a)B b)E c)D d)F e)Cannot be determined 
 
Q5.Which play is Staged on thursday? 
a)B b)E c)D d)F e)cannot be determined 
 
Answer- 
Pattern-E->C->A->F->B->D 
1)C 2)B 3)E 4)D 5)D 
 
 

Plutus Academy-(152)
Study the following information carefully and answer the questions given below:-(SBI 
Associates PO 2014) 
Each of the six people,U,V,W,X,Y and Z stayed for a different number of days in india. X stayed 
for less number of days than only V. Y stayed for more number of days than only two people. U 
did not stay less than Y. W did not stay for the minimum number of days. The one who stayed 
for the second lowest number of days stayed for eight days. U stayed for 17 days in india. 
 
Q6.The one who stayed for the maximum number of days stayed 15 more than W.Which of the 
following is possibly the number of days for which it stayed in india? 
a)5 b)32 c)23 d)20 e)17 
 
Q7.Which of the following is true with respect to Z as per the given information? 
a)The possible number of days for which z stayed in india is 14 days. 
b)Z stayed for the minimum number of days 
c)None of the given options is true 
d)Only two people stayed for more number of days than Z. 
e)Z definitely stayed for more number of days than Y. 
 
Q8.Which of the following may be the possible number of days for which Y stayed in india? 
a)11 b)8 c)19 d)5 e)22 
 
Answer- 
Pattern-V->X->U->Y->W->Z 
6)C 7)B 8)A 
 
Study the following information carefully and answer the questions given below. 
A building has 7 floors numbered 1 to 7, in such a way that ground floor is numbered one, the 
floor above it, number 2 and so on such that the topmost floor is numbered 7. One out of the 7 
persons,, viz. P, Q,R , S, T,U and V Lives on each floor, but not necessarily in the same order. 
each one of them is travelling to different places,viz., Bangalore, Chennai, Delhi, Jaipur, 
Kolkata ,Mumbai and Patna, but not necessarily in the same order.  
3 persons live on the floors above the floor of P. there is only one person between P 
and the person travelling to Bangalore. U lives immediately below the person who is travelling 
to Mumbai.The person who is travelling to Mumbai lives on an even numbered floor.P live 
below the person travelling to Mumbai. Two persons are leaving between the persons who are 
travelling to Bangalore and Patna respectively.T live immediately above R.T is not travelling to 
Patna. Two persons Live between Q and the person travelling to Kolkata. The person who is 
travelling to Delhi is not living immediately above or below the floor of Q. the person who is 
travelling to Kolkata lives below Q. S does not live immediately above or below the floor of P. V 
is not traveling to Chennai. The person who is travelling to Delhi does not live on the ground 
floor. 
(IBPS RRB Officer scale 1 CWE 2014). 
 

Plutus Academy-(153)
Q1)Who among the following lives on the topmost floor? 
1)U 2)Q 3)V 4)T 5)S 
Q2)Four of the following five are alike in a certain way and hence they form a group based on 
the given arrangement. which one of the following does not belong to that group? 
1)R 2)S 3)V 4)U 5)T 
Q3) who among the following travel to Delhi? 
1)T 2)U 3)S 4)R 5)P 
Q4) how many persons Live between the person who is travelling to Mumbai and S?  
1)three2)four 3)One 4)Two 5)five 
Q5)Who among the following does live on the floor immediately above the floor of T?  
1)S 2)Q 3)U 4)P 5)V 
 
 
 
 
 
 
 
 
 
 
 
 
 
 
 
 
 
 
 
 
 
 
 
 
 
 
 
 
 
 
 
 

Plutus Academy-(154)
MACHINE INPUT 
In  this  type  of  questions  we  have  some  sequence  of  words  and  numbers  which  is  known  as 
Input.After  that  some  rearrangement  will  be  done  in  various  steps  on  this  given  input  by 
following  some  particular  order.At  the  end  we  get  the  sequenced  output  in  a  particular  order. 
This whole process is known as sequential output tracing or Machine Input Output. 
 
Input------------------->Machine Processing or Sequential output tracing---------------------->Output 
(doing some arrangement after following some particular pattern/Order) 
  
To solve these questions we need to identify two things. 
1)Direction: This defines from which direction is inputting.   
2)Pattern/Order: This defines the type of pattern machine is using to arrange the data or to trace 
sequential output. Pattern can be alphabetical order,reverse order alphabets,ascending 
order,descending order….etc. 
 
After that we give solution to given question based on these two things i.e. Direction and 
Pattern/order. 
 
TYPES OF INPUT OUTPUT 
1. Arrangement Type 
2. Shifting Type 
3. Combined Type 
4. Mathematical Type 
 
ARRANGEMENT TYPE: 
In this type, the position of the first element remains same  
Ex1. 85 16 36 04 19 97 63 09 (Bank PO 1995) 
StepI  97 85 16 36 04 19 63 09 
StepII 97 85 63 16 36 04 19 09 
Step III  97 85 63 36 16 04 19 09 
Step IV  97 85 63 36 19 16 04 09  
Step V 97 85 63 36 19 16 09 04 
 
Here we can find that sequence is arranging in descending order by altering only one position in 
a single step. 
so,start marking the sequence using 123....,If some number/word is already in a sequence or at 
the legal position then mark that as ​AutoFill(AF) ​by following the sequence no.   
Direction-Left to Right 
pattern-Number(Descending order) and Only one value at a time 
Input- 85 16 36 04 19 97 63 09 
 
AF(1.1) AF(4.4) 3 AF(5.5) 4 1 2 5 

Plutus Academy-(155)
 
As per the rules followed in above Steps.Find out the Answer for the following questions. 
 
 
Ques1. 25 08 35 11 88 67 23  find step  
AF(3.1) AF(5.1) 3 5 1 2 4 
 
so the ans is 88 67 35 25 23 11 08 
______________________________________________________________________________ 
 
Ques2. 09 25 16 30 32 19 17 06 find step III 
- 3 - 2 1 - - -   
 
so the ans is  32 30 25 09 16 19 17 06 
 
 
Ques3.  16 09 25 27 06 05 Find last step 
AF(2.1) AF(2.2) 2 1 AF(2.3)  AF(2.4) 
 
so the ans is  27 25 16 09 06 05(No of steps required are 2) 
______________________________________________________________________________ 
 
Ques 4. 03 31 43 22 11 09  What will be 
last step-Step V  
AF(5.1) 2 1 3 4 5 
so the ans is  43 31 22 11 09 03 
 
______________________________________________________________________________ 
 
Ques 5.  92 86 71 69 15 19 06 63 58 This is given 
the Step IV.Find ​What will be the Input.  
Ans:=Answer cannot be determined as we can have multiple way to find Input. 
 
______________________________________________________________________________ 
 
Ex2)Input--- Go for to Though By easy To Access at (SBI PO 1997) 
I Access Go for to Though By easy To at 
II Access at Go for to Though By easy To 
III Access at By Go for to Though easy To 
IV Access at By easy Go for to Though To 
V Access at By easy for Go to Though To 
VI Access at By easy for Go Though to To 
VII Access at By easy for Go Though To to 

Plutus Academy-(156)
Sol: ​Direction-Left to Right 
Pattern-words(alphabetical order) and only one word at a time 
Go for to Though By easy To Access at 
AF(5.1) 5AF (7.1) 6 3 4 7 1 2 
 
Ans Access at By easy for Go Though To  to 
 
Ques 6: story For around on was He at Write step IV 
- 3 1 - - 4 2 
Write data from 1-4 as we have marked and remaining data as it is in the given input. 
 
Ans- around  at For He story on was 
______________________________________________________________________________ 
Ques 7: for the given sequence which step will be---------an and every for peer to 
every and peer to an for  
 
AF(2.1) 2 AF(3.1) AF(3.2) 1 3  
Ans -Step III  
______________________________________________________________________________ 
Ques 8:Find last but one/Second Last   
Together over series on feast the so  
AF(6.1) 3 4 2 1 6 5 
Ans-Step V is the second last Step i.e.  
feast  on  over series  so  together the 
______________________________________________________________________________ 
Ques 9:Which of the following would definitely be the input when Step-II is given. 
Step-II and Do pet to on that 
Options:- 
A) Do  on  pet to  and  that 
B) Do  pet  to  and  that on 
C)  Do and pet to   on  that 
1.1 1 2.1 3.1 2 3 
D) Cannot be determined 
E) None of these 
Do  on  pet to  and  that 
Af(2.1) - 2 - 1 - 
So the Step II is- and Do pet on to that (FALSE-as doesn’t match with given step) 
Do  pet  to  and  that on 
1.1 1.2 - 1 - 2 
So the step II is- and Do pet on to that (FALSE-as doesn’t match with given step) 
Do and pet to   on  that 
1.1 1 2.1 3.1 2 3 
Answer is (e) None of these 

Plutus Academy-(157)
Q10. Which step number will be the last step of the below input? 
Over  Go  For  through  at  one 
Answer- 4.1(af) 3 2 4.2(af) 1 4 
So the answer is 4. 
______________________________________________________________________________ 
 
Ex 3.Input 63 lovely 18 star 45 92 game honest (Bank PO) 
I 92 63 lovely 18 star 45 game honest 
II 92 game 63 lovely 18 star 45 honest 
III 92 game 63 45 honest lovely 18 star  
IV 92 game 63 45 honest 18 lovely star 
Solution----- 
Direction=Left to Right 
Pattern=Number(Descending)+Words(Alphabetical order) and two values are placing in a single 
step. 
 
Seq. 63 lovely 18 star 45 92 game honest  
Q11. find Last Step  
Year month  23 date   15   19 
Seq. 5.1(af) 4 1 2 5 3 
Last step is 5. 
 
Q12.Find step 3rd. 
Lotus like 45 26 Lily  
Seq. _ 2 1 3 _ 
III- 45 like 26 lotus lily 
 
Ex quick fire 15 28 39 war 19 yellow 
I Yellow quick fire 15 28 39 war 19 
II Yellow 15 quick fire 28 39 war 19 
III Yellow 15 war quick fire 28 39 19 
IV Yellow 15 war 19 quick fire 28 39 
V Yellow 15 war 19 quick 28 fire 39 
 
Pattern-words(Reverse)+number(ascending order) and one values is placing in a single step. 
Direction-Left to Right 
 
Q13.Find Last but one step(i.e. Second Last step) 
Step II zebra  12  bank  carriage 46 31 29 dusk  
1 2 af(6.1) 5 af(6.2) 6 4 3  
 
1 and 2 position are already fixed 
 

Plutus Academy-(158)
Q14.How many steps will be require to complete the rearrangement? 
 
Age   die 72 53 35 hold goal 26 
af(6.1) 5 af(6.2) 6 4 1 3 2 
 
Answer-6 
 
Q15.Find Last step 
win 12 92 for 81 always 36 home 
0.1 0.2 af(5.1) 3 4 5 2 1 
Sol-Last step =win 12 home 36 for 81 always 92 
 
Q16.Step III-Train  23 star 61 32 fall hard 53 (Find Input) 
Sol-Cannot be determined as term can be rearranged in several way. 
 
Ex- 48 245 182 26 99 542 378 297 
I 542 48 245 182 26 99 378 297  
II 542 26 48 245 182 99 378 297 
III 542 26 378 48 245 182 99 297  
IV 542 26 378 48 297 245 182 99  
V 542 26 378 48 297 99 245 182 (SBI PO 2000) 
Pattern-number(highest to lowest)+number(Lowest to highest) and one value at a time. 
Direction-Left to Right 
 
Q17. Step-II 765 42 183 289 542 65 110 350 Find Step IV? 
1 2 3 4 
Answer- 765  42 542 65  183  289  110 350 
 
Q18. show the output of last step? 
39 88 162 450 386 72
29  
Seq 3.1(af) af(5.1) 4 1 3 5
2  
Output-450 29 386 39 162 72 88  
------------------------------------------------------------------------------------------------------------------------------------------------ 
Ex- worst pink 48 lovely 17 98 33 camel 
I 98 pink 48 lovely 17 33 camel worst 
II 98 48 lovely 17 33 camel pink worst 
III 98 48 33 17 camel lovely pink worst 
 
Pattern- Number(Descending)+Words(Reverse alphabetical order) two values at a time. 
Direction- Number(Left to right) +Words(right to Left) 
-------------> ←------------- 

Plutus Academy-(159)
Q19. Find Last Step 
17 gate gain groom 45 ghost 28 54  
3.1(af) A.3 A.3.1(af) A.1 2 A.2 3 1 
Last step is= 54 45 28 17 gain gate ghost  groom   
 
Q20. How many steps required to get an output. 
37  hold 89 joker 67 kite fish 45 
af(3.1) A.3 1 A.2 2 A.1 af(A.3.1) 3 
 
So total steps required=3. 
 
Directions (21-23): ​Study the given information carefully and answer the given questions. An 
input-output is given in different steps. Some mathematical operations are done in each step. 
No mathematical operation is repeated in next step. (SBI PO MAINS 2017) 

 
As per the rules followed in the steps given above, find out in each of the following questions 
the appropriate step for the given input.  

 
Q21. Find the addition of the two numbers obtained in step III?  
(a) 1.5 (b) 3 (c) 7 (d) 3.5 (e) None of these  
 
Q22. Find the difference between sum of numbers which obtained in 1st step and sum of 
numbers obtained in all other steps?  
(a) 232 (b) 185 (c) 188 (d) 183.5 (e) None of these  
 
Q23. Find the multiplication of the numbers obtained in step II?  

Plutus Academy-(160)
(a) 426 (b) 462 (c) 188 (d) 98 (e) None of these 
 
21. Sol: 
For step-I, both the numbers of 1st block is written as, Ist number of block-1 of the Input is 
multiplied with 2nd no. of block-4 of the Input same as 2nd number of block-1 is multiplied with 
1st number of block-4. This process is same for Block-2 and Block-3 in step-1.  
For step-II, All 1st digit of each block is added and that sum is written in 1st block and all 2nd 
digit of each block is added and that sum is written as 2nd block.  
For step-III, Half of the addition of 1st and 2nd digit of each block.  
For Step- IV, Subtraction of both numbers of Step-3.  
So, INPUT: 42 51 29 32 71 14  
Step-1: 86 57 89  
Step-2: ….21….22….  
Step-3: ….1.5….2….  
Step-4:…...0.5……  

 
22. Ans. (b) 
23. Ans. (b) 
 
 
 
 
 
 
 
 
 

Plutus Academy-(161)
EXERCISE    
Q1.​Study the following information carefully and answer the given questions. 
Input:​ daily 79 do diverse 57 14 dear 86 63 domain 42 dog 
Step I:​ diverse daily 79 do 57 dear 86 63 domain 42 dog 41 
Step II:​ domain diverse daily 79 do 57 dear 86 63 dog 41 24 
Step III:​ daily domain diverse 79 do dear 86 63 dog 41 24 75 
Step IV:​ dear daily domain diverse 79 do 86 dog 41 24 75 36 
Step V:​ dog dear daily domain diverse do 86 41 24 75 36 97 
Step VI:​ do dog dear daily domain diverse 41 24 75 36 97 68 
Step VI is the largest step of above arrangement as the intended arrangement is obtained 
As per the rules followed in the given steps, find out the appropriate steps for the Input. 
Input: table 63 tour 19 typhoon 72 25 to tea 48 tablet 56 
1. Which element comes exactly between ‘typhoon’ and ‘tour’ in Step III of the given input? 
2. Which of the following combinations represent the sixth and eighth element in Step II of 
the given input from left end? 
3. If in the last step ‘2’ is added to each of the odd numbers and ‘1’ is subtracted from each 
of the even numbers, then how many numbers multiple of ‘3’ will be formed? 
4. Which element is fourth to the left of one which is ninth from the left end in second last 
step? 
5. If the Step IV, ’72’ interchange its position with ‘table’ and ‘typhoon’ also interchanges its 
position with ’91’ then which element will be to the immediate right of ’91’? 
 
Q2.Directions (1 – 5): Answer the questions on the basis of the information given below. 
A number arrangement machine when given an input of words/numbers, rearranges them 
following a particular rule in each step. The following is an illustration of input and steps of 
rearrangement. 
Input: 29 oat 45 lap ice 18 21 ace 37 van 91 sun 
Step 1: ace 29 oat 45 lap ice 21 37 van 91 sun 18  
Step 2: ace ice 29 oat 45 lap 37 van 91 sun 18 21 
Step 3: ace ice oat 45 lap 37 van 91 sun 18 21 29 
Step 4: ace ice oat lap 45 van 91 sun 18 21 29 37 
Step 5: ace ice oat lap sun van 91 18 21 29 37 45  
Step 6: ace ice oat lap sun van 18 21 29 37 45 91  
Step 7: 22 19 12 15 9 1 18 21 29 37 45 91 
Step 8: 4 10 3 6 9 1 9 3 11 10 9 10 
This is the final arrangement and step 9 is the last step for this input. 
Input: fat pin 94 uni 41 46 oak 13 eat hut 28 93 
1. Which element is 3rd to right of 6th from the right end in step 3? 
2. How many numbers/words are there between uni and 41 in step 6? 
3. If in a certain way ‘oak’ is related to ‘pin’ in step 4 and ‘pin’ is related to ‘13’ in step 5, then 
in the same way ‘8’ is related to what in step 8? 
4. What is the difference in the numbers which is 4th from right end in step 6 and which is 
4th from the right end in last step? 
5. Find the sum of numbers in the last step of the input. 
 
Q3 Directions (1 – 5): Answer the questions on the basis of the information given below. ​A 
number arrangement machine when given an input of numbers/words, rearranges them 

Plutus Academy-(162)
following a particular rule in each step. The following is an illustration of input and steps of 
rearrangement. 
Input: been laws mind need seat then they 
Step 1: need been laws they mind seat then  
Step 2: they need been then laws mind seat 
Step 3: then they need seat been laws mind  
Step 4: seat then they mind need been laws 
Step 5: mind seat then laws they need been 
Step 6: laws mind seat been then they need 
Step 7: been laws mind need seat then they  
Step 8: 26 55 40 28 45 47 58 
Step 9: 26 28 40 45 47 55 58 
This is the final arrangement and step 9 is the last step for this input. 
Input: data high late paid plan risk what 
1. Which word/number is third to right of seventh element from right end in step 4? 
2. In step 5, if ‘plan’ interchanges position with ‘high’ and ‘what’ with ‘data’, then which 
word is between ‘plan’ and ‘data’? 
3. How many words are there in between words ‘late’ and ‘what’ in step 6? 
4. Which number is third to left of 5th from left end in step 8? 
5. Find the difference in numbers which is 4th from right end in step 9 and 4th from 
left end in step 8. 

Q4.Directions (1 – 2): Answer the questions on the basis of the information given below. 
Input:car and home loan borrowers are being addressed. 
Step I: addressed car and home borrowers are being loan. 
Step II:addressed and car borrowers are being loan home. 
Step III:addressed and are borrowers being loan home car. 
Step IV:addressed and are being loan home car borrowers. 
Step V: addressed and are loan home car borrowers being.  
Step V is the last step of the rearrangement of the above input. 
As per the rules followed in the above steps, find out in each of the following questions the 
appropriate step for the given input. 
Input- each of the five questions has only one distinct answer 
Q.1. How many steps would be needed to complete the arrangement? 
(1) Four (2) Six (3) Five (4) Seven (5) None of these 
Q.2. Which step number would be the following output? 
answer each of one only distinct the question has five 
(1) IV  (2) V (3) VI (4) VII (5) None of these 
 
Q5 . ​Directions (1 – 5): Answer the questions on the basis of the information given below. 
Input : 52 peak 91 snow freeze 46 cold 15 high 31 73 trek 
Step I : 15 52 peak snow freeze 46 cold high 31 73 trek 91 
Step II : 15 31 52 peak snow freeze 46 cold high trek 73 91 
Step III : 15 31 46 peak snow freeze cold high trek 52 73 91 
Step IV : 15 31 46 cold peak snow freeze high trek 52 73 91 
Step V : 15 31 46 cold freeze peak snow high trek 52 73 91 
Step VI : 15 31 46 cold freeze high peak snow trek 52 73 91 
 

Plutus Academy-(163)
Step VI is the last step of the rearrangement. As per the rules followed in the above steps, find 
out in each of the following questions the appropriate steps for the given input. 
Input for the questions. 
Input: 67 hot sun 19 best 83 ice 49 ace 77 cut 37 
1. How many steps would be needed to complete the arrangement? 
2. Which Step number would be the following output?:  
19 37 49 ace best hot sun ice cut 67 77 83 
3. Write the Step I ? 
4. Write the last Step? 
5. In Step IV, which of the following word/number would be on 7th position (from the 
right)? 
 
Q6. ​Directions (1 – 5): Answer the questions on the basis of the information given below. 
When an input of words and numbers are given to a word and number 
arrangement machine its rearranges them following a particular rule in each 
step. The following is an illustration of an input and rearrangement given below. 
Input: telecom turtle tares 225 64 toll 49 81 treason 256 
Step I: tares telecom turtle 225 64 toll 81 treason 256 49 
Step II: tares telecom toll turtle 225 81 treason 256 49 64 
Step III: tares telecom toll treason turtle 225 256 49 64 81 
Step IV: tares telecom toll treason turtle 256 49 64 81 225 
Step V: tares telecom toll treason turtle 49 64 81 225 256 
Step VI: 19 13 12 14 5 7 8 9 15 16 
Step VII: 1 4 3 5 5 7 8 9 6 7 
Step VIII: 1 3 4 5 5 6 7 7 8 9 
Step VIII is the last step of the above input arrangement. 
as per the rules followed in the above steps, find out in the following question the 
appropriate step for the given input. 
 
Input: shadow shade 100 324 shack shabby 441 676 snake 289 Squad 841 
1. Write step V? 
2. Step III of above input is: ‘shabby shack shade shadow 441 676 snake Squad 841 100 
289 324’. How many more steps will be required to complete the above rearrangement? 
3. How many steps will be required to complete the above Input? 
4. What will be the last but one step? 
 
Q7. ​Directions (1 – 5): Answer the questions on the basis of the information given below. 
Input: artificials extraordinary can't have superficial elephant 
Step 1: extraordinary artificials can't have superficial elephant 
Step 2: extraordinary superficial artificials can't have elephant 
Step 3: extraordinary superficial artificials elephant can't have 
Step 4: extraordinary superficial artificials elephant have can't 
Step 5: 25 12 19 20 5 20 
Step 6: 7 3 1 2 5 2 
 
Input: Can curious frequently apparent elements fantabulous 
1. Write last step? 
2. Write last but one step? 

Plutus Academy-(164)
3. How many steps will be required to complete the above Input? 
Q 8 Directions (1-5): ​Study the following information carefully and answer the given questions. 
A word and number arrangement machine when given an input line of words and numbers 
rearranges them following a particular rule in each step. The following is an illustration of input 
and rearrangement. ​(BOB PRELIMS 2016)  
Input: 42 chair 65 rocket 29 egg 98 still 85 52 holy apple.  
Step I: apples 42 chair 65 rocket egg 98 still 85 52 holy 28  
Step II: 41 apples 65 rocket egg 98 still 85 52 holy 28 chairs  
Step III: eggs 41 apples 65 rocket 98 still 85 holy 28 chairs 51  
Step IV: 64 eggs 41 apples rocket 98 still 85 28 chairs 51 holys  
Step V: rockets 64 eggs 41 apples 98 still 28 chairs 51 holys 84 
Step VI: 97 rockets 64 eggs 41 apples 28 chairs 51 holys 84 stills 
 
Step VI is the last step of the above arrangement. As per the rules followed in the steps given 
above, find out in each of the following questions the appropriate step for the given input.  
Input: call 85 76 implicit land 59 94 under ape sold 15 90.  
 
1. In which step are the elements ’apes 94 under sold 90’ found in the same order?  
(a) Fifth (b) Seventh (c) Third (d) Fourth (e) There is no such step  
 
2. How many steps will be required to complete the above input?  
(a) Eight (b) Six (c) Five (d) Seven (e) None of these  
 
3. How many words or numbers are there between ‘under’ and ‘84’ as they appear in the fifth 
step of the output? 
(a) Six (b) Three (c) Four (d) Five (e) Seven  
 
4. In the third step of the rearrangement, ‘land’ is related to ‘85’and ‘calls’ is related to ‘14’ in a 
certain way. Which of the following would ‘sold’ be related to, following the same pattern?  
(a) under (b) apes (c) 90 (d) 14 (e) 94  
 
5. Which of the following represents the position of ‘58’ in the last but one step?  
(a) Ninth from the right end (b) Tenth from the left end (c) Seventh from the right end  
(d) Fourth from the left end (e) Both (a) and (d) 
 
Q 9 Directions (1 – 5): ​Answer the questions on the basis of the information given below. A 
number arrangement machine when given an input of words, rearranges them following a 
particular rule in each step. The following is an illustration of input and steps of rearrangement. 
(SBI CLERK Mains 2016) 
Input : 98 Bite 102 Legal 88 54 Mango 21 Cool Zing.  
 
Step1: Bite 98 Legal 88 54 Mango 21 Cool Zing 102  
Step2: Cool Bite Legal 88 54 Mango 21 Zing 102 98  

Plutus Academy-(165)
Step3: Legal Cool Bite 54 Mango 21 Zing102 98 88  
Step4: Mango Legal Cool Bite 21 Zing 102 98 88 54  
Step5: Zing Mango Legal Cool Bite102 98 88 54 21  
 
This is the final arrangement and step 5 is the last step for this input.  
As per the rules followed in the above steps, find out in each of the following questions the 
appropriate steps for the given input. Input for the questions. 
Input: Kite 19 54 Give 31 Right 72 87 Dream Ace.  
1. In step-3, what is position of ‘Ace’ from the left end?  
(a) Third (b) Fifth (c) Second (d) Sixth (e) First  
 
2. In step-5 , how many Letters/Numbers are between Dream and 72?  
(a) Five (b) Two (c) Four (d) Six (e) Three  
 
3. ‘Kite Give Dream Ace Right 87 72 54 31 19’ in which of the following step?  
(a) Step-2 (b) Step-4 (c) Step-5 (d) There is no such Step (e) Step-3  
 
4. In Step-2, which of the following letter/number is 5th right of Kite?  
(a) 31 (b)Give (c)Right (d) 87 (e) 54  
 
5. How many steps would be needed to complete the arrangement?  
(a) Four (b) Two (c) Seven (d) Six (e) Fifth 
 
Q 10 Directions (1 – 5):​ Study the following information carefully to answer the given questions: 
A word and number arrangement machine when given an input line of words and numbers 
rearranges them following a particular rule. The following is an illustration of input and 
rearrangement. ​(IBPS RRB PO 2017) 
Input: 97 nosy 21 snow cold 32 asian 46 65 viral 83 high  
Step I: 211 97 nosy snow cold asian 46 65 viral 83 high 322  
Step II: asian 211 97 nosy snow 46 65 viral 83 high 322 cold  
Step III: 463 asian 211 97 nosy snow viral 83 high 322 cold 654  
Step IV: high 463 asian 211 97 snow viral 83 322 cold 654 nosy  
Step V: 835 high 463 asian 211 snow viral 322 cold 654 nosy 976  
Step VI: snow 835 high 463 asian 211 322 cold 654 nosy 976 viral  
Step VI is the last step of the rearrangement.  
As per the rules followed in the above steps, find out in each of the following questions the 
appropriate steps for the given input.  
Input: peak 18 utility 76 emerge 27 beautiful 37 51 visible 86 know  
1. How many steps would be needed to complete the arrangement?  
(a) X (b) VIII (c) V (d) VI (e) None of these  
 
2. What will the addition of the numbers which is fifth from the left end in step II and 5th from 
the right end in step IV?  

Plutus Academy-(166)
(a) 312 (b) 210 (c) 162 (d) 165 (e) None of these  
3. Which of the following would be the difference of the numbers which is 2nd from left end in 
step IV and 2nd from right end in Step II?  
(a) 290 (b) 83 (c) 193 (d) 101 (e) None of these  
 
4. Which of the following element will be 6th from the left of 3 rd from the right end in step V?  
(a) 181 (b) beautiful (c) 373 (d) know (e) None of these  
 
5. In Step IV, which of the following word/number would be on 4th position (from the left end)?  
(a) visible (b) 181 (c) 97 (d) utility (e) None of these  
 
Q 11 Directions (1 – 5):​ Study the following information carefully and answer the given 
questions. (​SBI PO MAINS 2016​) 
A word and number arrangement machine when given an input line of words and numbers 
rearranges them following a particular rule in each step. The following is an illustration of input 
and rearrangement.  
Input: 23 56 price 59 terrific 72 unit situation 86 information  
Step I : 88 informatioish 23 56 price 59 terrific 72 unit situation  
Step II : pricish 74 88 informatioish 23 56 59 terrific unit situation  
Step III: 57 situatioish pricish 74 88 informatioish 23 56 terrifiish unit  
Step IV : terrifiish 58 57 situatioish pricish 74 88 informatioish 23 unit  
Step V : 21 uniish terrifiish 58 57 situatioish pricish 74 88 informatioish  
Step V is the last step of the above arrangement.  
 
As per the rules followed in the steps given above, find out in each of the following questions 
the appropriate step for the given input.  
Input : 22 39 since 12 growth sector 76 future demand 25  
1. Which step number would be the following output?  
futurish 37 78 demanish 22 since 12 sector growth 25  
(a) Step IV (b) Step V (c) Step VI (d) Step VII (e) There will be no such step  
 
2. How many elements (words or numbers) are there between ‘futurish’ and ‘24’ as they appear 
in the last step of the output?  
(a) One (b) Three (c) Four (d) Five (e) None of these  
 
3. What is sum of the numbers which is second from the right and fourth from the left in the 
third step?  
(a) 37 (b) 39 (c) 33 (d) 49 (e) None of these  
 
Directions 12 (1-5):​ A number arrangement machine when given an input line of numbers 
rearranges them following a particular rule in each step. The following is an illustration of input 
and rearrangement. (IBPS CLERK MAINS 2016) 
Input : where age 42 earn 17 Money in daddy 7 9  

Plutus Academy-(167)
Step I : age when 42 earn 17 Money in daddy 7 9  
Step II : age 42 we earn 17 Money in daddy 7 9  
Step III : age 42 earn we 17 Money in daddy 7 9  
Step IV : age 42 earn 7 we 17 Money in daddy 9  
Step V : age 42 earn 7 in we 17 Money daddy 9  
Step VI : age 42 earn 7 in daddy we 17 Money 9  
Step VII : age 42 earn 7 in 17 daddy we Money 9  
Step VIII : age 42 earn 7 in 17 daddy 9 we Money  
Step IX : age 42 earn 7 in 17 daddy 9 Money we  
Step IX is the last step.  
 
1. If the following is the II nd step of an input what will be Vth step?  
Step II : After 89 she 38 wins 11 Olympic 22 the 7  
(1) after 89 she 7 the 22 Olympic 11 wins 38  
(2) after 89 Olympic she 38 wins 11 22 the 7  
(3) after 89 Olympic 7 she 38 the wins 11 22  
(4) after 89 Olympic 7 she 38 the 11 wins 22  
(5) None of the above  
 
2. Which of the following is the last step for the Input ‘eat 9 fast icecream 22 3 umbrella cat 5’?  
(1) cat eat 9 fast 5 icecream 22 umbrella 3  
(2) eat 22 icecream 3 umbrella 9 cat 5 fast  
(3) eat 22 umbrella 3 icecream 9 cat 5 fast  
(4) eat 22 icecream 3 umbrella5 cat 9 fast  
(5) None of the above  
 
3. Which step will be the last step for the Input ‘elephant 17 free open 41 27 danger 15’?  
(1) IV (2) V (3) VI (4) VII (5) None of these  
 
4. Which word/number will be at 4th from the left in step V for the given input in above 
question?  
(1) 41 (2) danger (3) open (4) 15 (5) None of these  
 
5. Which word/number will be 3rd to the right of ‘41’ in step IV for the given input in Q. 3?  
(1) open (2) danger (3) 15 (4) 17 (5) None of these  
 
 
 
 
 
 
 
 

Plutus Academy-(168)
ANSWER KEYS 
Q no 1 Solutions:

1. A
2. D
3. A
4. C
5. A
 
Q no 2 Solutions: 
Words are arranged according to starting letter – first vowels, then consonants 
For numbers: Multiply the digits of numbers and arrange in increasing order and then arrange in 
steps 
like in given example: numbers are 21, 18, 91, 29, 45, 37 
Multiply the digits 2, 8, 9, 18, 20, 21. Arrange according to this order from back 
Step 7: number corresponding to first letter of word. ace(1) ice(9) oat(15) lap(12) sun(19) 
van(22) 
Put in back order: 22 19 12 15 9 and 1 
Step 8: add the digits of numbers. 
Input: fat pin 94 uni 41 46 oak 13 eat hut 28 93 
Step 1: eat fat pin 94 uni 41 46 oak hut 28 93 13 
Step 2: eat oak fat pin 94 uni 46 hut 28 93 13 41 
Step 3: eat oak uni fat pin 94 46 hut 93 13 41 28 
Step 4: eat oak uni fat pin 94 hut 93 13 41 28 46 
Step 5: eat oak uni fat pin 94 hut 13 41 28 46 93 
Step 6: eat oak uni fat pin hut 13 41 28 46 93 94 
Step 7: 8 16 6 21 15 5 13 41 28 46 93 94 
Step 8: 8 7 6 3 6 5 4 5 10 10 12 13  
1. C 
2. E 
3. A 
4. E 
5. B 
 

Plutus Academy-(169)
Q no 3 Solutions: 
Observe that input and step 7 is same. 
In steps 1 to 7: 
Fourth word is taken at beginning and last word at fourth place. In each step this is performed.  
Step 8: Addition of all numbers corresponding to letters in word. been = 2+5+5+14 = 26 
Step 9: All numbers arranged in ascending order 
Input: data high late paid plan risk what 
Step 1: paid data high what late plan risk  
Step 2: what paid data risk high late plan 
Step 3: risk what paid plan data high late 
Step 4: plan risk what late paid data high 
Step 5: late plan risk high what paid data 
Step 6: high late plan data risk what paid 
Step 7: data high late paid plan risk what 
Step 8: 26 32 38 30 43 57 52 
Step 9: 26 30 32 38 43 52 57 
1. C 
2. E 
3. D 
4. A 
5. C 
 
Q no 4 Solutions: 
1. 3 
2. 1 
 
Q no 5 Solutions: 
Explanation: 
There are six numbers and six words in the input. The three numbers are placed in the beginning 
and the remaining three numbers are placed in the last. The numbers are rearranged in 
ascending order. The six words are rearranged in alphabetical order in the middle. 
Input : 67 hot sun 19 best 83 ice 49 ace 77 cut 37 
Step I : 19 67 hot sun best ice 49 ace 77 cut 37 83 
Step II : 19 37 67 hot sun best ice 49 ace cut 77 83 
Step III : 19 37 49 hot sun best ice ace cut 67 77 83 
Step IV : 19 37 49 ace hot sun best ice cut 67 77 83 
Step V : 19 37 49 ace best hot sun ice cut 67 77 83 
Step VI : 19 37 49 ace best cut hot sun ice 67 77 83 
Step VII : 19 37 49 ace best cut hot ice sun 67 77 83 

Plutus Academy-(170)
Seven steps are needed to complete the arrangement. 
1. 7 
2. Step V 
3. 19 67 hot sun best ice 49 ace 77 cut 37 83 
4. 19 37 49 ace best cut hot ice sun 67 77 83 
5. Sun 
 
Q no 6 Solutions: 
1. Shabby shack shade shadow snake Squad 841 100 289 324 441 676   
2. 6 
3. 9 
4. 7 2 5 5 5 4 1 8 9 3 8 2 
 
Q no 7 Solutions: 
1. 1 7 1 1 2 5 
2. 19 25 19 19 20 14 
3. 7 
 
Q no 8 Solutions: 
Directions (1-5): Logic: In the given machine input, in first step smallest letter according to 
dictionary is arranged with addition of “s” at extreme left side and the smallest number among 
all by subtracting “1” is arranged at extreme right side and in next step second smallest number 
among all by subtracting “1” is arranged at extreme left side and second smallest letter 
according to dictionary is arranged with addition of “s” at extreme right side. And this process is 
continued upto last step. Input: call 85 76 implicit land 59 94 under ape sold 15 90.  
Step I: apes call 85 76 implicit land 59 94 under sold 90 14  
Step II: 58 apes 85 76 implicit land 94 under sold 90 14 calls  
Step III: implicits 58 apes 85 land 94 under sold 90 14 calls 75  
Step IV: 84 implicits 58 apes 94 under sold 90 14 calls 75 lands  
Step V: solds 84 implicits 58 apes 94 under 14 calls 75 lands 89 
Step VI: 93 solds 84 implicits 58 apes 14 calls 75 lands 89 unders  
1. (d)  
2. (b) 
3. (c)  
4. (a)  
5. (e)  
 
Q no 9 Solutions: 
Direction (1-5)  
Input: Kite 19 54 Give 31 Right 72 87 Dream Ace.  
Step1: Ace Kite 19 54 Give 31 Right 72 Dream 87.  
Step2: Dream Ace Kite 19 54 Give 31 Right 87 72.  

Plutus Academy-(171)
Step3: Give Dream Ace Kite 19 31 Right 87 72 54.  
Step4: Kite Give Dream Ace 19 Right 87 72 54 31.  
Step5: Right Kite Give Dream Ace 87 72 54 31 19.  
‘Step-5’ is the final step of this input.  
1. (a)  
2. (b)  
3. (d)  
4. (c)  
5. (e)  
 
Q no 10 Solutions:  
Logic: - There are six numbers and six words in the input. In the first step the numbers are 
arranged in ascending order from both the ends with a natural number starting from 1 at unit 
place in left end number and with a natural number 2 at unit place in right end number. After that 
in second step the words are arranged in alphabetical order from both the ends. And then again 
number are arranged in third step and words are arranged in forth step and so on.  
Input: peak 18 utility 76 emerge 27 beautiful 37 51 visible 86 know  
Step I: 181 peak utility 76 emerge beautiful 37 51 visible 86 know 272  
Step II: beautiful 181 peak utility 76 37 51 visible 86 know 272 emerge  
Step III: 373 beautiful 181 peak utility 76 visible 86 know 272 emerge 514  
Step IV: know 373 beautiful 181 utility 76 visible 86 272 emerge 514 peak  
Step V: 765 know 373 beautiful 181 utility visible 272 emerge 514 peak 866  
Step VI: utility 765 know 373 beautiful 181 272 emerge 514 peak 866 visible  
1. (d)  
2. (c)  
3. (d)  
4. (b)  
5. (b)  
 
Q no 11 Solutions:  
The machine rearranges one number and one word in each step. Words are arranged in an 
alphabetical order whereas numbers are arranged in an order of decreasing order and there is 
an addition of two in every even numbers and subtraction of two in every odd numbers. In first 
step, firstly there is an arrangement of number and then alphabet. In second step, firstly there is 
an arrangement of alphabet and then number and so on. And there is replacement of last digit 
of alphabets with ‘ish’ after rearrangement.  
Input: 22 39 since 12 growth sector 76 future demand 25  
Step I : 78 demanish 22 39 since 12 growth sector future 25  
Step II : futurish 37 78 demanish 22 since 12 growth sector 25  
Step III: 23 growtish futurish 37 78 demanish 22 since 12 sector  
Step IV: sectoish 24 23 growtish futurish 37 78 demanish since 12  
Step V : 14 sincish sectoish 24 23 growtish futurish 37 78 demanish  
1. (e)  

Plutus Academy-(172)
2. (e)  
3. (d) 
 
Q no 12 Solutions:  
1. 3  
2. 2  
3. 3  
4. 4  
5. 2 
 
 
 
 
 
 
 
 
 
 
 
 
 
 
 
 
 
 
 
 
 
 
 
 
 
 
 
 
 
 
 
 
 
 

Plutus Academy-(173)
Data Sufficiency 
In  this  type  of  questions  a  problem  is  given. Problem can be related to any topic such as puzzle 
test,  coding-decoding,  distance  and  directions,  blood  relation  etc.  is  given,  followed  by  certain 
statements.  These  statements  will  give  certain clues to solve the given question. The candidate 
is required to find out which statements is/are sufficient to answer the given question. 
 
Q1. Who among siddhartha, Nikunj, Vipul and Mukul is the youngest? 
I. Vipul is younger than Mukul but older than siddhartha and Nikunj. 
II. Mukul is the oldest. 
III. Siddhartha is older than Nikunj. (SBI PO 1997) 
1. Only I 
2. I and II together 
3. II and III together 
4. I and III together 
5. None of these 
 
Directions(2-7)  :  Each  of  the  questions  below  consists  of  a  question  and  the  two  statements 
numbered  I  and  II  are  given  below  it.  you  have  to  decide  whether  the  data  provided  in  the 
statements are sufficient to answer the question. Read both the statements and -
(IBPS PO/MT 2014) 
 
Give  answer(1)  If  the data in statement I alone are sufficient to answer the question, while the 
data in statement II alone are not sufficient to answer the question. 
 
Give  answer(2)  If  the  data  in  statement  II  alone  are  sufficient  to  answer  the  question,  while 
the data in statement I alone are not sufficient to answer the question. 
 
Give  answer (3) If the data in statement I alone or in statement II alone sufficient to answer the 
question. 
 
Give  answer  (4)  If the data in both the statements I and II together are not sufficient to answer 
the question. 
 
Give  answer  (5) If the data in both the statements I and II together are necessary to answer the 
question. 
 
Q2. 6 friends A B C D E and F are sitting around a circular table. some of them are facing outside 
while some others are facing towards the centre. What is the position of C with respect F? 
I.   C  is  sitting  second  to  the  left  of  D,  D  is  facing  towards  the  centre.F  is  an  immediate 
neighbour  of  both A and D. E is sitting second to the right of the B. B.is not an immediate 
neighbour of A. F faces just opposite to that of B. 

Plutus Academy-(174)
II. Two  persons  are  sitting  between  D  and E. D and E are facing towards the centre.  E is an 
immediate  neighbour  of  both  C  and  A.  F  faces  the  same  direction  as  that  of  B.  D  is  an 
immediate neighbour of both B and F. F is not an immediate neighbour of C. 
 
Q3.  7 people P, Q, R, S, T, W and X are sitting in a straight line facing North but not necessarily in 
the same order. how many people sit to the right of P? 
I. R  sits  at  one  of  the  extreme  end  of  the line. T has as many people sitting on his right, as 
to his left. 
II. S  sits  third  to  the  left  of  X.  Q  sits  to  the  immediate left of the W. Q does not sit at any of 
the extreme ends of the line. 
 
Q4. Points M is towards which direction from point H? 
I.   If  a  person  walks  6  metres  towards  west  from  point  M,  takes  a  left  turn  and  walks  6 
metres again, he would be 5 metres away from the point H. 
II. Point  M  is  towards  the  north  of  point  N;  point  N  is  towards the east of point T and point 
H is towards the East of point T. 
 
Q5. How part is written in a certain code language? 
I. In  that  code language ‘going to a party’ is written as ‘la fa gi ne’ and ‘for a party’ is written 
as ‘fa di ne’. 
II. In  that  code  language  ‘start  the  party’  is  written  as  ‘ne  bs  am’  and  ‘going  to  start’  is 
written as ‘gi bs la’. 
Q6. How is ranjay related to Parvati? 
I. Ranjay is son of Parvati’s grandfather’s only daughter. 
II. Ranjay has no siblings. Parvati has only one brother. 
 
Q7. Among M, R, H, D and S who scored the highest marks in an Examination? 
I. R scored more than D but less than S. 
II. M scored less than H and D. H has not scored the highest marks. 
 
Directions(8-12)  :  Each  of  the  questions  below  consists  of  a  question  and the two statements 
numbered  I  and  II  are  given  below  it.  you  have  to  decide  whether  the  data  provided  in  the 
statements are sufficient to answer the question. Read both the statements and -
(SBI PO 2014) 
 
Give  answer(1)  If  the data in statement I alone are sufficient to answer the question, while the 
data in statement II alone are not sufficient to answer the question. 
 
Give  answer(2)  If  the  data  in  statement  II  alone  are  sufficient  to  answer  the  question,  while 
the data in statement I alone are not sufficient to answer the question. 
 
Give  answer (3) If the data in statement I alone or in statement II alone sufficient to answer the 
question. 

Plutus Academy-(175)
 
Give  answer  (4)  If the data in both the statements I and II together are not sufficient to answer 
the question. 
 
Give  answer  (5) If the data in both the statements I and II together are necessary to answer the 
question. 
 
Q8. Who is the lightest amongst A, B, C, D, And E. 
I. D is heavier than C and lighter than A. 
II. B is heavier only than E. 
 
Q9. On which date of the month was rajiv born? 
I. Rajiv’s mother correctly remembers that he was born after 12th and before 18th of May. 
II. Rajiv’s father correctly remembers that he was born before 23rd and after 16th of May. 
 
Q10. How many brothers does sonal have? 
I. Nisha, the mother of sonal’s brother has only two children. 
II. Sonal’s brother has only one sister. 
 
Q11. Town M is towards which direction of Town L? 
I. Driving 50km in a straight line from Town M leads to Town L? 
II. Towns M and L are located between towns K and P. 
 
Q12. How is ‘Late’ coded in the language? 
I. ‘Came late to office’ is coded as ‘so ti ly ja’ and late in the night ‘ is coded as ‘fo pa ti me’. 
II. ‘It  was  late  summer’  is  coded as ‘ru ki ne ti’ and ‘reached two hours late’ is coded as ‘ti le 
di co’.  
 
Directions(13-14)  :  Each  of  the  questions  below  consists  of  a  question  and  three  statements 
numbered  I,II  and  III  given  below  it.  You  have  to  decide  whether  the  data  provided  in  the 
statements are sufficient to answer the question: (IBPS PO/MT 2013) 
Q13. Who is the daughter in law of B? 
I. I is the brother of D. S is the wife of J’s nephew. 
II. R is the brother of N. T is the son of N. S is the mother of T. 
III. B is the wife of D. D is the father of N. D has two children. 
(1) Only I and III 
(2) All I, II and III 
(3) Only II and III 
(4) Questions cannot be answered even with all I, II and III. 
(5) Only I and II 
 
Q14. How many students are there in the class? 
I. There are more than 22 but less than 36 students in the class. 

Plutus Academy-(176)
II. If students of the class are divided into groups each group has exactly 11 students. 
III. There are more than 29 but less than 45 students in the class. 
(1) Only I and II 
(2) Only II and either I or III are required to answer the question 
(3) Only II and III 
(4) All I, II and III are required to answer the question 
(5) All I, II and III are not sufficient to answer the question. 
 
Directions(15-18)  Each  of  the  questions  below  consists  of  a  questions  below  consists  of  a 
question  and  three  statements numbered I,II and III given below it. You have to decide whether 
the data provided in the statements are sufficient to answer the question. 
(SBI ME 2014) 
 
Q15.  Who  amongst  P,Q,R,S,T  and  V  ,each  securing  different  marks,  secured  the  second  lowest 
marks? 
I. R and T secured more marks than P and Q 
II. V secured the highest marks 
III. S secured more marks than P but less than Q. 
(1) Only I and III 
(2) All I,II and III are required to answer the question 
(3) Only II and III 
(4) Question cannot be answered even with all I, II and III 
(5) Only I and II 
 
Q16. Which village is to the North-East of Village R? 
I. Village  S  is  to  the  South-East  of  Village  N  which  is  to  the  South-West  of  village  P  and 
Village P is to the North of Village Q. 
II. Village T is to the North-West of Village Q which is to the south of village P. 
III. Village  R  which  is  to  the North of Village S, lies between villages N and Q and village N is 
to the West of Village R. 
(1) Only I and II 
(2) Only II and III 
(3) All I, II and III are not sufficient to answer the question  
(4) All I, II and III are required to answer the question 
(5) Only I and III or only II and III are required to answer the question 
 
Q17. What is the rank of Animesh in a class of 17 students? 
I. Nirmal  who  is  thirteen  from the bottom is six rank ahead of bhumika who is two position 
below Animesh. 
II. Bhumika is four position ahead of kamal. 
III. Bhumika is two position below Animesh and Kamal’s rank is 15th. 
(1) Only I and III 
(2) Only I and II 

Plutus Academy-(177)
(3) Only I or II and III together are required to answer the question 
(4) Only II is required to answer the question 
(5) All I, II and III are not sufficient to answer the question 
 
Q18. How is ‘them’ written in a code language? 
I. ‘Tell  them  young’  is  written  as  ‘seme  ye’  and  ‘wise  young  sharp  tell’  is  written  as  ‘me yo 
na ye’ in that code language. 
II. ‘Clever  sharp  come  tomorrow’  is  written  as  ‘na  ki  pa  lo’  and  ‘bring  clever  young  them’ is 
written as ‘ki po se ye’ in that code language. 
III. ‘Clever  sharp  come  tomorrow’  is  written  as  ‘pa  na  se  ki  te’  and  ‘yellow  come  sharp  run 
clever no’ is written as ‘ki ni pa be te na’ in that code language. 
(1) Only III is required to answer the question  
(2) Only I and II are required to answer the question 
(3) Only I or II and III together are required to answer the question. 
(4) Only II is required to answer the question 
(5) All I,II and III are not sufficient to answer the question. 
 
Answer Key- 
1. (4) I and III together 
2. (3) 
3. (5) 
4. (4) 
5. (5) 
6. (5) 
7. (5) 
8. (2) 
9. (5) 
10. (1) 
11. (4) 
12. (3) 
13. (3) 
14. (1) 
15. (2) 
16. (5) 
17. (3) 
18. (1) 
 
 
 
 
 
 
 

Plutus Academy-(178)
 

Data sufficiency questions asked in previous years 


Directions  (1-2):  In  each  of  the  following  questions,  a  question  is  followed  by  three  statements 
numbered  I,  II  and  III.  Read  all  the  statements  to  find  the  answer  to  given  question  and  then 
answer accordingly that which statement/s can give the answer alone/together.   
(SBI PO MAINS 2017) 
Q1. What is the direction of point U with respect to point X? 
Statement  I​:  Point  R  is  7 m to the North of point Q. Point P is 8 m to the West of point Q. Point R 
is 6 m to the West of point U. 
Statement  II:  Point  B  is  9  m  to  the  North  of  point A. Point P is 5 m to the North of point Z. Point 
Z is 4 m to the West of point A. 
Statement  III​:  Point  C  is  7  m  to  the  East  of  point A. Point X is 2 m to the East of point F. Point F 
is 3 m to the North of point C 
(a) Both I and III 
(b) Both II and III 
(c) All I, II and III 
(d) II and either I or III 
(e) Even I, II and III together are also not sufficient 
Q2. What does the code ‘bp’ stand for in the given code language? ​(SBI PO MAINS 2017) 
Statement  I:  In  the  language,  ‘black  white  red’  is  coded  as  ‘df  dc  or’  and  ‘green  blue  grey’  is 
coded as ‘st hn wo’ 
Statement  II:  ​In  the  language,  ‘blue  pink  brown’  is  coded  as  ‘er  bp  hn’  and  ‘pink  blue  white’  is 
coded as ‘hn or bp’ 
Statement  III:  In  the  language,  ‘green  violet  orange’  is  coded  as  ‘pa  wo  kl’  and  ‘yellow  pink 
brown’ is coded as ‘bp bi er’ 
(a) Both II and III 
(b) I and either II or III 
(c) II and either I or III 
(d) Both I and III 
(e) All I, II and III 
Directions  (3-7)  ​–  Each  of  the  following  questions  below  consists  of  a  question  and  two 
statements  numbered  I  and  II  given  below  it.  You  have  to  decide  whether  the  data  provided  in 
the statements are sufficient to answer the question. Give Answer ​(IBPS CLERK MAINS 2016) 
a)  If  the  data  in  Statement  I  alone  are  sufficient  to  answer  the  question,  while  the  data  in 
Statement II alone are not sufficient to answer the question 
b)  If  the  data  in  Statement  II  alone  are  sufficient  to  answer  the  question,while  the  data  in 
Statement I alone are not sufficient to answer the question 
c)  If  the  data  either  in  Statement  I  alone  or  Statement  II  alone  are  sufficient  to  answer  the 
question 
d) If the data in both the Statement I and II together are not sufficient to answer the question 
e) If the data in both the Statement I and II are together necessary to answer the question 
 
 

Plutus Academy-(179)
3.​ What is the difference in the age of Paarth and Keshav? 
I. Paarth is 20 years older than Mohan 
II. Mohan is 2 years younger than Zorawar 
 
4.​ The sum of ages of Manak, Neesha and Opendar is 50 years. What is Neesha’s age? 
I. Neesha is 10 year older than Manak 
II. Opendar is 30 year old 
 
5.​ How many students are there in the class? 
I. There are more than 20 but less than that 27 students in the class. 
II.  There  are  more  than  24  but  less  than 31 students in the class. The number of students in the 
class can be divided into groups such that each group contains 5 students. 
 
6.  ​Among  five  friends  –  Jatin,  Kalu, Lucky, Manish and Naresh each of a different height, who is 
the second tallest? 
I. Naresh is taller than Manish and Kalu. Kalu is shorter than Manish. 
II. Lucky is taller than Naresh. Jatin is not the tallest. 
 
7.​ How is ‘call’ written in a code language? 
I. ‘call me back’ is written as ‘531’ in that code language. 
II. ‘you can call me any time’ is written as ‘94163’ in that code language 
 
Direction  (8-12):  Each  of  the  questions  below  consists  of  a  question  and  two  statements 
numbered  I  and  II  given  below  it.  You  have  to  decide  whether  the  data  provided  in  the 
statements are sufficient to answer the question. Read both the statements and give answer.  
a)  If  the  data  in  statement  I  alone  are  sufficient  to  answer  the  question,  while  the  data  in 
statement II alone are not sufficient to answer the question.  
b)  If  the  data  in  statement  II  alone  are  sufficient  to  answer  the  question,  while  the  data  in 
statement I alone are not sufficient to answer the question.  
c)  If  the  data  either  in  statement  I  alone  or  in  statement  II  alone  are  sufficient  to  answer  the 
question.  
d) If the data in both statements I and II together are not sufficient to answer the question.  
e) If the data in both statements I and II together are necessary to answer the question. 
(BOB 2016) 
8.  ​There  are  six  persons  P,  Q,  R,  S, T and U sitting around a table. Who is on the immediate right 
of T? 
I. Only U is sitting between T and P. 
II. P is third to the right of S. 
 
9. ​What is the shortest distance between two points `A’ and `B’?  
I. ‘A’ is 15 km North from another point ‘ P’, which is to the East of ‘B’ at a distance of 22 km. 
II.  point  ‘X’ is to the West of ‘B’ at a distance of 5.6 km and to the North of ‘A’ at a distance of 6.5 
km. 
 
10.​ Who is Kavita’s sister? 
I. Sapna is the granddaughter of Amit, who is the father of Kavita’s father. 
II. The name of Kavita’s sister starts with the letter ‘S’. 

Plutus Academy-(180)
11.​ Who among the five friends Sumit, Tare, Mukesh, Vikrant and Basuki is the youngest? 
I. Basuki is older than three of them but Sumit is younger than Mukesh. 
II. Tare is older than Vikrant and Basuki. 
 
12.​ What is the code for ‘book’ in a code language? 
I.  In  that  language  `pik  tik  rik’means  ‘I  like  reading’  and  ‘mik  nek  bek  fek’  means  ‘this  book  is 
interesting’. 
II.  In  the  same  language  ‘pik  tik  mik  rik’means  ‘I  like  reading  book’ and ‘mik juk pan’ means ‘you 
write book’. 
 
Solutions: 
1. (c) All I, II and III 
2. (c) II and either I or III 
3.  (d)Relation  between  Paarth  and  Keshav  can  not  be  established  with  the  help  of  both 
Statements I and II. So, both the statements are not sufficient to answer the question. 
4. (e) 
5.  (e)  From  Statement  I  Number  of  Students  in  class  =  21  or  22  or  23  or  24  or  25  or  26  From 
Statement  II  Number  of  Students  in  class  =  25  or  26  or  27  or  28  or  29  or  30  Since,  number  of 
students  can  be  divided  into  groups  of  5  students,  therefore  number  of  students  must  be 
multiple  of  5.  Number  of  students  in  the  class  =  25  or  30  On  combining  Statements  I  and  II 
Number  of  students  in  the  class  = 25 Hence, the data in both the Statement I and II together are 
necessary to answer the question. 
6. (d)From Statement I: Naresh > Manish > Kalu 
From Statement II: Lucky > Naresh 
Hence from both the Statements 
Lucky > Naresh > Manish > Kalu 
But height of Jatin is not clear. 
Thus, second tallest friend can’t be determined. 
Hence, data neither in Statement I nor in Statement II are sufficient to answer the question. 
7. (d) Both the statements are not sufficient. 
8. (d) Data in both the statements I and II together are not sufficient to answer the questions. 
9.  (c)  Data  either  in  statement  I  alone  or  in  statement  II  alone  are  sufficient  to  answer  the 
question. 
10.  (d)  From  statement  I  and  II  together  still  it  is  still  not  clear  who  is  Kavita’s  sister.  It  is  not 
sure whether Sapna is sister or cousin of Kavita. 
11. (d) From I and II : – Still we do not know who is the youngest. 
12. (b) From II. we get code for book is “mik”. 
 
 
 
 
 
 
 
 

Plutus Academy-(181)
Statement and Assumption
Format of Question:

In each question, below is given a statement followed by 2 assumption numbered 1 & 2.

An ​assumption ​is something supposed or taken for granted. You have to consider the
statement and the following assumptions and decide which of the assumption is implicit
in the statement.

Key Points:
● Statement is a part of our speech of which most of the ideas remain
unexpressed.
● Assumption is something in which the reader assume something on the basis of
the given statement.
● If statement and assumption are not co-related, then it is not implicit.
● Assumption is always positive regarding the completion of the work favoring the
executor.
● Request/advise/order is considered valid in the case of assumption.

There may be 4 types of question.


1. Advertisement based
2. Warning based
3. Information based
4. Passage based.

Directions(1-4):​ In each of these question given a statement followed by two


assumptions numbered I and II. An assumption is something supposed or taken for
granted. You have to consider the statement and the following assumptions and decide
which of these is/are implicit in the statement.Give answers:

(1) Only I is Implicit


(2) Either I or II is implicit
(3) Only II is implicit

Plutus Academy-(182)
(4) Both I and II are implicit
(5) Neither I nor II is implicit

1. St: An advertisement in the newspaper- “Learn a foreign language course to get a


high paying job”
Assumption: I. All those who learn foreign language get highly paid job.
II. Only a foreign language can get someone a high paying job.

2. St: “Do not lean out of the door of the bus”- A warning in a school bus.
Assumption: I.Leaning out of running bus is dangerous.
II. Children do not pay attention to such warning

3. St: “Do not enter- avoid the risk of getting infected with the ABC disease”- written
outside the quarantine ward no. 2(meant only for ABC disease of a hospital)
Assumption: I.Disease ABC is contagious.
II. All the patients in ward no 2 suffer from disease ABC.

4. St: India’s contribution to the growing global market of management consultancy may
touch 5 billion dollars by 2030, which is 10% of the current export size.
Assumption: I.The current export size is 50 billion dollar.
II. The current year is 2010.

Q1. The constable has been recommended for a suitable reward by his superior in
recognition of his sincere duty and busting of several gangs of criminals actively
involved in the loot and incidents of pickpocketing.
Which of the following can be a possible assumption of the above statement?
(1) The superior is certain that recommendation would be denied.
(2) The number of criminals apprehended by this particular constable was
exceptionally high.
(3) The constable desires to be monetarily compensated for his efforts.
(4) The superior wants to set an example for his other juniors by recommending the
reward.
(5) Rewards recognising the sincerity and accomplishments of policemen are given.
(IBPS SO CWE 2013)

Plutus Academy-(183)
Q2. The Government has appealed to all citizens to use potable water judiciously as
there is an acute shortage in supply. Excessive use may lead to huge scarcity in future
months.
Which of the assumptions is implicit in the above statement?(An assumption is
something supposed or taken for granted)
(1) People may ignore the appeal and continue using water as per their
consideration.
(2) Government may be able to tap those who do not respond to the appeal
(3) Government may be able to put in place alternate sources of water in the event
of a crisis situation.
(4) Large number of people may positively respond to the Government’s appeal and
help tide over the crisis.
(5) Only poor are going to suffer from this shortage of water supply.
(IBPS PO 2011)
Q3. Directions(3-4): In each of these question given a statement followed by two
assumptions numbered I and II. An assumption is something supposed or taken for
granted. You have to consider the statement and the following assumptions and decide
which of these is/are implicit in the statement.

3. Statement: ​Send employee XYZ for a training in UK to gain more insight into the
project which he is handling at present. (SBI Associates PO 2014)
I. Some similar training programs are available in the employee’s own country as
well.
II. With the present skill sets, employee XYZ is incapable of handling the project.
(1) Only I is Implicit
(2) Either I or II is implicit
(3) Only II is implicit
(4) Both I and II are implicit
(5) Neither I nor II is implicit

4. Statement: ​Minute traces of nitrate sometimes present in cattle fodder B may be


responsible for the poor health of the cattle therefore use fodder G to ensure that your
cattle remain healthy.
I. Cattle fodder G does not contain nitrate.
II. Nitrate is toxic to all living beings.
1. Only I is Implicit
2. Either I or II is implicit
3. Only II is implicit
4. Both I and II are implicit

Plutus Academy-(184)
5. Neither I nor II is implicit

Q5. Statements: ​The ‘X’ Finance company has launched a new tractor loan scheme. It
has brought prosperity among the farmers.
Which of the following assumptions is implicit in the above statements?
(1) The prosperity of farmers solely depends upon the tractor loan scheme.
(2) Tractor is an important input for farmers to enhance the farm activities
(3) Earlier there was no tractor loan scheme for farmers
(4) Farmers do not prefer to take loans for buying tractors.
(5) None of these (IBPS PO/MT 2013)

Directions(6-11): In each question below is given a statement followed by two


assumptions numbered I and II. An assumption is something supposed or taken
for granted. You have to consider the statement and the following assumptions
and decide which of the assumptions is implicit in the statement.
Give answer (1) if only assumption I is implicit.
Give answer (2) if only assumption II is implicit.
Give answer (3) if either assumption I or assumption II is implicit.
Give answer (4) if neither assumption I or assumption II is implicit.
Give answer (3) if both assumptions I and II are implicit. (BOI PO 2010)

Q6. Statement: ​Job rotation helps employee get an overview of the organisation
Assumptions:
I. Job relation is the only method to get an overview of the organisation.
II. It is required to have an overview of the organisation.

Q7. Statement: Let us appoint Ms. X as the CEO of our Company so that the
company’s products are also perceived to be genuine.
Assumptions:
I. CEO can change the perception of products.
II. Perception is same as the actual reality.

Q8. Statement: ​An advertisement -


The new model has been launched with K-series engine.
Assumptions:
I. People know about K-Series engine.
II. Engine type/series is important for buyers

Q9.Statement: ​Mohan tells Nita, “Let us meet over lunch tomorrow”.

Plutus Academy-(185)
Assumptions:
I. Lunch timings are known to both.
II. Both are aware of the venue for launch.

Q10. Statement: ​You need to be talented to identify talent


Assumptions:
I. Talent is acquired and developed.
II. Talent is hereditary.

Q11. Statement : ​The movie is a super-duper hit and has broken all the records.
Assumptions:
I. There is no authentic criterion to judge a hit or a flop.
II. The performance of earlier movies is known.

Directions(12-16): In each question below is given a statement followed by two


assumptions numbered I and II. An assumption is something supposed or taken
for granted. You have to consider the statement and the following assumptions
and decide which of the assumptions is implicit in the statement.
Give answer (1) if only assumption I is implicit.
Give answer (2) if only assumption II is implicit.
Give answer (3) if either assumption I or assumption II is implicit.
Give answer (4) if neither assumption I or assumption II is implicit.
Give answer (5) if both assumptions I and II are implicit.
(United India Insurance AAO 2012)
12. Statement: ​“Though the candidates have been instructed to bring pencil, yet
provide some pencils with each invigilator”-
An instruction to test administration staff.
Assumptions:
I. Pencils are in short supply
II. All the candidates will bring the pencil.

13. Statement: ​“Please drop this letter in the letterbox”- An officer tells his assistant.
Assumptions:
I. The Assistant knows the address where the letter is to be sent.
II. The Assistant would follow the instructions.

14. Statements: ​If you want timely completion of work-Provide independent cabins”- An
employee tells the director of a Company.
Assumptions:

Plutus Academy-(186)
I. There are not enough cabins
II. Others presence hinders timely timely completion of work.

15.Statements: ​“We need to appoint more teachers”-Principal informs the school staff.
Assumptions:
I. Teachers are available.
II. Present teachers are not good.
Answer Key:
1. 5
2. 4
3. 5
4. 1
5. 2
6. 2
7. 1
8. 5
9. 5
10. 1
11. 2
12. 4
13. 4
14. 4
15. 1

Plutus Academy-(187)
 

Statement  and  assumption  questions  asked  in  previous  years(IBPS 


PO/SBI PO) 
1.  Statement:-  By  asking  five  prominent  hospitals  in  the  national  capital  to  deposit  nearly  Rs. 
600  crore  to  compensate  for their failure to treat poor patients, the Delhi government has drawn 
attention  to  the  social  obligation  of  healthcare  providers  in  the  corporate  sector  as  well  as  the 
need  for  timely  enforcement  of  applicable  regulations.  According  to  the  Delhi  government, 
trusts  and  registered  societies  to  which  public  land  was  allotted  to  establish  hospitals  were 
required  to  earmark  a  percentage  of  their  medical  facilities  and  services  for  indigent  patients. 
(IBPS PO MAINS 2016) 
Assumptions: 
I.  This  strengthens  the  case  for  private  hospitals  to  dedicate  a  part  of  their  services  to  those 
who cannot afford treatment. 
II.Social  responsibility  of  hospitals  must  be  monitored  and  central  government  should  ensure 
compliance by the corporate hospitals. 
III.  There  is a great need for the government to monitor and enforce “health services” so that the 
poor  who  cannot  afford  modern  health  facilities  that  are  extremely  costly-  are  able  to  be 
benefited. 
(a) All are implicit 
(b) Only III is implicit 
(c) Only I and III are implicit 
(d) Only I and II are implicit 
(e) None is implicit 
2.  The  ancient  Nubians  inhabited  an  area  in  which  typhus  occurs,  yet  surprisingly  few  of  their 
skeletons  show  the  usual  evidence  of  this  disease.  The  skeletons  do  show  deposits  of 
tetracycline,  an  antibiotic  produced  by  a  bacterium  common  in  Nubian  soil. This bacterium can 
flourish  on  the  dried  grain  used  for  making  two  staples  of  Nubian  diet,  beer  and  bread.  Thus, 
tetracycline  in  their  food  probably  explains the low incidence of typhus among ancient Nubians.  
(IBPS PO MAINS 2016) 
Which of the following is an assumption that can be drawn from the passage? 
I.  Infectious  diseases  other  than  typhus  to  which  the  ancient  Nubians  were  exposed  are 
unaffected by tetracycline. 

Plutus Academy-(188)
II.  Tetracycline is not rendered ineffective as an antibiotic by exposure to the process involved in 
making bread and beer. 
III.  Typhus  cannot  be  transmitted  by  ingesting  bread  or  beer  contaminated  with  the  infectious 
agents of this disease. 
IV.  Bread  and  beer  were  the  only  items  in  the  diet  of  the  ancients  Nubians  which  could  have 
contained tetracycline. 
V. Typhus is generally fatal. 
(a) Only II follows 
(b) Only III & V follow 
(c) Only III follows 
(d) Only II,III,IV and V follow 
(e) None of these 
3.  The  Government  has  appealed  to  all  citizens  to  use  potable  water  judiciously  as  there  is  an 
acute  shortage  in  supply.  Excessive  use  may  lead  to  huge  scarcity  in  future  months.  Which  of 
the  assumptions  is  implicit  in  the  above  statement?  An  assumption  is  something  supposed  or 
taken for granted) ​(SBI PO MAINS 2017) 
(a)People may ignore the appeal and continue using water as per their consideration 
(b)Government may be able to tap those who do not respond to the appeal 
(c)Government  may  be  able  to  put  in  place  alternate  sources  of  water  in  the  event  of  a  crisis 
situation 
(d)Large  number  of  people  may  positively  respond  to  the  Government’s  appeal  and  help  tide 
over the crisis 
(e)Only poor are going to suffer from this shortage of water supply 
4.  Statement:  Science  is  a  sort  of  news agency comparable in principle to other news agencies. 
But  this  news  agency  gives  us  information  which  is reliable to an extraordinary high degree due 
to  elaborate  techniques  of  verification  and  its  capacity  to  survive  centuries.  So,  science should 
be read with as much interest as we read news. ​(SBI PO MAINS 2017) 
Assumptions: 
I. Science encourages investigative spirit. 
II.People read news out of interest. 
(a) If only assumption I is implicit 
(b) If only assumption II is implicit 
(c) If either I or II is implicit 
(d) If neither I nor II is implicit 

Plutus Academy-(189)
(e) Both I and II are implicit 
5.  Twenty  percent  of  all  energy  consumed  in  the  country  is  consumed  by  home  appliances.  If 
appliances  that  are  twice  as  energy-efficient  as those currently available are manufactured, this 
figure will eventually be reduced to about ten percent.​(IBPS PO MAINS 2016) 
The argument above requires which of the following assumptions? 
(a)  Home-appliance  usage  would  not  increase  along  with  the  energy  efficiency  of  the 
appliances. 
(b) It would not be expensive to manufacture home appliances that are energy-efficient. 
(c)  Home-appliance  manufacturers  now  have  the  technology  to  produce  appliances  that  are 
twice as energy-efficient as those currently available. 
(d)  The  cost  of  energy  to  the  consumer  would  rise  with  increases  in  the  energy  efficiency  of 
home appliances. 
(e) None of these 
SOLUTION: 
1.  (c);Such  strict  actions  of  Delhi  government  made  private  hospitals  to  serve  their  services  to 
poor  people.  So  I  is  implicit.  But  nothing  is  mentioned  about  central  government  in  the 
statement  II  also  it is a course of action, So, II is not implicit. According to statement Delhi Govt. 
wanted  to  provide  medical  facilities  for  indigent  patients.  So  we  can  assume  that there is need 
for the govt. to monitor and enforce health services to the needy person. 
2.  (a);Statement  II  is  a  valid  assumption  as  tetracycline  is  produced  by  a  bacterium  which  is 
found  in  Nubian  soil  and  the  bacterium  can  develop  on the grain which is used in making bread 
&  beer.so  tetracycline  is  formed  through  the  process  of  making  of  bread  &  beer.  Statement I is 
not  valid because nothing is said in the passage about the disease other than typhus. Statement 
III  is  also  not  valid  because  Typhus  is  a  disease  and  the  statement  talks  about  the 
contamination  of  a  disease  by  another  disease  which  is  wrong  in  the  context  of  this  passage. 
Statement  IV  &  V  is  invalid  as  No  information  is  given  in  the  passage  that  bread  &  beer  were 
their only items in the diet and also about Typhus is fatal. 
3.  Ans.(d)  Sol.  Option  (d)  is  an  assumption.  Any  appeal  has  some  effects  and  people generally 
respond positively to any appeal. 
4.  Ans.(d)  Sol.  Both  I  and  II  can’t  be  assumed  from  the given statement as statement I is vague 
and  II  statement  is  also  not  implicit  because  it  is  not  clear  from  the  given  statement  that 
whether people are interested in such news or not. 
5.  Sol:  1.  (a)  This  argument  is  based  on  the  assumption  that  if  the  appliances  become  more 
efficient,  then  more  homes  would  not  be  tempted  to  increase  their  utility/consumption  in  the 
household. 

Plutus Academy-(190)
Statement and Arguments

● Mostly these questions are followed by two contrary argument and you have to
decide which of the given argument is strong and which is weak.
● It is necessary to understand which one is weak and which one is strong.
● Strong arguments are those which are both important and directly related to the
question.
● Weak arguments are those which are less important and not directly related to
the question.

STRONG ARGUMENTS:
An argument is considered as strong argument in the following cases.
1. When related to development: If an argument is related to development of a
country whether infrastructure development or educational development, etc,
then it is considered as strong argument.
2. Universal truth: If any argument is universally accepted, that cannot be denied
and it is related with the statement, then it is a strong argument.
Ex: Computer is beneficial to us.
3. Analyzed Truth: ​The decision taken by our government/ constitutional body/
united nation or any supreme authority is always welcomed and is deemed as a
strong argument.
Ex: Primary education has been incorporated as fundamental rights in India.
4. Experience based: ​If any argument is on the basis of experience, then it will be
accepted.
Ex: E-commerce should be encouraged in our country.

WEAK ARGUMENTS:
An argument is considered as weak argument in the following cases.
1. Ambiguous:​ The argument which are not clear in meaning and creates doubt or
confusion, then it is considered as weak argument.
Ex: One should eat, drink freely because tomorrow one has to die.
2. Copied Argument: ​If any argument shows that the sentence is immitating or
copying anything then it can’t be accepted.
Ex: M S Dhoni should take retirement from cricket because most of the cricketer
prefer to retire at this age.
3. Opinion Based: ​If there is any personal suggestion, opinion, comments etc.,
then such type of argument can never be accepted.

Plutus Academy-(191)
Ex: Relocating the factories out of the city is the only way to save our city from
the pollution
4. Superfluous: ​The argument that can’t be accepted is known as superfluous
argument.
Ex: Import of foreign books should be banned.
 
Directions to Solve 
Each  question  given  below  consists  of a statement, followed by two arguments numbered I and 
II.  You  have  to  decide  which  of  the  arguments  is  a  ‘strong’  argument  and  which  is  a  ‘weak’ 
argument. 

Give answer: 

(A) If only argument I is strong 

(B) If only argument II is strong 

(C) If either I or II is strong 

(D) If neither I nor II is strong and 

(E) If both I and II are strong. 

1. Statement: Military service should be made compulsory in the country. 

Arguments: I. Yes, every citizen should protect his country. 


II. No, it is against the policy of non-violence. 
 
2. Statement: Non vegetarian foods should be totally banned in our country. 

Arguments: I. Yes,it is expensive and therefore beyond the means of most people in our country. 
II. Nothing should be banned in a democratic country. 
 
3. Statement: Should mercy death be legalised? 

Arguments:  I.  Yes,patients  undergoing  terrible  suffering  and  have  absolutely  no  chance  of 
recovery should be liberated from sufferings through mercy death. 
II. No, even mercy death is a sort of killing and killing can never be legalised. 
 
 

 
 
 
 

Plutus Academy-(192)
Exercise 
Q1. Statement: ​Doctors found that company M is selling drugs without testing the
important aspects of the drugs. Such practice tends the doctor to negate the use of
drug.
Which of the following arguments would strengthen the stance of the Company
M?
(1) The commercial production of any drug is permitted after its repeated trial and
verification of result.
(2) Some other companies are also selling the drugs which are sold by the company
M.
(3) Some people do not agree with the view put forward by the doctors.
(4) Any drug is put on test by the pharmacist. Doctors are not competent to approve
or disapprove any drug.
(5) Doctors promote those drugs which are very costly as they have some share in
the profit.
(IBPS PO/MT CWE-IV 2014)

Directions(2-4) Read the following statements carefully and answer the questions
which follow. (IBPS SO CWE 2013)
Q2. Statement: The ministry of sports has been advised by a committee to take the
highest award in the field of sports back from two players who were allegedly involved in
match fixing.
Which of the following statements would weaken the argument put forward by the
committee to the sports ministry?
1) A good conduct in the past and a lack of evidence against the players make the
case against them very weak.
2) The ministry of sports has never declined the recommendations made by the
committee earlier.
3) Taking the award back from the players would set a good example to other
players for avoiding such actions in the future.
4) There have been past cases where the award had to be taken back from the
players owing to some misconduct later on.
5) The committee is constituted of some of the most respected and esteemed
members from the field of sports and politics.
Q3. Statement: Many organizations have been resorting to recruitment based upon
performance at graduate/post graduate level exams rather than conducting exams for
the same purpose.

Plutus Academy-(193)
Which of the following statements would strengthen the argument given in the
above statement?
1) A recent study shows no link of past performance with the performance in
recruitment exams.
2) The graduate/post graduate exams are considered to be severely deficient in
training in job related environment.
3) Organisations which had undertaken recruitment on the basis of graduate/post
graduate exams report a significant drop in the quality of the recruited
employees.
4) Such policies would add to unemployment among students having below
average performance in graduation or post graduation.
5) Such policies could save time,money and resources of the organisation which
are wasted in the conduct of recruitment examinations.

Q4. Statement: According to a recent government directive, all bank branches in rural
areas should be computerized.
Which of the following statements would weaken the government’s argument?
1) Computerization of bank branches in urban areas has helped in making their
performance more efficient and fast.
2) Lack of skilled and qualified manpower has been suitably substituted by
computers in banks.
3) Non- computerised bank branches in the rural areas have been proved to be as
efficient as their computerized counterparts.
4) The government has introduced a special test for computer knowledge in all
recruitment exams for banks.
5) Unemployment in the rural areas could be controlled by training more and more
professionals in computers.

Q5. Statement : It has been reported in a recent study that intake of moderate quantity
of milk chocolate reduces the risk of suffering from the central nervous system related
illnesses.
Which of the following would weaken the findings of the study reported in the
above statement?
1) People generally prefer to eat chocolate when they are young.
2) Majority of those not diagnosed with diseases related to central nervous system
have stayed away from eating chocolates in their lives.
3) Chocolates contain certain elements which strengthen the functions of the central
nervous system.

Plutus Academy-(194)
4) Majority of those suffering from the central nervous system related diseases are
middle aged.
5) Many of those who suffer from diabetes also suffer from other major ailments.
(IBPS SO CWE 2012)
Q6. ​A very large number of technically qualified young indians are coming out of
colleges every year though there are not enough opportunities for them to get gainful
employment.
Which of the following contradicts the views expressed in the above statement?
1) Technically qualified persons are far superior to those with standard degrees like
BA/B.Sc /B.Com etc.
2) The Government has not done effective perspective planning for engaging
technically qualified personnel while authorising the setting up of technical
colleges.
3) A huge gap exists between the level of competence of technically qualified
graduates and requirement of the industry.
4) Majority of the technically qualified persons are migrating from india to developed
countries for better opportunities.
5) None of these (IBPS PO/MT 2011)

Directions(7-11) In making decision about important questions, it is desirable to


be able to distinguish between “Strong” argument and “Weak” argument.
Give answer (1) if only argument I is strong.
Give answer (2) if only argument II is strong.
Give answer (3) if either argument I or II is strong.
Give answer (4) if neither argument I nor II is strong.
Give answer (5) if both argument I and II are strong. ​(SBI Associates bank PO 2007)

Q7. Statement:​ Should the habit of late coming in educational institutions be checked?
Arguments:
I. No,Until it affects the work.
II. Yes. discipline must be maintained.

Q8. Statement: ​Should seniority be the only criterion for the promotion?
Arguments:
I. No, All the senior employees are not interested in promotion.
II. Yes. Otherwise senior employees do feel humiliated.

Q9. Statement: ​Should children be prevented completely from watching television?


Arguments:

Plutus Academy-(195)
I. No. We get vital information regarding education through television
III. Yes. It hampers the study of children.

Q10. Statement:​ Should trade unions be banned completely?


Arguments:
I. No. This is the only way through which employees can put their demands before
management.
II. Yes. Employees get their illegal demands fulfilled through these unions.

Q11. Statement: ​Should women be given equal opportunity in matter of employment in


every field?
Arguments:
I. Yes. They are equally capable.
II. No. They have to shoulder household responsibilities too.

Q12. Statement​:Many Major automakers will increase the prices of their vehicles by 7
percent from next month, mainly to offset higher input and operational costs.
Which of the following arguments would weaken the facts given in the statement?
Arguments:
1) The largest auto maker in the country, Verontee had failed to make any profit
when prices for its vehicles were increased by almost 4 percent.
2) Operational and input costs continue to be 85% of the total costs of the vehicles.
3) Both input and operational costs were gradually increased by the government to
avoid sudden transit losses to the companies.
4) At any given time, a price rise of more than 5 % definitely leads to substantial
decrease in sales of the vehicles.
5) Rather than increasing profit, 7 % price rise would bring it at the same level
which existed before the rise in input/operational costs.

Answer
1) 1
2) 1
3) 5
4) 3
5) 5
6) 4
7) 2
8) 4

Plutus Academy-(196)
9) 1
10)1
11)1
12)4

Statement  and  argument  questions  asked  in  previous  years(IBPS  PO/SBI 


PO/BOB) 
1.  It  is  not  often  that  professional  footballers  retire  from  internationals  at  the  peak  of  their 
game.  When  the  29-year-old  Lionel  Messi,  widely  reckoned  to  be  the  most  skilled  footballer 
today,  announced  after  the  2016  Copa  America  Final  against  Chile  that  he  would  not  wear  the 
Argentinian  shirt  again,  he  took  everyone  by  surprise.  It  is  not  clear  if  he  has  made  the 
retirement  call  in  the  heat  of  the  moment.  It  came  after  Messi  failed,  yet  again,  to  win  a  major 
title  with  the  Argentinian  team;  he  also  missed  a  crucial  penalty  in  the  shootout  against  Chile 
after  efficient but goal-less play in regulation and extra time. In fact, Messi’s overall international 
record  with  Argentina  has  been  good,  if  not  spectacular.  With  him,  the  team  has  reached  the 
finals  of  four  important  tournaments:  the  World  Cup  in  2014  and  the  Copa  America  in  2007, 
2015 and 2016. 
According  to  the  given  question,  you have to decide which of the statement is/are strong on the 
basis of the given passage:- 
After  defeating  in  Copa  America  Final,  Should  Messi  continues  his  international  career  for  two 
years more? ​(IBPS PO MAINS 2016) 
I. Yes, because it is the only way he can prove himself to the world as a challenging person. 
II.  No,  It  is  his  personal  decision  to  play  no  more  for  Argentina  and  as  a  player  he  gave  a  lot of 
contribution to his nation. So we should respect his decision. 
III.  No,  because  Messi’s  record  as  a  player  of  Barcelona  is  quite  better  than  as  a  player  of 
Argentina. So he should focus on one side rather than playing for both of them. 
(a)Only II & III are strong 
(b) Only I & III are strong 
(c) Only II is strong 
(d) Only III is strong 
(e) None is strong 
2.  The  mushrooming  of  business  schools  in  the  country  is  a  cause  for  shortage  of  faculty with 
Ph.D  qualification.  In  addition,  the higher pay and generous fringe benefits given by industry has 
encouraged  qualified  people  to  not  seek  academic  positions.  Which  of  the  following 
statements, if true, would tend to STRENGTHEN the argument? ​ (SBI PO MAINS 2017) 

Plutus Academy-(197)
(a)  The  average  salary  for  industry  positions  in  Gujarat  is  more  than  the  average  salary  for 
faculty positions in some business schools in Ahmedabad by around 30% 
(b)  The  average salary for industry positions in Gujarat is less than the average salary for faculty 
positions in a top business school in Ahmedabad by around 30% 
(c)  The  average  salary  for  recent  Ph.D  graduates  in  the  industry  is  20%  higher  than  that  in 
academics 
(d)  The  rate  of  growth  of  salaries  for  the  industry  positions  is  equal  to  the  rate  of  growth  of 
salaries for academic positions for the past three years 
(e) None of the above 
3.  The  rate  of  violent  crime  in  this  state  is  increased  up  to  30%  from  last  year.  The  fault  lies 
entirely  in  our  system  of  justice.  Recently  our  judge’s  sentences  have  been  so  lenient  that 
criminals can now do almost anything without fear of a long prison term. ​(SBI PO MAINS 2017) 
The argument above would be weakened if it were true that 
(a) 85% of the other States in the nation have lower crime rates than does this state 
(b) White-collar crime in this state has also increased by over 25% in the last year 
(c) 35% of the police in this state have been laid off in the last year due to budget cuts 
(d) Polls show that 65% of the population in this state opposes capital punishment 
(e) None of the above 
4.  Landmark  preservation  laws  unfairly  impinge  on  the  freedom  of owners to develop their own 
property  as  they  see  fit.  In  some  cases,  owners  of  hotels  and  office  buildings  designated  as 
landmarks  have  been  forbidden  to  make  changes  in  the  original  facades  or  interiors,  even 
though  they  reasonably  believe  that  the  changes  would  enhance  the structures and make them 
more  valuable.  Which  of  the  following  statements,  if  true,  seriously  weakens  the  author’s 
argument? ​(BOB PO 2016) 
(a)  Altering  the  appearance  of  a  historic  structure  sometimes  does  not  enhance  its  beauty  or 
value. 
(b) In traditional legal doctrine, ownership of a property implies the right to alter it at will. 
(c) Only buildings over 75 years old are normally affected by landmark preservation laws. 
(d) Landmark designations must be approved by a local regulatory body before taking effect. 
(e)  Historic  buildings  represent  a  cultural  heritage  which  the  community  has  a  legitimate  stake 
in preserving. 
5.  The  percentage of family income spent on entertainment has remained almost the same over 
the  past  twenty  years  –  about  twelve  per  cent.  When  new  forms  of  entertainment  become 
popular,  they  do  not  expand  this  percentage;  instead,  they  take  consumer  spending  away  from 
other  forms  of  entertainment.  Therefore,  film  producers  have  observed  the  video  boom  with 
concern,  knowing  that  every  dollar  spent on rental of videos means a dollar less spent on movie 

Plutus Academy-(198)
theatre  admissions.  Which  of  the  following,  if  true,  most forcefully undermines the argument of 
the passage above? (BOB PO 2016) 
(a)  The  cost  of  renting  a  video  is  generally  substantially  less  than  the  price  of  a  movie  theatre 
admission. 
(b)  Most  film  producers  receive  a  portion  of  the  income  from  the  sale  of  video  rights  to  their 
movies. 
(c)  Fears  of  some  film  producers  that  videos  would  completely  supersede  movies  have  not 
come to pass. 
(d)  Since  the  start  of  the video boom, money spent on forms of entertainment other than videos 
and movies has dropped. 
(e)  Some  movies  that  were  unprofitable when shown in theatres have become successful when 
released in video form. 
 
 
 
SOLUTIONS: 
1.(c); 
Only  II  is  strong  argument  because  it  clearly  explains  the  freedom which is given to a person to 
take decisions about his life. So it gives a valid reason. 
Argument  I  is  a  weak  statement  because  of  the  ‘only’  word  used  in  the  statement.  Messi  can 
prove himself by all other ways also. 
Argument  III  is  also  a  weak  Statement  because  there  is  a  comparison  between  the  position  of 
Messi as a player of Argentina and the position of Messi as a player of Barcelona club. 
2. Ans.(a) 
Statement  (a)  is  the correct option because it clearly strengthen the argument that persons with 
sufficient  qualification  prefer  to  go  to  industrial field rather than to go in academics as a faculty 
in business schools. 
3. Ans.(c) 
Statement  (c)  is  the  correct  option  because  according  to  this,  the  increase  in  crime  rate  has 
been  contributed  by  other  factors,  not  leniency  in  the  punishment.so  it  weakens  the  given 
passage. 
4. (e); 
The  conclusion  of  the  argument  is  that  landmark  preservation  laws  deprive  landlords  of  their 
right  to  use  their  own  property.  (e)  comes  to  grips  with  this  assumption  by  nothing  that  a 
landmark  building  may  not  be  purely  private  property  and  some  part  of  building  may  belong  to 

Plutus Academy-(199)
the  community  at  large.  (a)  does  not  valid  as  it  represents only a partial attack on argument.(b) 
strengthens  the  claims  that  landmark  preservation  laws  represent  an  unwanted  interference 
with the rights of the landlord. 
5. (d); 
The  author  argues  for  the  following  connection:  videos  take  money  away  from  movies.  What 
choices  (d)  asserts,  in  effect,  is  that  the  money  spent  on  videos  came  from  some  other 
source.so,(d) statement undermines the given passage 
 
 
 
 
 
 
 
 
 
 
 
 
 
 
 
 
 
 
 
 
 
 
 
 
 
 
 
 
 
 
 
 
 
 

Plutus Academy-(200)
Statement and Conclusion/Inferences 
A ​statement​ is a group of words arranged to form a meaningful sentence. 
A ​conclusion​ is a judgement or decision reached after consideration of the given statement.  
An  ​inference  is  something  that  is  not  directly  stated  but  can  be  inferred  from  the  given 
information. 

In  this  type  of  questions,  you  have  to  choose  correct  or  appropriate  conclusion  or  inferences 
based  on  the  given  problem  after  analysing  the  whole  conditions  given  for  that  problem.This 
requires a very logical approach. 

KEY POINTS: 
1. If  statement  is  formed  with  two  or  more  sentences,  then  there  should  be  no  mutual
contradiction in sentence.
2. Statement  and  conclusion  should  not  go  against  established  facts  and  prevailing
notions of truth.
3. If  words  of  perfection  like  all, always, atleast, only, exactly and so on are used, then such
words make the conclusion ambiguous.
4. Always try to find keywords as they enhance our analysis.
5. If the conclusion is provided with a stated example, then conclusion is invalid.

Directions to Solve 
In  each question below is given a statement followed by two conclusions numbered I and II. You 
have  to  assume  everything  in  the  statement  to  be  true,  then  consider  the  two  conclusions 
together  and  decide  which  of  them  logically  follows  beyond  a  reasonable  doubt  from  the 
information given in the statement. 
Give answer: 
(A) If only conclusion I follows
(B) If only conclusion II follows
(C) If either I or II follows
(D) If neither I nor II follows and
(E) If both I and II follow.

1. Statement:  Sealed  tenders  are  invited  from  competent  contractors  experienced  in  executing
construction job.
Conclusion: I. Tenders are invited only from experienced contractors.
II. It is difficult to find competent tenderers in construction jobs.

2. Statement: The boss humiliated Rahul in the presence of his colleagues.


Conclusion: I. The boss did not like Rahul.
II. Rahul was not popular with his colleagues.

Plutus Academy-(201)
3.  Statement:  The  distance  of  600  km  by  road between Mumbai and Goa will be reduced to 400 
km by sea. This will lead to saving of Rs 5.96 crore per annum on fuel. 
Conclusion: I. Transportation by sea is cheaper than that by road. 
II. Fuel must be saved to a great extent. 
 
4.  Statement:  Mr Z is one of the probable candidate to be shortlisted for the post of dean of XYZ 
Institute. 
Conclusion: I. Mr Z will be selected as the dean of XYZ Institute. 
II. Mr Z will not be selected as the dean of XYZ Institute. 
 
5.  Statement:  Research  has  proved  that  people  eating  high  fat  diets  coupled  with  decreased 
level of exercises are prone to heart disease. 
Conclusion: I. People should reduce their high fat diet as a preventive measure. 
  II.  People  must  have  sufficient  level  of  exercise  to  reduce  their  chances  of  having a 
heart disease. 
 
 
EXERCISE 
Directions(1-5):  ​Below  is  given  a passage followed by several possible inferences which can be 
drawn  from  the  facts  stated  in  the  passage.  You  have  to  examine  each  inference  separately  in 
the context of the passage and decide upon its degree of truth or falsity. 
(IBPS SO CWE 2013) 
Mark  Answer  (1)  ​If  the  inference is “Definitely true” i.e. it properly follows from the statement of 
facts given. 
Mark  Answer  (2)  If  the  inference  is  “Probably  true” though not “definitely true” in the light of the 
facts given. 
Mark  Answer  (3)  If  the  data  is  inadequate  i.e  from  the  facts  given , you cannot say whether the 
inference is likely to be true or false. 
Mark  Answer  (4)  If  the  inference  is  “Probably  false”  though  not  “definitely  false”  in  the  light  of 
the facts given. 
Mark  answer  (5)  If  the  inference  is  “Definitely  false”  i.e.  it  cannot  possibly  be  drawn  from  the 
facts given or it contradicts the given facts. 
(Note:  Each  of  the  five  questions  has  only  one  distinct  answer  i.e.  no  two  questions  can have 
the  same  answer.  If  you  get  the same answer for more than one question, consider both again 
and  decide  which  one  of  the  two  would  more  definitely  be  that  answer  and  in  the  same  way 
review the other also.) 
Cardiovascular  disease  is  so  prevalent  that  virtually  all  businesses are likely to have employees 
who  suffer  from  ,  or  may  develop,  this  condition.  Research  shows  that  between  50-80  per  cent 
of  all  people  who  suffer  a  heart  attack  are  able  to  return  to  work.  However,  this  may  not  be 
possible  if  they  have  previously  been  involved  in  heavy  physical  work.  In  such  cases,  it  may  be 
possible  to  move  the  employee  to  lighter  duties,  with  appropriate  retraining  where  necessary. 

Plutus Academy-(202)
Similarly,  high  pressure  ,  stressful  work,  even  where  it  does  not involve physical, should also be 
avoided.  Human  Resource  managers  should  be  aware  of  the  implications  of  job  roles  for 
employees with a cardiac condition. 
Q1. Employee who suffer from cardiovascular disease are mostly unable to return to work. 
Q2. Employee who suffer from cardiovascular disease are unable to handle stressful situations. 
Q3.Employees above the age of 50 are found to suffer from cardiovascular disease. 
Q4. Physical and stressful work definitely leads to heart attack. 
Q5. Heart disease can affect employees in any type of business. 
 
Q6.  Statement  :  ​Expert  define  the social entrepreneurship as such that pursues opportunities to 
serve  the  mission  to  give  solutions  to  social  problems.  Business  entrepreneurs  typically 
measure performance in profit and return. (IBPS PO 2013) 
Which of the following can be concluded from the above statements? 
1) It is possible to address social problem while making business profitable. 
2) Social entrepreneurship is different from the business entrepreneurship. 
3) Business  entrepreneurship  does  not  care  social  problems  while  devising  their  business 
policy. 
4) Business entrepreneurship is more important that social entrepreneurship. 
5) None of these 
 
Q7.  Statement:  A  few  travellers  were  severely  beaten  by villagers recently in a remote rural part 
of  the  state  as  the  villagers  found  the  movement  of  the  travellers  suspicious.  The  district 
authority has sent a police team to nab the culprits. (IBPS PO 2011) 
Which of the following inference can be drawn from the above statement? 
1) The villagers dislike presence of strangers in their vicinity. 
2) Villagers are Generally suspicious in nature. 
3) Travellers prefer to visit countryside. 
4) The Government generally provides protection to travellers across the country. 
5) None of these 
 
Q8.​Read the following information carefully and answer the questions which follow: 
Small  brands  are  now  looking  beyond  grocery  stores and are tying up with supermarkets such 
as big bazaar to pull their business out of troubled waters. 
Which  of  the  following  can  be  inferred  from  the given information?(An inference is something 
that is not directly stated but can be inferred from the given information) 
(IBPS PO 2012) 
1) Merchandise  of  smaller  brands  would  not be available at local grocery stores in the near 
future. 
2) Smaller brands cannot compete with bigger ones in a supermarket set-up. 
3) There  is  a  perception  among  small  brands  that  sale  in a supermarket is higher than that 
of small grocery stores. 
4) Supermarkets  generate  more  revenue  by  selling products of bigger brands as compared 
to the smaller ones. 

Plutus Academy-(203)
5) Smaller  brands  have  always  had  more  tie-ups  with  supermarkets  as  compared  to  small 
grocery stores. 
Q9.  Statement  :  Always  remain  in  an  air  conditioned  environment  for  better  health  and  well 
being. (SBI PO 2014) 
I. Filters in the air- conditioners lower exposure to allergy creating pollen and other outdoor 
allergies. 
II. Spending  too  much  time  in  an  air-conditioned  environment  result  in  gradual  intolerance 
of body towards natural temperatures. 
1) Statement I weakens the information while statements II is a natural statement. 
2) Both statements I and II weaken the information. 
3) Statement I strengthens the information while statement II weakens the statement. 
4) Statement I weakens the information while statement II strengthens the statement. 
5) Both statements I and II strengthen the information. 
 
Q10.  Expert  A  says  that  dinosaurs  became  extinct  due  to  climatic  changes  occurred  on  the 
earth due to volcanic eruptions some 65 million years ago. 
Expert  B  does  not  agree  with  the  volcanic  eruption  theory.  According  to him dinosaurs became 
extinct due to the impact of asteroid. 
Which of the following statements may provide support to the theory propounded by expert B? 
A) The frigid and sweltering climatic extremes caused the extinction of dinosaurs. 
B) A  wide  crater  lying  just  of  the  yucatan  peninsula  was  created  due  to  the  impact  of 
asteroid. 
C) Scientists  have  discovered  levels  of  iridium  30  times  greater  than  average  in  the 
Cretaceous/  Tertiary boundary, the layer of sedimentary rock laid down at the time of the 
dinosaur extinction. 
D) Some  palaeontologists  after  analysing  the  fossil  record  believe  that  dinosaurs  were 
doing quite well prior to the end of the cretaceous, when the dinosaurs became extinct. 
1) Only A 
2) Only A and C 
3) Only C and D 
4) Only A and B 
5) Only B, C and D (RBI Grade-B 2014) 
 
 
 
 
 
 
 
 
 
 
 

Plutus Academy-(204)
Answer Key- 
1) 5 
2) 2 
3) 3 
4) 4 
5) 1 
6) 2 
7) 4 
8) 3 
9) 3 
10) 5 
 
 
 
 
 
 
 
 
 
 
 
 
 
 
 
 
 
 
 
 
 
 
 
 
 
 
 
 
 
 
 
 

Plutus Academy-(205)
Course of Action 
A ​statement​ means something that is said or expressed by someone. 
A  ​course  of  action  ​is a step to be taken for improvement of situation or leading to further action 
with regards to the problem, on the basis of the information given in the statement. 

This type of question comes in logical reasoning. In each question a statement is given followed 
by  2  or  more  course  of  action.  After  reading  and  analysing  the  statement,  the  candidate has to 
take  decision  to  solve  the  problem  or  for  improvement  or  for  follow-up  or  for  further  action  or 
for taking care of solution or for immediate action. 
We  should  remember  below  terms  when  we  take  decision  or  choose  appropriate  course  of 
action in respect of statement. 

● The  decision  should  either  lessen  the  problem  or  should  improve  upon  the  present
situation.
● Simple  problem  must have simple course of action and not a complex one so that it may
solve or reduce the problem and not create more issue.
● The decision should not be harsh(Termination,suspension).
● The solution should be viable(practicable/workable/Feasible).
● The  decision  must  give  immediate  solution  for  the  problem  means  time  taking  solution
should be avoided.
● The solution should not break the service.
● The solution should not break the hierarchy.
● The solution which does not give surety about the solving of problem must be avoided.
● The government related things will be true.
● The solution which break independence will be wrong.
● Decisions or solutions which were successful in past will be true.
● The decisions which creates an example will be true.
● Initiative and regulative decisions will be true.
● The solution should not break the service.
● The solution which bring loses will be wrong.

Type 1 
Q1.  There  is  an  alarming  trend  of  skewed  sex  ration  against  women  in  india  during  the  past 
decade  and  situation  may  get  out  of  hand  if  adequate  decisions  or  steps  are  not  taken  to stop 
female foeticide. 
Which of the following can be an effective decision or solution to reverse the trend? 
(A) The  government  should  completely  ban  use  of  scanners/sonography  on  expectant
mothers at all health centers immediately .
(B) The  government  should  announce  a  substantial  incentive  scheme for couples who have
at least one girl child.

Plutus Academy-(206)
(C) The  government  should  launch  a  nationwide  campaign  to  create  awareness  against 
female foeticide. (IBPS PO 2011) 
(1) A only  
(2) A and B only 
(3) B and C only 
(4) All A ,B and C 
(5) None of these 
 
Q2.  Statement-  ​The  prices  of  vegetables  and  other  food  articles  have  decreased  in  the  recent 
months  raising  hope  among  policy  planners  that  the  RBI’s  tight  grip  on  supply  of liquid money 
in the market for controlling inflation may be eased. 
Which of the following may be a possible action or decision of the above situation?    
(IBPS PO 2012) 
(1) The reserve bank of india may not reduce its key interest rates in near future.  
(2) The  government  may  step  in  and  make  certain  concessions  to  the  common  people  on 
various issues. 
(3) The  reserve bank of india may consider lowering certain rates to bring in more liquidity in 
the market. 
(4) The RBI may wait for at least another year before taking any step. 
(5) The  RBI  may  collect  more  data  from  the  market  and  wait  for  another  four  months  to 
ensure they take the correct step. 
 
Type 2 
Q1.  Directions(1-5):  In  each  question  below  is  given  a  statement  followed  by  two  courses  of 
action  numbered  I  and  II.  A  course  of  action  is  a  step  or administrative decision to be taken for 
improvement,  follow-up  of  further  action  in  regard  of the problem, policy etc.On the basis of the 
information  given  in  the  statement,  you  have  to  assume  everything  in  the  statement  to  be  true, 
then decide which of the suggested courses of action logically follows for pursuing. 
Give answer (1) if only course of the action I follows. 
Give answer (2) if only course of action II follows 
Give answer (3) if either course of action I or II follows. 
Give answer (4) if neither course of action I nor II follows. 
Give answer (5) if both course of action I and II follow. (SBI ME Exam-2014) 
 
Q1.  Statement:  Drinking  water  supply  to  many  parts  of  town  is  disputed  due  to  loss  of  water 
because of leakage in pipe supplying water. 
Courses of action: 
I The government should order an enquiry into the matter. 
II  The  Civic  body  should  set  up  a  fact  finding  team  to  assess  the  damage  and  take  effective 
step. 
 
 

Plutus Academy-(207)
Q2  Statement:  ​There  is  an  alarming  increase  in  the  number  of  people suffering from malaria in 
many parts of the city. 
Courses of action: 
I  The  municipal  corporation  has  advised  all  the  government  hospital  to  store  adequate  supply 
of Malaria drugs. 
II  The  Municipal  Corporation  has  urged  people  to  use  mosquito  repellants  and  keep  their 
premises clean. 
 
Q3.Statement:  Many  people  have  encroached  into  the  government  property  and  built  their 
houses and business establishments. 
Courses of action: 
I  The  government  should  take  immediate  steps  to  remove  all  unauthorised  constructions  on 
government land. 
II  All  the  encroachers  should  immediately  be  put  behind  bars  and  also  be  slapped  with  a  hefty 
fine. 
 
Q4.Statement:  The  Meteorological  Department  has  predicted  normal  rainfall  throughout  the 
country during the current monsoon.  
Course of action: 
I The government should reduce the procurement price of food grains for the current year. 
II The government should reduce subsidy on fertilizers for the current year. 
 
Q5.Statement:  The  number  of  dropouts  in  Government  schools  have  significantly  increased  in 
the urban areas over the past few years. 
Courses of action: 
I  The  government  should  immediately  close  down  all  the  schools  in  the urban areas where the 
dropout goes beyond 20%. 
II  The  parents  of  all  students  who  dropped  out  of  the  government  schools  in  the  urban  areas 
should be punished. 
 
Directions(6-10): ​In each question below is given a statement followed by two courses of action 
number  I  and  II.  A  course  of  action  is  a  step  or  administrative  decision  to  be  taken  for 
improvement,  follow-up  or  further  action  in  regard  of  problem  policy  etc.  On  the  basis  of  the 
information  given  in  the  statement,  you  have  to  assume  everything  in  the  statement  to  be  true, 
then decide which of the suggested courses of action logically follows for pursuing. 
mark answer 1 if only course of action I follow. 
mark answer 2 if only course of action II follows. 
mark answer 3 if either course of action I or course of action II follows. 
mark answer 4 if neither course of action I or course of action II follows. 
Mark answer 5 if both course of action I and courses action II follow.  
(BOI banking officer 2010) 
 

Plutus Academy-(208)
Q6.  Statement:  Many school children died in few accidents caused due to poor maintenance of 
school buses during the last few months. 
Courses of action: 
I.   The  government  should  set  up an expert group to inspect the condition of school buses 
to avoid such accidents. 
II. The  government  should  suspend  the  licence  of  all  school  buses  which  are  not  properly 
checked. 
 
Q7.  Statement  :  Many  customers  complained  to  the  manager  of  the  local  branch  of  the  bank 
against high handedness of the bank employees while dealing with the customers. 
Courses of action: 
I.   The  Bank  management  should  immediately  suspend  all  them  employees  of  the branch 
and deploy a different set of employees. 
II.   The  Bank  management  should  ask  for  a  report  from  the  manager  of  the  branch  for 
taking necessary steps. 
 
Q8.  Statement:    Two  persons,  while  on  their  daily  walks  in  the  Jogger’s  Park  were  killed  by 
unidentified miscreants early in the morning. 
Courses of action: 
I.   The  police  authorities  should  deploying  police  constables  near  the  Joggers  Park  to 
prevent search criminal acts in future. 
II.   The  citizens  of  the  locality  should  go  for  early  morning  off  in  groups  to  avoid  search 
attacks. 
 
Q9.  Statement:  A  huge  oil  tanker  overturned  on  the  highway  leading  to  the  state  capital  and 
blocked most part of the highway from vehicular traffic. 
Course of action: 
I.   The  government  should  immediately  constitute  a  high-level  enquiry  committee  to  look 
into the matter. 
II. The driver of the old tanker should immediately be arrested. 
 
Q10.  Statement:  ​A  very  large  number  of  recently recruited officers left the bank after attending 
the training program on branch banking operations conducted by the bank. 
Courses of action: 
I. The  bank  should  take  a  bond  of  at  least  2  years  from  each  newly  recruited  officers 
before putting them into the training programme. 
II. The  bank  should  conduct  the training program at least one year after the newly recruited 
officers joined the bank. 
 
Directions  (11-12):  In  each  question  below  are  given  a  statement  followed  by  two  courses  of 
action  numbered  I  and  II.  You  have  to  assume  everything in the statement to be true and on the 
basis  of  the  information  given  in  the  statement,  decide  which  of  the  suggested  courses  of 
action logically follow(s) for pursuing. ​ (SBI PO MAINS 2017) 

Plutus Academy-(209)
Give answer  
(a) If only I follows;  
(b) If only II follows,  
(c) If either I or II follows;  
(d) If neither I nor II follows and  
(e) If both I and II follow. 
  
Q11.  Statement​:  Every  year,  at  the  beginning  or  at  the  end  of  the  monsoons,  we  have  some 
cases  of  conjunctivitis,  but  this  year,  it  seems  to  be  a  major  epidemic,  witnessed  after  nearly 
four years.  
Courses of action:  
I. Precautionary measures should be taken after every four years to check this epidemic.  
II. People should be advised to drink boiled water during rainy season.  
 
Q12.  ​Statement:  Researchers  are  feeling  agitated  as  libraries  are  not  equipped  to  provide  the 
right  information  to  the  right  users  at  the  right  time  in  the  required  format.  Even  the  users  are 
not aware about the various services available for them.  
Courses of action:  
I.  All  the  information  available  to  the libraries should be computerized to provide faster services 
to the users.  
II. Library staff should be trained in computer operations.  
 
 
 
 
 
 
 
 
 
 
 
 
 
 
 
 
 
 
 
 
 

Plutus Academy-(210)
Answer Keys 
Type 1 
1. 3 
2. 3 
Type 2 
1. 4 
2. 5 
3. 1 
4. 4 
5. 4 
6. 1 
7. 2 
8. 5 
9. 4 
10. 1 
11. Ans.(b) Sol. The disease occurs at the end of monsoons every year. So, precautionary
measures every four years shall not help. The second course of action shall be a
preventive measure. So, only course II follows.
12. Ans.(e) Sol. Clearly, the library needs to be provided with the essential facilities and
trained personnel for better services. So, both the courses follow.

For video solutions: ​https://www.youtube.com/watch?v=H6Ct1pvwUx4


 
 
 
 
 
 
 
 
 
 
 
 
 
 
 
 
 

Plutus Academy-(211)
DECISION MAKING 
 
Q1.​ Study the following information carefully and answer the questions given below- 
(IBPS SO CWE 2012) 
Following are the conditions for selecting Manger-HR in an organisation: 
The candidate must- 
(i) be at least 30 years and not more than 35 years as on 1/3/12. 
(ii) have secured at least 60 percent marks in graduation in any discipline. 
(iii) have secured at least 65 % marks in the post graduate degree/diploma in personnel 
Management / HR. 
(iv) have post qualification work experience of at least five years in the personnel/HR 
department of an organisation. 
(v) Have secured at least 50 % marks in the selection process. 
In the case of a candidate who satisfies all the above conditions except- 
(a) At (ii) above, but has secured at least 55 % marks in graduation in any discipline and at 
least 70 % marks in post graduate degree/Diploma in personnel Management/HR, the 
case is to be referred to GM- HR. 
(b) At (iv) above, but has post qualification work experience of at least four years out of 
which at least two years as Deputy Manager-HR. The case is to be referred to President- 
HR. 
In each question below are given details of one candidate. You have to take one of the following 
courses of actions based on the information provided and the conditions and sub conditions 
given above and mark the number of that course of action as your answer. You are not to 
assume anything other than the information provided in each question. All these cases are given 
to as on 1/3/2012. 
Mark answer(1)-​ If the candidate is not selected. 
Mark answer(2)-​ If the data provided are not adequate to take a decision. 
Mark answer(3)-​ If the case is to be referred to president-HR. 
Mark answer(4)-​ If the case is to be referred to GM-HR. 
Mark answer(5)- ​If the candidates is to be selected. 
 
Q1. Rita Bhatt was born on 25th July 1978. She has secured 62% marks in Graduation and 65% 
marks in Post Graduation diploma in management. She has been working for the past six years 
in the personnel department of an organisation after completing her post graduation. She has 
secured 55% marks in the selection process. 
 
Q2. Ashok Pradhan was born on 8th August 1980. He has been working in the personnel 
Department of an organisation for the past four years after completing his post graduate with 
67 %. Out of his entire experience, he has been working for the past two years as deputy 
Manager-HR. He has secured 62% marks in graduation and 58 percent marks in the selection 
process. 
 

Plutus Academy-(212)
Q3. Alok Verma was born on 4th March 1976. He has been working in personnel department of 
an organisation for the past 6 years after completing his post graduate diploma in personnel 
management with 66% marks. He has secured 7% marks in the selection process and 63% 
marks in graduation. 
 
Q4. Satish has been working in the personnel department of an organisation for the past 5 years 
after completing his postgraduate degree in HR with 72% marks. He has secured 56% marks in 
graduation. He was born on 22nd May 1977 . He has secured 58% marks in the selection 
process. 
 
Q5. Seema Mahal has been working in the personnel department of an organisation for the past 
7 years after completing her post graduation in personnel management with 70% marks.She 
was born on 5th July 1979 . She has scored 65% marks in graduation and 50% marks in the 
selection process. 
 
Q6. Study the following information carefully and answer the questions given below.
(SBI PO MAINS 2017)
Following are the conditions for selecting Marketing Manager in an organization. The Candidate
must-
(i) Be a Graduate in any discipline with at least 55% marks.
(ii) Have secured at least 40% marks in the selection interview.
(iii) Have post qualification work experience of at least five years in the Marketing division of an
organization.
(iv) Have secured at least 45% marks in the selection examination.
(v) Have a post Graduate degree/diploma in Marketing-Management with at least 60% marks.
Study the following information carefully and find which of the following condition shows
candidate is not selected?
(a) Candidate is daughter of a renowned freedom fighter from another state.
(b) Candidate has a post Graduate degree in Finance with 60% marks.
(c) Candidate has completed his graduation with 80% marks.
(d) Candidate does not own a house in Noida.
(e) Candidate has secured 56% marks in Cap Gemini’s interview.

ANSWERS 
1. Answer-5 
2. Answer-3 
3. Answer-1 
4. Answer-4 
5. Answer-5 
6. Ans.(b) 

Plutus Academy-(213)
Cause & Effect 
 
Directions(1-4):​ Study the following information and statements given below it carefully and 
answer the questions which follow: (IBPS PO/MT CWE-IV 2014) 
 
Advertisements play an important role in the sell of a product. Advertisements help consumer to 
decide which product he/she should buy . In the absence of advertisements consumer has very 
little choices to buy the products of his/her needs. 
(A) Advertisements highlight only the bright side of the product. They do not reveal the weak 
features of the product. 
(B) Most of the people wish that companies should stop issuing advertisements as these 
attract people and they start using the glamorous articles. 
(C) The increased frequency of advertisements attracts more different classes of people to 
buy the products. 
(D) The products which are heavily advertised attract a large number of people and they 
start buying the products. For example, a large number of people buy potato chips. 
(E) Consumers lose faith in the advertisement when they do not find the products upto 
mark. 
Q1. Which of the statements numbered (A), (B), (C), (D), and (E) mentioned above represents an 
effect of the given information most appropriately? 
(1) Only A 
(2) Only B 
(3) Only C 
(4) Only C and D 
(5) Only D and E 
 
Q2. Which of the statements numbered (A), (B), (C), (D), and (E) mentioned above would 
weaken/contradict the facts presented in the paragraph? 
(1) Only A and B 
(2) Only B and C 
(3) Only C and D 
(4) Only D 
(5) Only E 
 
Q3. Which of the statements numbered (A), (B), (C), (D), and (E) mentioned above represents a 
cause for issuing advertisements? 
(1) Only A 
(2) Only C and D 
(3) Only B 
(4) Only B and E 
(5) Only A and E 
 

Plutus Academy-(214)
Q4. Which of the statements numbered (A), (B), (C), (D), and (E) mentioned above highlights the 
limitation of an advertisement? 
(1) Only D 
(2) Only E 
(3) Only A 
(4) Only B and E 
(5) Only C and E 
 
Directions(5-9): ​Below in each question are given two statements (A) and (B). These 
statements may be either independent causes or may be effects of independent causes or a 
common cause. One of these statements may be the effect of the other statement. Read both 
the statements and decide which of the following answer choices correctly depicts the 
relationship between these two statements. (SBI PO 2014) 
Mark answer(1) if statement (A) is the cause and statement (B) is its effect. 
Mark answer(2) if statement (B) is the cause and statement (A) is its effect. 
Mark answer(3) if both the statements (A) and (B) are independent causes. 
Mark answer(4) if both the statements (A) and (B) are effects of independent causes. 
Mark answer(5) if both the statements(A) and (B) are effects of some common cause 
 
Q5.(A) Municipal corporation of a city has started providing meals at the school to support the 
education of a slum children. it also started providing books free of cost to the children so that 
it could help them to study and monetary issue do not hamper their education. 
(B) In order to help slum children in their study the prices of books have been reduced by 40% . 
The prices of uniform have also been reduced by 50%. 
 
Q6.(A) A legal notice was issued by the high court in the city A. According to this notice the slum 
area in the city A should not be removed. The legal notice supports the slum area in city A. The 
slum area should be provided with better facilities. 
(B) Builders of the city A have increased the rate of residential area in the city by 40% The 
residential flats have become more costly than earlier. 
 
Q7.(A)A Hospital has lost the case against its patient. The patient has charged that by eating 
food from hospital he has got down by the food poisoning. 
(B) Hospital management has become strict in bringing food from outside the hospital premise. 
The food made in the canteen will only be served inside the hospital premise. 
 
Q8(A)The temperature of the city was lowest during the week. The city recorded low 
temperature of the decade. The city was also engulfed in fog. 
(B) There was unusual delay in the flights from the city and so passengers were stranded at the 
airport. The passengers faced many problems at the airport. 
 
Q9(A) The roads of city A get flooded with vehicles during the evening hour and major parts of 
city face the problem of traffic jam. 

Plutus Academy-(215)
(B) The level of air pollution has been increased in the city A these days. 
 
Q10.Effect- ​The temple at the religious site wears a deserted look with the number of devotees 
trickling down. 
which of the following can be a possible cause of the above effect? 
(1) A structural engineer had visited the temple a month back and had declared the 
structure unsafe. 
(2) The temple is facing a drastic depletion of its funds which had accumulated over the 
years due to offerings made by devotees 
(3) The local Corporation decided to denote a huge amount of money to the temple for its 
renovation. 
(4) The village housing the religious site has qualified priests to perform religious 
ceremonies. 
(5) A famous actor recently visited the temple and paid his respects to the deity.  
(IBPS SO CWE 2013) 
 
Q11. Effect: ​As a step to regulate private hospitals,the State Health Department is framing rules 
to ensure all such hospitals are registered with it. which of the following can be a possible 
cause of the above statement? 
(1) The department realised the private hospitals charge much less for treatment as 
compared to government hospitals. 
(2) Government run hospitals do not maintain the same standards as private hospitals. 
(3) The department realised that several hospitals were rejecting cases is stating lack of 
infrastructure. 
(4) Apart from the number of doctors, nurses and beds, the kind of procedures a hospital 
can carry out based on its infrastructure will also be registered and detailed. 
(5) Private hospitals not registering with the department shall be forced to do so and will 
have to pay hefty penalties  (IBPS SO CWE 2013) 
 
Q12. Statement: ​The income tax authorities carried out raid at 3 different business houses in 
the City last week. 
which of the following can be a possibility of the above statement? 
(1) The three business houses are regular defaulters in payment of their income tax. 
(2) Income tax department had received the tip of about illegal activities going on in the 
three business houses. 
(3) The government decided to look into the matter and has appointed an inquiry 
committee. 
(4) Other business houses took immediate action to clear of all their income tax dues in 
order to avoid a raid on their establishments 
(5) The authorities intend to conduct raids in several other business houses in the vicinity.
(IBPS SO CWE 2013) 
 

Plutus Academy-(216)
Q13.​ ​Statement: ​The college has finally received accreditation and has gained the status of a 
deemed university. 
which of the following can be a possible effect of the above statement? 
(1) The principal of the college well now have to be a retired government official. 
(2) Number of students seeking admission to this college in the next academic year would 
drop significantly. 
(3) The college will charge lesser fees from all students despite not getting a grant from the 
government. 
(4) The college will reduce the number of courses that it runs by significant margin. 
(5) The reputation of college amongst the student population in general has improved. 
 
Q14. Statement:​ According to the survey innumerable diseases and unhealthy habits from the 
recent pandemic to cancer, AIDS and obesity have plagued the mankind. 
Which of the following may be the most plausible cause of the situation given in the above 
statements? 
(1) The lifestyle and food habits of people are responsible for this State. 
(2) Environmental degradation has led to the present situation. 
(3) The people remain ignorant above their health. 
(4) Technological advancement and fast pace of life are contributing to the spread of 
diseases. 
(5) none of these (IBPS PO/MT CWE-III 2013) 
 
 
 
 
Answer Key 
1) 4 
2) 5 
3) 2 
4) 3 
5) 5 
6) 2 
7) 1 
8) 1 
9) 1 
10) 1 
11) 3 
12) 4 
13) 5 
14) 1 
 
 
For video solutions: ​https://www.youtube.com/watch?v=tARakLOhZPg 

Plutus Academy-(217)
Combined Reasoning 
Q1​.All  the members of the Student Rights Coalition signed the petition calling for a meeting with 
the  university  trustees.  Philip  must  be  a  member  of  the  Student  Rights  Coalition  since  his 
signature  appears  on  the  petition.  Which  of  the  following  best  states  the  central  flaw  in  the 
reasoning above?​(BOB PRELIMS 2016) 
(a)  Some  members  of  the  Student  Rights  Coalition  may  not  support  all  of  the  organization’s 
positions.  
(b)  It  is  possible  that  Philip’s  signature  on  the  petition  was  forged  by  a  member  of  the  Student 
Rights Coalition.  
(c)  Any  member  of  the  student  body  is  eligible  to  sign  a  petition  dealing  with  university affairs. 
(d) Philip may have resigned from the Student Rights Coalition after signing the petition.  
(e) Some of those who signed the petition may not be members of the Student Rights Coalition. 
 
Q2.  ​Directions  (2-3):  Read  the following passage carefully and answer the questions that follow: 
The  Budget  session  began  on  a  stormy  note  as  opposition  members  demanding  a  debate  on 
the  recent  hate crimes against Indians living in the USA. Congress blamed the Modi government 
for  remaining  silent  on  the  attacks  against  Indians  in  US. In his reply, Union Home Minister said 
that  the  government  has  taken  a  serious  note  and  PM  will  reply  in  next  week  of  Parliament 
session.  In  recent  weeks,  at  least  two  Indians  have  been  killed  in  incidents  of  hate  crime in the 
US.  “Each  attack  involved  a  slogan.  Go  back  to  your  country”.  America  condemned  it  but is not 
taking any steps to address the issue. ​(SBI PO MAINS 2017) 
 
Q2. ​What may be the repercussions after recent attacks on Indians in US?  
(I) There may be impact on Indian-US trade policy.  
(II) Indian Government will coerce US Government.  
(III) American Govt. will give life imprisonment to accused to make a set mark that America is 
against racism.  
(a) Only I  
(b) Only I and III  
(c) Only III  
(d) Only I and II  
(e) None of these  
 
Q3. Which of the following substantiates the laxity of Indian-American Govt.? (SBI PO MAINS 
2017) 
(I) Modi govt. has taken a serious note and ready to reply in next week of parliament session.  
(II) Condemning the attack but not taking any steps by American Govt.  
(III) Still there is no action as racism slogan “Go back to your country” is spreading.  
 
(a) Only III (b) Only I and II (c) All of three (d) Only II (e) None of these  
 

Plutus Academy-(218)
Q4. During the SARS days, about 23,500 doctors who had treated SARS sufferers died and about 
23,670  doctors  who  had  not  engaged  in  treatment  for  SARS  sufferers  died.  On  the  basis  of 
those  figures,  it  can  be  concluded  that  it  was not much more figures, it can be concluded that it 
was  not  much  more  dangerous  to  participate  in  SARS  treatment  during  the  SARS  day  than  it 
was not to participate in SARS treatment. ​ (SBI PO MAINS 2017) 
 
Which of the following would reveal most clearly the absurdity of the conclusion drawn above?  
(a)  Counting  deaths  among  doctors  who  had  participated  in  SARS  treatment  in  addition  to 
addition to deaths among doctors who had not participated is SARS treatment  
(b)  Expressing  the  difference  between  the  numbers  of  deaths  among  doctors  who  had  treated 
SARS  sufferers  and  doctors  who  had  not  treated  SARS  suffers  as  a  percentage  of  the  total 
number of deaths  
(c)  Separating  deaths  caused  by  accidents  during  the  treatment  to  SARS  suffers  from  deaths 
caused by infect of SARS suffers  
(d)  Comparing  death  rates  per  thousand  members  of  each  group  rather  than  comparing  total 
numbers of deaths  
(e) None of the above  
  
Directions  (5-7):  Study  the  information  carefully  answers  the  questions  given  below.  @  means 
either  hour  hand  or  minute  hand  is  at  8  #  means  either  hour  hand  or  minute  hand  is  at  5  $ 
means either hour hand or minute hand is at 4 % means either hour hand or minute hand is at 12 
&  means  either  hour  hand  or  minute  hand  is  at  2 £ means either hour hand or minute hand is at 
3  Note:  if  two  symbols  are  given  than  by  default  first  symbol  is  consider  as  hour  hand  and 
second one is considered as minute hand. And all time are consider at PM.  

​(SBI PO MAINS 2017) 


Q5.  If  A  takes  25  min  to  reach  railway  station and his train is scheduled at #& then at what time 
should he leave to reach the station 5 minute earlier?  
(a) $% (b) $& (c) &S (d) $@ (e) £$  
 
Q6.  If  a  train  departed  from  a  station  at  &£  and  it  takes  2  hours  to  reach  the  destination  then 
when it will reach to the destination?  
(a) $£ (b) $% (c) #$ (d) $# (e) £$  
 
Q7.  A  person  has  to catch a train that is scheduled to depart at ‘@%’. It takes the person 4 hours 
and  15  minutes  to  reach  the  railway  station  from  his  home.  At  what  time  should  he leave from 
his  home  for  the  railway  station  to  arrive  at the station at least 25 minutes before the departure 
of the train?  
(a) %@ (b) £$ (c) %+ (d) +@ (e) None of these 
 
 
 
 

Plutus Academy-(219)
ANSWERS 
1.   (e);  In  the  above  question  it  is  asked  to  choose  from  a  given  option  which  weaken  the 
statement or make it imperfect so correct answer will be (e). 
2. Ans.(d) Sol. As stated in statement America is not taking any steps to address the issue, 
so to calm down the situation Indian Govt. may pressurize American Govt. or they may 
be impact on Indian-US trade. And as stated in statement America is not taking any 
action despite of racism attack on Indians which results death of two citizens. So it 
cannot be assumed that American Govt. will give life imprisonment according to the 
statement.  
3. Ans.(c) Sol. All of three supports the negligence of Indian-American Govt. as Modi Govt. 
denies replying in same week after racial attack. American Govt. is not ready tackle the 
issue despite of environment of racism is spreading. 
4. Ans.(d)  Sol.  Most  logically  such  comparison  should  reveal  mortality  rate  per  thousand 
doctors  indulged  in  SARS  treatment  and  not  indulged  in  treatment.so  statement  (d)  is 
the correct option. 
5. Ans.(d)  Sol.  train  is  scheduled  at=  5  hour  10  minute  =  #&  Time should he leave to reach 
the station 5 minute earlier= 5 hour 10 minute-(25+5) minute= 4 hour 40 minute  
6. Ans.(a) Sol. 4 hour 15 minute  
7. Ans.(b)  Sol.  Scheduled  time  of  departure  of  train=  @%=  8:00  PM  Time  for  travel+  early 
arrival=  4  hour  15  minutes  +  25  minutes  =  4  hour  40  minutes  8:00  PM-  4  hour  40 
minutes= 3:20 PM= £$ 
 
 

Plutus Academy-(220)

You might also like